Constitutional Law Chemerinsky/Outline I

From wikilawschool.net. Wiki Law School does not provide legal advice. For educational purposes only.
Constitutional Law I
Authors Erwin Chemerinsky
Text Image of Constitutional Law: [Connected eBook with Study Center] (Aspen Casebook)
Constitutional Law: [Connected eBook with Study Center] (Aspen Casebook)
Taught by
Taught at
Related course(s)

THE FEDERAL JUDICIAL POWER[edit | edit source]

'The Authority for Judicial ReviewThe Source of Federal Judicial Power: Article III, Section 1 provides that “the judicial power of the United States shall be vested in one Supreme Court and in such inferior Courts as the Congress may from time to time ordain and establish.”The Scope of Federal Judicial Power: Article III, Section 2 limits the jx of the federal courts to:'

  • Ø Cases, in law and equity, arising under the U.S. Constitution, federal law, and treaties,'
  • Ø Cases affecting ambassadors, public ministers, and consuls,'
  • Ø Cases of admiralty and maritime jdx,'
  • Ø Controversies in which the United States is a party;'
  • Ø Controversies between two or more states,'
  • Ø Cases between a state and a citizen of another state, and'
  • Ø Cases between citizens of different states (diversity of citizenship).'

ORIGINAL JURISDICTION: The Supreme Court’s power to be the initial court to hear certain types of cases. Under Article III of the Constitution, the Supreme Court has original jurisdiction of all cases affecting ambassadors, other public ministers and consuls, and cases in which a state is a party. [“...in all cases affecting ambassadors, other public ministers and consuls and those in which a state shall be a party.”] APPELLATE JURISDICTION: The Supreme Court’s power to review decisions of other courts. Article III of the Constitution gives the Supreme Court appellate jurisdiction of, among other cases, all cases arising under the Constitution, laws, and treaties of the United States.

  • Exceptions and Regulations Clause
    • o Congress cannot enlarge or restrict the Supreme Court’s original jurisdiction (Marbury v. Madison).
    • o Congress can expand or limit the Supreme Court’s appellate jurisdiction by passing relevant legislation. Congress can also create and regulate the jurisdiction of the lower federal courts, subject to the power and limitations set forth in Articles I and III. (Example: Judiciary Act of 1789)

FEDERALISM: Federalism is the vertical allocation of power between the federal government and the states. The Constitution is structured so that each branch of government has defined powers that may not be encroached on by other branches, with a set of checks built in to keep any one branch from usurping too much authority, or ‘aggrandize itself at the expense of the other branches.’ (See The Federalist No. 49).COUNTER-MAJORITARIAN RULE (perceived problem with judicial review of legislative (or popularly created laws)): Marburywas a massively important contribution to jurisprudence, but it also raises an important question about the antidemocratic nature of a court of unelected judges invalidating the act of a democratically elected legislature. This is referred to as the counter-majoritarian difficulty. The judiciary is meant to function as an independent check on the democratic arms of government. Article II gives the president the authority to appoint federal judges, with the advice and consent of the senate. Those judges, at all levels of the federal court system, are given lifetime appointments with removal only for bad behavior. Judges are meant to be immune to the impulses of the majority.LAST-IN-TIME RULE: The last in time rule makes clear that where two statutes or a treaty and a statute conflict, the one last enacted will prevail. ARTICLE III

  • Ø Section 1: creates Supreme Court
  • Ø Section 2: defines judicial power
    • o Cases “arising under” federal law
    • o Admiralty
    • o Controversies b/w states
    • o Diversity jurisdiction
    • o Foreign state or citizens
  • Marbury v. Madison (1803) [Page 2]
    • o TOPIC: Power of JR. Authority for judicial review.
    • o FACTS: William Marbury (P) was appointed as a justice of the peace at the very end of John Adams’ presidency. Thomas Jefferson, the incoming president, chose to disregard the appointments because formal commissions had not been delivered before the end of Adams’ term. Marbury (P) and others took their case to the Supreme Court, seeking a writ of mandamus [order directing that an official perform an act] that would order Madison (D), Jefferson’s Secretary of State, to deliver the commissions. John Marshall was Secretary of State under Adams, but had since been appointed Chief Justice of the Supreme Court by the time the Court heard the case.
    • o RULE: Under Marbury v. Madison, federal courts have the authority to review acts of Congress and the Executive Branch (executive orders, but not veto’s of a bill or appointees) and they have the power to invalidate those acts that violate the Constitution.
    • o ANALYSIS: Marbury does 3 things: 1). Creates authority for judicial review; 2). Establishes that Art. III is the ceiling of federal court jx (Congress cannot expand this); and 3). Establishes the authority for judicial review of legislative/executive acts and the ability to rule them unconstitutional
      • Declared §13 of the Judiciary Act of 1789 (which granted the Supreme Court power to issue writs of mandamus to all persons holding office in the US) conflicted with Art. III and was unconstitutional.
      • Chief Justice Marshall: It is inherent to the judicial role to decide the constitutionality of the laws that it applies. “It is emphatically the province and duty of the judicial department to say what the law is.”
      • Borrows language from Fed. No. 78: The powers of the legislature are defined and limited; and that those limits may not be mistaken or forgotten, the constitution is written.
      • 3 questions: (1) Does Marbury have a right to the commission? (2) If he has a right, and his right to the commission has been violated, do the laws afford Marbury a remedy? (3) Can the Supreme Court issue this remedy? Is mandamus an appropriate remedy?
        • (1) Marbury had a right to the commission because all appropriate procedures were followed.
          • o “It is … decidedly the opinion of the court, that when a commission has been signed by the President, the appointment is made; and that the commission is complete, when the seal of the United States has been affixed to it by the Secretary of State . . . To withhold the commission, therefore, is an act deemed by the court not warranted by law, but violative of a vested legal right.”
        • (2) Yes. The judiciary could provide remedies against the executive when there is a specific duty to a particular person, but not when it is a political matter left to executive discretion.
          • o “[W]here the heads of departments are the political or confidential agents of the executive, merely to execute the will of the President, or rather to act in cases in which the executive possesses a constitutional or legal discretion, nothing can be more perfectly clear than that their acts are only politically examinable. But where a specific duty is assigned by law, and individual rights depend upon the performance of that duty, it seems equally clear that the individual who considers himself injured, has a right to resort to the laws of his country for a remedy.”
        • (3) Yes, the Supreme Court can issue this remedy. Some matters—such as whether to veto a bill or whom to appoint for an office—are entirely within the president's discretion and cannot be judicially reviewed. But where the executive has a legal duty to act or refrain from acting, the federal judiciary can provide a remedy, including a writ of mandamus.
          • o BUT: grant of original jurisdiction by 13 of the Judiciary Act of 1789 was unconstitutional.
        • In resolving the question, Marshall effectively defined judicial review: “It is emphatically the province and duty of the judicial department to say what the law is. Those who apply the rule to particular cases, must of necessity expound and interpret that rule. If two laws conflict with each other, the courts must decide on the operation of each.” Further, “A law repugnant to the Constitution is void.”
  • Remember that Article III grants og jx in “Cases affecting Ambassadors, other Public Ministers and Consuls, and those in which a State shall be a party.”* The canon of constitutional avoidance says that when a statute may be reasonably interpreted in more than one way, courts should choose the construction that avoids raising a constitutional question. Supreme Court can review state decisions; established by 2 early 19th century casesRule: Under Article III of the United States Constitution, the United States Supreme Court has authority to exercise appellate review of state court decisions (Martin v. Hunter’s Lessee [land dispute case] and Cohens v. Virginia [holding criminal defendants can seek Supreme Court review on claims that their convictions are unconstitutional]).Note: This is an example of appellate jurisdiction that cannot be limited by Congress.
  • *Martin v. Hunter’s Lessee (1816) [Page 10]
    • o TOPIC: Power of JR. Authority for Supreme Court to review state court judgments.
    • o FACTS: Virginia confiscated land owned by Martin (P), a British subject, and granted it to Hunter. Martin (P) claimed the confiscation was ineffective under treaties between the United States and England. The Virginia Court of Appeals held in favor of Hunter’s Lessee (D), and the United States Supreme Court reversed and remanded with instructions to enter judgment in favor of Martin (P). The Virginia court refused to comply with the Supreme Court’s order on remand, finding § 25 of the Judiciary Act [authorizes the Supreme Court’s exercise of appellate jurisdiction over cases pending in state courts] unconstitutional because it placed the courts of one sovereign under the direct control of another.
    • o RULE: Under Martin v. Hunter’s Lessee, the constitution permits the Supreme Court to exercise appellate jurisdiction over cases pending in state courts.
    • o ANALYSIS: Justice Story explained the importance of Supreme Court review of state courts. Justice Story said that although he assumed that “judges of the state courts are, and always will be, of as much learning, integrity, and wisdom as those of courts of the United States,” the Constitution is based on a recognition that “state attachments, state prejudices, state jealousies, and state interests might sometimes obstruct, or control, or be supposed to obstruct or control, the regular administration of justice.”
      • If the Supreme Court did not have this power, then they would be powerless to hear any cases, except for the few which fit its OG.
  • *Cohens v. Virginia (1821) [Page 11]
    • o TOPIC: Power of JR. Reaffirmed Supreme Court can review state court judgments/states can’t be trusted to protect federal rights so Supreme Court can review when individuals think their Constitutional rights are violated.
    • o FACTS: Two brothers were convicted in Virginia state court of selling District of Columbia lottery tickets in violation of Virginia law. The DFDs sought review in the SCOTUS because they claimed the Constitution prevented their prosecution for selling tickets authorized by Congress. Virginia argued: (1) in general, the Supreme Court had no authority to review state court decisions; and (2) in particular, review was not allowed in criminal cases and in cases where a state government was a party.
    • o RULE: Under Cohens v. Virginia, the Supreme Court reaffirmed the constitutionality of §25 of the Judiciary Act and the authority of the Supreme Court to review state court judgments.
    • o ANALYSIS: The Court emphasized that state courts often could not be trusted to adequately protect federal rights because “[i]n many States the judges are dependent for office and for salary on the will of the legislature.” The Court thus declared that criminal defendants could seek Supreme Court review when they claimed that their conviction violated the Constitution.

State Jurisdiction

  • Ø To avoid federal review:
    • o Must “clearly and expressly” state
    • o Wholly based on state law
  • Ø Supreme Court can only hear case after
    • o Ruling by highest state court, or
    • o Party has exhausted all state remedies
  • Ø May hear federal cases for comity (The legal principle that political entities (such as states, nations, or courts from different jurisdictions) will mutually recognize each other’s legislative, executive, and judicial acts. The underlying notion is that different jurisdictions will reciprocate each other’s judgments out of deference, mutuality, and respect.)
  • Ø Judiciary Act of 1789, §25
    • o Three criteria for Supreme Court jurisdiction over state court judgment on appeal:
      • The validity of a federal statute or state action must be drawn into question, or the construction of a federal law is at issue such that the state court’s construction has drawn a federal privilege or right into question;
      • The federal law or state action must be drawn into question on grounds that it is repugnant to the constitution, treaties and laws of the United States.
      • The decision of the state court must be against the validity of the federal law or in favor of the validity of the state action, or its construction of a federal law must be against a privilege or right under the federal law.
    • o Notes about §25
      • No general federal question jurisdiction granted to lower federal courts until 1875.
      • Even some federal issues excluded from Supreme Court review when the state “over-vindicated” the claim.
      • Review at the instance of either side of federal issues was not clearly available until 1914.
      • A state may challenge the constitutionality of a federal statute by filing a lawsuit in court seeking to declare the federal law unconstitutional. Such a lawsuit is decided by the courts, with the Supreme Court having final jurisdiction. (Nullification)
        • Technically, any court can issue a ruling that a law is unconstitutional. That ruling would serve as binding precedent within the jurisdiction of that court. However, the only court that has the ultimate, binding say on the matter is the U.S. Supreme Court for national constitutional matters, or the individual State Supreme Courts for state constitutional matters.

Limits on the Federal Judicial Power[edit | edit source]

Interpretive Limits:How should the Constitution be interpreted? Static – Originalism; Dynamic – Non-Originalism

  • Ø Originalism: judges deciding constitutional issues should confine themselves to enforcing norms that are stated or clearly implicit in the written Constitution. [Narrow judicial power]
  • Ø Non-Originalism: the view that courts should go beyond that set of references and enforce norms that cannot be discovered within the four corners of the document, since the Constitution should evolve [Broad judicial power]
  • Ø Judiciary Act of 1789, §25: Three criteria for Supreme Court jurisdiction over state court judgment on appeal:
    • o The validity of a federal statute or state action must be drawn into question, or the construction of a federal law is at issue such that the state court’s construction has drawn a federal privilege or right into question;
    • o The federal law or state action must be drawn into question on grounds that it is repugnant to the constitution, treaties and laws of the United States.
    • o The decision of the state court must be against the validity of the federal law or in favor of the validity of the state action, or its construction of a federal law must be against a privilege or right under the federal law.

District of Columbia v. Heller (2008)[edit | edit source]

[Page 13]

    • o TOPIC: Limitations on JR. Interpretative limits: clauses in a Const. Amend. must be read together
    • o FACTS: District of Columbia (D) ordinances essentially prohibited the possession of handguns. In addition, no person could carry a handgun without a license, but the Chief of Police was authorized to issue licenses for one-year periods. Guns kept in a person’s home were required to be stored in a way that made them inoperable. Heller (P) was a District of Columbia special police officer. He was authorized to carry a handgun while on duty at the Federal Judicial Center. Heller (P) applied to register a handgun to keep at home, but the District (D) refused. Heller (P) then filed suit in the district court seeking to enjoin the District (D) from enforcing its gun laws on Second Amendment grounds.
    • o RULE: Words and phrases in the Constitution are to be interpreted according to their normal and ordinary meanings as understood when the provision in question was adopted.
    • o ANALYSIS: Justice Scalia, writing for the majority, relies on a historical interpretation to determine the original intent of the Second Amendment. Justice Stevens would rely more heavily on precedent. Justice Breyer would emphasize deference to the judgment of the legislature.
      • So which approach should you take? Operative and prefatory clauses read together or separately? Context? Intent of the framers? The history of the time? Words having different meanings over time?
      • Prefatory clause – announces a purpose; operative clause – the function, the rule having effect

Congressional Limits[edit | edit source]

The ability for Congress to restrict federal court jurisdictionIssue: May Congress strip the jurisdiction of the court?Rule: In cases that don’t affect ambassadors, other public ministers and consuls, and those where a state is a party, the court’s appellate jurisdiction is limited to the exceptions and regulations that Congress makes.Congressional Checks to Judicial Power

  • Ø Congress can create or abolish
  • Ø Congress decides types of cases
    • o But may not expand beyond constitutional limits as outlined in Art. III
  • Ø Removal of judges
  • Ø Impeachment power

SEPARATION OF POWERS AS A LIMIT:

  • Rule: Based on the principle of separation of powers in the United States Constitution, the legislative branch may not impair or direct the exclusive powers of the judicial or executive branches (United States v. Klein [legislative enactment requiring the Supreme Court to find that it had no jx if it found that a plaintiff was entitled to property rights on a presidential pardon].
  • Marcozzi- limits Klein by saying that Congress can change the statutory basis of a decision, but it cannot change the rules of statutory interpretation.
  • Rule: Congress may not pass retroactive legislation that has the effect of forcing the courts to reopen final judgments, as this violates the separation of powers in the Constitution (Plaut v. Spendthrift).
ART III’s Exceptions and Regulations Clause – What does it mean?
  • Ø “Supreme Court shall have appellate jurisdiction, both as to Law and Fact, with such Exceptions, and under such Regulations as the Congress shall make.”
  • Ø Argument: If Congress has broad powers to remove matters from SCOTUS scope of influence?
    • o Check on judiciary power
    • o Congress didn’t vest SCOTUS with appellate jx over all types of cases
  • Ø Argument: If Congress is limited in its ability to control SCOTUS jx?
    • o Framers concerned about SCOTUS ability to overturn fact-finding by lower courts.
    • o Congress can create exception to SCOTUS jurisdiction re: matters of fact, but can’t eliminate SCOTUS jurisdiction re: matters of law
  • Ø Congress can only limit SCOTUS jx in so far as Article III allows! (Could it be that the separation of powers limits the ability of Congress to restrict SCOTUS jx?”

HABEAS CORPUS: A judicial mandate to a prison official ordering that an inmate be brought to the court so it can be determined whether or not that person is imprisoned lawfully and whether or not he should be released from custody. A habeas corpus petition is a petition filed with a court by a person who objects to his own or another’s detention or imprisonment. Latin for “you have the body.”Habeas Corpus Acts of 1867 and 1868

  • (1867) Be it enacted . . . That the several courts of the United States, and the several justices and judges of such courts, within their respective jurisdictions. . . shall have power to grant writs of habeas corpus in all cases where any person may be restrained of his or her liberty in violation of the constitution, or any treaty or law of the United States. . . . Appeal may be to . . . The Supreme Court of the United States.
  • (1868) Be it enacted . . . That so much of the act approved February 5, 1967, . . . as authorizes an appeal from the judgment of the circuit court to the Supreme Court, or the exercise of any such jurisdiction by said Supreme Court on appeals which have been or may hereafter been taken, be, and the same is, hereby repealed.
  • Ex Parte McCardle (1868) [Page 34]
    • o TOPIC: Limitations on JR. Congress can create exceptions and regulations to Supreme Court appellate jx.
    • o FACTS: In 1867, Congress gave federal courts jurisdiction to grant habeas corpus where a person may be restrained in violation of federal law. The 1867 Act also authorized appeals to the Supreme Court in cases where circuit courts denied applications for a writ of habeas corpus. McCardle (P) was a Mississippi newspaper editor imprisoned for publishing “incendiary and libelous” articles tending to incite violence and impede Reconstruction. The Circuit Court denied McCardle’s (P) petition for habeas corpus, so he appealed to the Supreme Court under the 1867 Act. The Court heard arguments in McCardle’s (P) case. However, while the case was pending in the Supreme Court, Congress passed a new law repealing the part of the 1867 Act that permitted Supreme Court appellate review of writs of habeas corpus. President Andrew Johnson vetoed this legislation, but Congress immediately overrode his veto and reinstated its repeal of the 1867 Act. The Supreme Court then considered whether it had jurisdiction to hear McCardle's constitutional claims in light of the recently adopted statute denying it authority to hear appeals under the 1867 Act that was the basis for jurisdiction in McCardle's petition.
    • o RULE: Under Ex parte McCardle and Article III of the Constitution, though the Supreme Court’s appellate jurisdiction is derived itself from the Constitution, Congress has the power to make exceptions and regulations to this jurisdiction.
    • o ANALYSIS: Congress grants the Court jurisdiction; it does not limit jurisdiction that the Constitution granted to the Court. We cannot inquire into Congress’ motives for repealing the 1867 Act.
      • This case arose in the political context of post-Civil War Reconstruction Through the Military Reconstruction Act, Congress imposed a type of military rule on ten former Confederate states. The constitutionality of the Military Reconstruction Act was questionable, and Congress knew it. The Supreme Court had hinted in a previous case that it might strike down the Act, so Congress stripped the Supreme Court of the power to rule just as the Court was about to render a decision on McCardle’s (P) petition. President Andrew Johnson vetoed Congress’s repeal of jurisdiction, but Congress overrode the veto. Note that the effect of McCardle may have been minimal. Under previous legislation, the Court had jurisdiction through discretionary review by writ of certiorari. So the Court had discretionary review, but not appellate review. So too, McCardle (P) could have petitioned the Court for an original writ of habeas corpus, rather than appellate review of the Circuit Court’s denial of his petition.
      • In Ex Parte Yerger, another case challenging the Reconstruction Acts, Yerger filed a habeas corpus petition based on pre-1867 legislation. After the Supreme Court upheld its jurisdiction to decide Yerger’s constitutional claims, the federal military authorities dismissed all charges against him, thereby again preventing Supreme Court review of the constitutionality of Reconstruction.
      • McCardle establishes that Congress may prevent Supreme Court review of constitutional issues. The fact that Congress intends to change the substantive law by limiting jurisdiction is deemed irrelevant, for they quote the McCardle Court's statement that “[w]e are not at liberty to inquire into the motives of the legislature. We can only examine into its power under the Constitution; and the power to make exceptions to the appellate jurisdiction of this court is given by express words.”
  • United States v. Klein (1871) [Page 36]
    • o TOPIC: Limitation on JR. Congressional limitations on judicial review/jx are ineffective IF they violate SOP.
    • o FACTS: Just after the Civil War, in 1863, Congress passed a law that allowed individuals to sue the federal government in the Court of Claims to recover property seized during the war. To be successful, the claimant had to prove that he had not offered to give aid or comfort to the enemy. The U.S. Supreme Court ruled that a presidential pardon could be used as proof in these cases. As there were many of these pardons, Congress reacted to the Supreme Court’s ruling by passing a bill which stated that, absent an explicit statement of innocence, pardons would have the effect of proving that the pardoned individual had aided the enemy. The statute also provided that federal court jurisdiction over the case would terminate upon “proof of such a pardon.” Klein (P) sued for recovery of property seized during the war and won the case in the Court of Claims. The pardon at issue in the case had been granted to an intestate to whom Klein (P) was the heir with regard to the property in question. The case was appealed to the U.S. Supreme Court. The Supreme Court held that the statute was unconstitutional. While acknowledging Congress's power to create exceptions and regulations to the Court's appellate jurisdiction, the Supreme Court said that Congress cannot direct the results in particular cases.
    • o RULE: Congress violated the separation of powers by passing a law rescinding the Supreme Court’s appellate jurisdiction in claims cases supported by a presidential pardon and by infringing the president’s exclusive power to pardon.
      • Based on the principle of separation of powers in the United States Constitution, the legislative branch may not impair or direct the exclusive powers of the judicial or executive branches.
    • o ANALYSIS: The law in question violates the separation of powers for two reasons. First, the law surpasses Congressional power to create exceptions and regulations to the Supreme Court’s appellate jurisdiction. Second, the law interferes with the Executive’s exclusive power to pardon, a power granted without limit by the Constitution. The Constitution, in Article 3, section 2, vests judicial power exclusively within the federal courts, and gives the Supreme Court appellate jurisdiction, subject to exceptions and regulations made by Congress. Because Congress has already given the Supreme Court appellate jurisdiction over Court of Claims cases, this law has only the effect of directing particular outcomes of cases already pending. This is not a proper exercise by Congress of the power to create exceptions and regulations on appellate jurisdiction. Under the law at question, the Supreme Court has jurisdiction over a case until the existence of a pardon is established, then must dismiss due to lack of jurisdiction, effectively deciding the outcome of the case. If the Congress had decided to deny appeals of all Court of Claims cases, it could have done so. Here, however, the language of the statute shows that the intent of Congress was to nullify the effects of the presidential pardons.
      • Opponents of jurisdiction stripping contend that Klein supports their position that Congress cannot restrict Supreme Court appellate review in an effort to direct particular substantive results.
      • Second, it can be argued that the statute in Klein unconstitutionally deprived property without just compensation or due process. Under the previous law, those pardoned had a vested right to the return of their property that had been seized. But the denial of jurisdiction prevented the federal courts from vindicating their protected property interest. Hence, the statute was unconstitutional.
      • Two features distinguish Klein
        • First, in the statute at issue in Klein, Congress was redefining the president's pardon power. The statute was arguably unconstitutional as an infringement of the executive's power under Article II of the Constitution.
        • Second, it can be argued that the statute in Klein unconstitutionally deprived property without just compensation or due process. Under the previous law, those pardoned had a vested right to the return of their property that had been seized. But the denial of jurisdiction prevented the federal courts from vindicating their protected property interest. Hence, the statute was unconstitutional.
  • Robertson v. Seattle Audobon Society (1992) [Page 38]
    • o TOPIC: Limitation on JR. Congressional limits are valid if they change the law and don’t direct the outcome.
    • o FACTS: The petitioners in this case appealed the Ninth Circuit court's ruling that stated that the Department of the Interior and Related Agencies Appropriations Act was unconstitutional. The petitioners contended that the lower court had improperly ruled that §318 violated Article III of the Constitution of the United States by impermissibly directing a specific judicial decision without amending or repealing the statutes that are underlying the litigation.
      • The Act both required the Bureau of Land Management to offer specified land for sale and also imposed restrictions on harvesting from other land. Additionally, the Act expressly noted two pending cases and said that “Congress hereby determines and directs that management of areas according to subsections (b)(3) and (b)(5) of this section on [the specified lands] is adequate consideration for the purpose of meeting the statutory requirements that are the basis for [the two lawsuits].”
    • o RULE: Congressional limit on JR is valid if Congress is not directing the outcome of pending litigation, but instead changing the law in question.
    • o ANALYSIS: The Supreme Court held that §318 did not violate Article III because it made changes to the law by replacing the legal standards of the five challenged statutes. It did not make any findings of law or fact, or compel any results in the pending cases. The explicit reference to the cases only served to identify the statutes underlying those cases.
      • The Ninth Circuit held that this provision was unconstitutional under Klein because Congress was directing the outcome of the pending litigation.
      • The Supreme Court disagreed, concluding that Congress had changed the law itself and did not direct findings or results under the old law. The Court read Klein as applying in a situation where Congress directs the judiciary as to decision-making under an existing law and not applying when Congress adopts a new law. By placing the Act into the latter category, the Court found Klein distinguishable and rejected the constitutional challenge.
  • Bank Markazi v. Peterson (2016) [Page 39]
    • o TOPIC: Limitation on JR. Congressional limit is valid if Congress makes
    • o FACTS: American nationals may seek money damages from state sponsors of terrorism in the courts of the United States. Prevailing plaintiffs, however, often face practical and legal difficulties enforcing their judgments. To place beyond dispute the availability of certain assets for satisfaction of judgments rendered in terrorism cases against Iran, Congress enacted the Iran Threat Reduction and Syria Human Rights Act of 2012. The Act makes a designated set of assets available to satisfy the judgments in a specific case, designated by docket number. Plaintiffs brought suit against Iran. They obtained default judgments and moved for the turnover of about $1.75 billion in bond assets held in a New York bank account allegedly owned by Bank Markazi, the Central Bank of Iran. The Bank argued that the federal statute violated separation of powers because it usurped the judicial role by directing a particular result in the pending enforcement proceeding.
    • o RULE: While Article III bars Congress from telling a court how to apply preexisting law to particular circumstances, Congress may amend a law and make the amended prescription retroactively applicable in pending cases. Congress can change the terms of the statute any time until there is a final judgment.
    • o ANALYSIS: The statute does just that: It requires a court to apply a new legal standard in a pending post-judgment enforcement proceeding. It does not matter that it meant that the result in the case was a “foregone conclusion.” A statute does not impinge on judicial power when it directs courts to apply a new legal standard to undisputed facts. Nor is the statute invalid because it prescribes a rule for a single, pending case identified by caption and docket number (and actually it was 16 cases).
      • The Court, per Justice Ginsburg, 6–2, held that Congress may enact new legislation which has the effect of changing the outcome in a pending civil case and that such legislation does not violate separation of powers.
      • In Klein, the Court had declared that Congress cannot “prescribe rules of decision to the Judicial Department in . . . [pending] cases.” However, Justice Ginsburg ruled that the prohibition in Klein does not apply when Congress enacts new legislation. The Court ruled that a statute does not trespass on judicial power “when it directs courts to apply a new legal standard to undisputed facts.”
      • Chief Justice Roberts, joined by Justice Sotomayor, dissenting, contended that the statutory provision was unprecedented because it “changed the law for a pending case in an outcome-determinative way and limited its effect to particular judicial proceedings.” As such, it violated the basic precept of Article III that judicial power is rested solely in the Judicial Branch.

*These cases establish no clear principles as to what the phrase “exceptions and regulations” means or when separation of powers prevents Congress from changing the law in response to a Supreme Court decision interpreting a statute. Ultimately, the arguments about Congress’s ability to check the federal judiciary, like so many areas of ConLaw, turn on disputes about the meaning of the Constitution’s language, the intent of the framers, and competing policy considerations.

Justiciability Limits[edit | edit source]

Judicially created limits on the matters that can be heard in federal courts.

  • 1) Prohibition of advisory opinions; 2) Standing; 3) Ripeness; 4) Mootness; 5) Political Q Doctrine

JUSTICIABILITY: The quality or state of being appropriate or suitable for review by a court.

  • Ø Doctrines founded in concern about the proper—and properly limited—role of the courts in a democratic society. Warth v. Seldin.
  • Ø Article III of the Constitution limits federal jurisdiction to “cases” and “controversies.” The justiciability doctrines define the separation of powers with respect to the judicial branch, stating fundamental limits on the power of an unelected, unrepresentative federal judiciary.
  • Ø “Constitutional” requirements meaning that Congress by statute cannot override them.
  • Ø “Prudential” requirements meaning that they are based on prudent judicial administration and can be overridden by Congress since they are not constitutional requirements.
  • Ø These doctrines raise basic policy q’s about the role of the federal judiciary in a democratic society.
  • Ø Chief Justice Warren explained, the “words [cases and controversies] define the role assigned to the judiciary in a tripartite allocation of power to assure that the federal courts will not intrude into areas committed to the other branches of government.”

RAMPS: Ripeness, Advisory Opinions, Mootness, Political Questions, Standing

[Exam Tip! Ripeness tends to be an issue when the claimant is seeking declaratory relief or when a statute has been enacted, but not yet enforced.]

MOOTNESS DOCTRINE: Prohibits courts from deciding issues that are only abstract and do not involve a real dispute, or which have already been resolved.POLITICAL QUESTION DOCTRINE: Requires courts to refrain from deciding issues which are more properly resolved by the other branches of government.RIPENESS DOCTRINE: Requires courts to decide only issues which involve a real dispute and an actual injury, and not merely potential or speculative harm.STANDING: The status of being qualified to assert legal rights in court because one has a sufficient stake in the outcome of the controversy.

  • Case or Controversy
    • o Case must not be an advisory opinion'
    • o Case must have standing
    • o Case must meet moot' and ripe standard'
  • In addition to the justiciability doctrines, the Supreme Court has said that it would follow certain “principles of avoidance” to ensure that it will reach constitutional questions only when necessary. Ashwander v. Tennessee Valley Authority, Justice Brandeis wrote:
    1. The Court will not determine the constitutionality of legislation in non-adversarial proceedings; this pretty much ended any hopes for "advisory" rulings from the Court.
    2. The Court will not anticipate a question of constitutional law.
    3. The Court will not formulate a rule of constitutional law which is broader than needed.
    4. The Court will not rule on constitutionality where there is another ground for deciding the case.
    5. The Court will not determine a statute's constitutionality unless a party has been injured by it.
    6. The Court will not invalidate a statute at the request of parties who have taken advantage of its benefits.
    7. The Court will always consider whether any reasonable interpretation of a statute allows it to avoid the constitutional issues.

a). Prohibition of Advisory Opinions: there must be an actual dispute between adverse litigants.ADVISORY OPINIONS: A decision by a court that is not binding on the parties to the case or that may be ignored by those charged with enforcing the decisions.

  • For a case to be justiciable and not an advisory opinion, two criteria must be met.
  • o First, there must be an actual dispute between adverse litigants.
  • o Second, in order for a case to be justiciable and not an advisory opinion, there must be a substantial likelihood that a federal court decision in favor of a claimant will bring about some change or have some effect.
  • Advisory Opinion
    • 'Issue': The issue is whether the complaint involves an advisory opinion.
    • 'Rule': An advisory opinion exists when there isn’t a dispute between two opposing parties, not to issue opinions that infringe on the power of other co-equal branches of government.
    • Application': Here, this [x] will [likely/unlikely] constitute an advisory opinion because the issues [is'/is not] concrete, [is/is not] hypothetical, because [x].
    • Conclusion':' Here, it is likely that this complaint involves/does not involve an advisory opinion.
  • Federal courts will not give advisory opinions to either the president or Congress concerning the constitutionality of proposed action or legislation; there has to be a real case or controversy.
    • DECLARATORY JUDGMENTS are permitted. Declaratory judgments are those that state the legal effect of a regulation or the conduct of parties in regard to a controversy.
    • State courts can issue advisory opinions.
  • Hayburn’s Case (1792) [Page 42]
    • o TOPIC: Advisory opinions
    • o ANALYSIS: Congress adopted a law permitting Revolutionary War veterans to file pension claims in the U.S. Circuit Courts. The judges of these courts were to inform the secretary of war of the nature of the claimant’s disability and the amount of benefits to be paid. The secretary could refuse to follow the court’s recommendation.
      • Five Supreme Court justices, while sitting as circuit judges, found this approach unconstitutional. The justices explained that the duty of making recommendations regarding pensions was “not of a judicial nature.” They said that it would violate separation of powers because the judicial actions might be “revised and controlled by the legislature, and by an officer in the executive department. Such revision and control we deemed radically inconsistent with the independence of that judicial power which is vested in the courts.”
  • Plaut v. Spendthrift Farm (1995) [Page 43]
    • o TOPIC: Advisory opinions; federal courts are only empowered to decide “cases and controversies”
    • o FACTS: In a 1991 ruling, the Supreme Court held that a class action securities case filed under federal statutes had a limitation period of three years from the time of the alleged violation and one year from the time that the facts of the violation became known. Subsequently, Congress passed legislation [Section 27A(b)] stating that cases within the aforementioned class that were filed prior to the 1991 ruling could proceed if the case was permissible under the former law.
    • o RULE: Legislation that directs the federal courts to reopen cases on which the courts have passed final judgment unconstitutionally violates the separation of powers.
      • Congress may not pass retroactive legislation that has the effect of forcing the courts to reopen final judgments, as this violates the separation of powers in the Constitution.
    • o ANALYSIS: Section 27A(b) violates the separation of powers doctrine because it requires the courts to decide that the law that applied to a completed case was different than the courts concluded it was. Prior to this case, the Supreme Court has held that two categories of cases are unconstitutional because they would have the federal courts act contrary to the judicial power articulated in Article III. The first case wasUnited States v. Klein,which held unconstitutional legislation that prescribed the outcomes of cases pending in the federal courts. InRobertson v. Seattle Audubon Soc,however, the Court clarified that legislation that amends the applicable law is constitutional. The second case was Hayburn’s Case, which held that Congress may not pass legislation that permits Executive review of federal court decisions. The law in the present case does not fall under either of these categories. Nevertheless, Section 27A(b) violates the separation of powers because it requires the courts to reopen cases on which a final judgment has already been rendered. This violates the Article III principle that “gives the Federal Judiciary the power, not merely to rule on cases, but to decide them.” Article III establishes a judicialdepartment,and vests the Supreme Court with the final voice on cases heard by the department. If the law is changed while cases are still being appealed, the higher courts are required to apply the new law. In this case, however, the last word had been given, and Congress cannot now require the federal courts to change that message.
      • If Section 27A(B) remained in effect, all the previous rendered decisions would become advisory opinions.
  • Nashville, C. & St. L. Ry. v. Wallace (1933) [Page 45]
    • o FACTS: A company sought a declaratory judgment that a tax was an unconstitutional burden on interstate commerce. The Supreme Court explained that because the matter would have been justiciable as a request for an injunction, the suit for a declaratory judgment was capable of federal court adjudication.
    • o ANALYSIS: “The Constitution does not require that the case or controversy should be presented by traditional forms of procedure, invoking only traditional remedies. [Article III] did not crystallize into changeless form the procedure of 1789 as the only possible means for presenting a case or controversy.” The Court emphasized that the focus was on “substance” and “not with form” and that the case was justiciable “so long as the case retains the essentials of an adversary proceeding, involving a real, not a hypothetical, controversy.”

b). Standing[edit | edit source]

  • v The Supreme Court has declared that standing is the most important justiciability requirement.
  • v Standing is the determination of whether a specific person is the proper party to bring a matter to the court for adjudication.
    • o “In essence the question of standing is whether the litigant is entitled to have the court decide the merits of the dispute or of particular issues.”
  • v There are three constitutional standing requirements.
    • o First [INJURY], the plaintiff must allege that he or she has suffered or imminently will suffer an injury.
    • o Second [CAUSATION], the plaintiff must allege that the injury is fairly traceable to the defendant’s conduct.
    • o Third [REDRESSABILITY], the plaintiff must allege that a favorable federal court decision is likely to redress the injury.
  • v There are two major prudential standing principles. Unlike constitutional barriers, Congress may override prudential limits by statute.
    • o First, a party generally may assert only his or her own rights and cannot raise the claims of third parties not before the court.
    • o Second, a plaintiff may not sue as a taxpayer who shares a grievance in common with all other taxpayers.

** ONLY ONE PLAINTIFF NEEDS TO MEET THE REQUIREMENTS FOR THERE TO BE STANDING **'''''Issue: The issue is whether [x] has standing.Rule': A party will have standing if they (1) suffer an injury in fact that, (2) caused by the governmental action, and (3) the injury may be redressed by a favorable decision (Lujan). Each plaintiff’s claim must be analyzed individually to sufficiently address the ability for the court to hear the complaint.

  • Elements (Constitutional Requirements):
    • Injury in-fact;'
    • Causation; and'
    • Redressability'
    • ** ONLY ONE PLAINTIFF NEEDS TO MEET THE REQUIREMENTS FOR THERE TO BE STANDING **
    • Organizations will not have standing w/o member-plaintiff

Application': Injury in fact (do this individually for each party)'''First: To satisfy the injury in fact requirement, a plaintiff must have suffered or imminently will suffer a sufficient, concrete, injury. First, [x]’s purported [state which type of injury] will likely be sufficient because [x]. [for this sentence, plug in one of the responses under the sufficient injuries]. However, if [x]’s injury is only deriving from the government simply failing to follow the law, this injury will not be sufficient (Allen). In this case, it [does/does not] appear that this injury is based off government action because [x]. So, the injury will likely be sufficient.Second, [x]’s injury is [likely/unlikely] to be actual or imminent because [x]. [insert one of the actual/imminent responses].Further, it is [likely/unlikely] that this injury is concrete and particularized because [x]. [Best case: Since [x]’s injury is sufficient, [actual/imminent] and is concrete and particularized, [x] will meet the injury in fact prong. Worst case: Thus, because [x]’s injury is [not sufficient/not actual or imminent/not concrete and particularized] because [x]. Do injury in fact for each party individuallyCausation and redressability (try to aggregate this and not do for each party)'''Second: Under Duke Power, [“the plaintiffs” or x] will be able to show that [their/his/her] injury would not have happened but-for the [act/gov’t action]. Here, [x]’s injury to be fairly traceable to the challenged action of the Government because [x]. It is [likely/unlikely] that causation will be satisfied by all of the parties.Third, Lastly, the injuries here must be redressable. Redressability request that a favorable decision will [most likely/not likely] redress all of the plaintiffs’ harms. This is similar to [Case] because [x]. It’s [likely/unlikely] that [x]’s injury will be sufficiently redressed.Injury In-Fact: P must show a “distinct and palpable” injury, not abstract or hypothetical'

  • (1) Invasion of legally protected interest (sufficient)
    • (Incl: constitutional rights, personal injury, stigmatic injury, statutory, common law, environmental)'
    • Allen'': just b/c gov’t fails to follow law doesn’t mean you suffered an injury'
    • Constitutional right'
      • Here, [x] is suffering a constitutional injury because [x]'. Conduct that infringes on a person’s constitutional right will likely be a sufficient injury.'
    • Economic Injury'
      • Here, [x]' is facing an economic injury because [x]. [x]’s economic harm will be appropriate for a finding of a sufficient injury. '
    • Personal injury'
      • Tort crime (assault, battery) against D will be sufficient for damages'
      • Lyons: This injury is similar to the physical injury faced by the plaintiff in Lyons because [x]'. '
    • Stigmatic injury'
      • (if black school children denied by schools and IRS kept granting tax exemptions to racist schools, stigmatic injury suffered)'
      • Allen: This injury is similar to the stigmatic injury of the plaintiffs in Allen because [x]'. According to the Court, stigmatic injuries will always be sufficient, provided that the party is personally suffering the injury and they are not just raising a generalized grievance (Allen). '
    • Statutory Injury'
      • Lujan: ESA, procedural right, any person able to enforce regardless of any harm; insufficient, too broad'
      • Trafficante: CRA, interracial association, white owner; sufficient, narrow enough to meet standard'
      • Must meet minimum standard that it is not just a generalized grievance or filing suit against the gov’t for not doing their job'
      • (1) What is the statute, (2) the right given by that statute and (3) who holds that right?'
      • “'[x]’s statutory injury is caused by [statute] which gives [him/her] a right to [what right is given]. This will be sufficient for a statutory injury because it is narrow enough to not be applicable to a wide group of people who have not actually experienced harm, and thus resembles the statutory injury in Trafficante.”'
    • Common law injury'
      • Lyons: Same as personal injury w/tort crime to be sufficient'
    • Environmental Injury'
      • Mass v. EPA: EPA’s failure to regulate emissions contributed to global warming; Statutory injury; Clean Air Act created a right to have procedures followed to ensure a clean environment; The violation of this act caused harm to the environment, including MA land; EPA argued that the statutory injury was too broad and failed to meet minimum requirements in order to be sufficient; The Court said that does not matter, so long as there was a real effect on the plaintiff  As long as the plaintiff was harmed in some way
    • Injury that derives from the government just not doing their job or failing to follow the law (pretty much anybody could sue at that point)'
    • There is a minimum standard, injury must be more than a general grievance, otherwise any single person can be injured by that violation and litigate'
    • Sufficient injuries''''''
    • Insufficient''' injury
  • (2) Concrete and Particularized
    • Particularized: Must affect P in a personal and individual way'
    • Concrete:'' Injury can’t be abstract, must be concrete enough to distinguish the interest of the plaintiff from generalized interest every citizen has'
    • Mass'': possible special circumstance b/c the party here is a state which may allow for a greater scope for injury'''
    • Two distinct harms that need to be met;'
    • If a harm is concrete, but it is widely shared, may not be a general grievance and suffices for the injury (Mass. v. EPA)'
Case ODFs Concrete and Particularized?

Lujan

ESA reinterpreted to apply only within US. Ps wanted injunction to return to original interpretation that applied globally.

No, it applied to anyone who worked or wished to view the animals. Ps not affected in any specific way.

MA v. EPA

MA suing EPA for not enforcing Clean Air Act → environmental harms such as losing coast land

Yes, States get special treatment. Loss of land is particularized for a landowner and MA is landowner.

Allen v. Wright

Parents of black children were suing IRS for tax exemptions for private schools → makes desegregation more difficult

No for stigmatic → any black person could sue, not personally experienced. Would have had to show they were denied entry to schools.Yes for other → children in suit were specifically being denied desegregated education

'

  • (3) Actual or Imminent
    • Clapper'': NSA surveillance where the party was concerned that they were going to lose communication with client’s because there would be a fear of having communications intercepted not sufficient, can’t just have an injury because you’re worried that an injury may exist later on
    • Lyons'':For an injunction, P must have shown that it was likely that he would imminently be injured by another inappropriate chokehold from the LAPD not sufficient, had to show that he was likely to suffer the injury again sometime in the future
    • Lujan'':Ps didn’t have concrete plans to exercise their right in the future, just that they wanted to view the animals at some point in the future; not actual or imminent, harm isn’t present or real, could POSSIBLY be sufficient if they had some arrangements that would’ve been injured by the gov’ts conduct
    • 'Actual: Injury that is immediately present; not abstract or hypothetical'
    • Imminent:' Injury that WILL occur in the future, not abstract or hypothetical'
Case ODFs Actual or Imminent Injury?

Lujan

ESA reinterpreted to apply only w/i US. Ps wanted injunction to return to original interpretation that applied globally.

No, individual Ps had no definite plans to return to view wildlife → no factual showing of perceptible harm. Generalized grievance for public interest not enough to be actual

Massachusetts v. EPA

MA suing EPA for not enforcing Clean Air Act → environmental harms such as losing coast land

Yes, loss of land was imminent. MA had loss of land and studies show it will continue

Allen v. Wright

Parents of black children were suing IRS for tax exemptions for private schools → makes desegregation more difficult

Yes for other, segregation happening nowNo for stigmatic, too general of a grievance and too widespread

Lyons

Lyons experienced police brutality

No, can’t show he will experience police brutality again

Clapper v. Amnesty International USA

Law authorizes intel to be collected on those associated w/ people of interest which could include clients of Amnesty

No, harm was hypothetical. No intel had been collected on them

''''Causation: Injury must have a causal connection with the challenged conduct'

  • Challenged action was the “but-for” cause of the injury'
  • The causation may not be so attenuated that the conduct isn’t fairly traceable to the injury (Allen)'
  • States will have special leeway in determining causation, just like w/injury'
Cases ODFs Causation?

Allen v. Wright

Parents of black children were suing IRS for tax exemptions for private schools → makes desegregation more difficult

No, not enough evidence to show but for the tax exemption the schools would be more representative of the population as a whole. Link too tenuous

Duke Power Co. v. Carolina Environmental Study Group Inc.

Act enabled building of nuclear power plants. Ps suing claimed heightened exposure to radiation.

Yes, but for the law, the nuclear reactor wouldn’t have been built and the people wouldn’t have been exposed.

Mass v. EPA

MA suing EPA for not enforcing Clean Air Act → environmental harms such as losing coast land

States are specialYes, scientific studies show link btwn emissions and global warming. US emissions contribute even in some small way → causation

Clapper v. Amnesty International USA

Law authorizes intel to be collected on those associated w/ people of interest which could include clients of Amnesty

No, can’t show intel collected is traceable to law. Requires guesswork as to how the law will be administered. Took precautions (claimed harm) to protect themselves → created own harm.

''''Redressability: A favorable decision must redress the injury of the P'

  • The party can’t contend that with a favorable decision they will have the relief they seek; '
    • Warth: the Ps in this case contended that adjusting zoning laws would allow companies to build affordable housing for low/moderate-income families Favorable decision that would allow companies to build homes doesn’t guarantee the low/moderate-income families would thus afford them to be able to move to town
Cases ODFs Redressability?

Lujan

ESA reinterpreted to apply only w/i US. Ps wanted injunction to return to original interpretation that applied globally.

No, US not sole funder and 3rd parties provided majority of $, so even if policy changed back it wouldn’t control actions of 3rd parties. (Too many other hands in the cookie jar.)

MA v. EPA

MA suing EPA for not enforcing Clean Air Act → environmental harms such as losing coast land

Yes, every little bit helps. Reduction in domestic emissions would slow the pace of global emission increases. Don’t have to stop harm completely just ↓ it.

Linda R.S. v. Richard D.

Deadbeat baby daddy, mom seeking child support. Mom was asking state to prosecute baby daddy for not paying.

No, even if state prosecuted people for not paying child support, she still wouldn’t be guaranteed the child support money. Baby daddy going to jail is only guaranteed result.

Warth v. Seldin

Zoning ordinances and practices discriminated against low-moderate income housing.

No, even if zoning ordinance was repealed, there’s no proof any of them could afford to live there or that housing would be built.

i. Constitutional Standing Requirements
  • Allen v. Wright (1984) [Page 46]
    • o TOPIC: Standing – injury requirement. No injury = no standing.
    • o FACTS: Parents of black school children (parents) (P) sued the Internal Revenue Service (IRS) (D) in a nationwide class action for failing to carry out its statutory mandate to deny tax-exempt status to private schools that discriminated on the basis of race. The parents (P) argued that this failure amounted to federal support for segregated schools, which in turn interfered with federal and local desegregation efforts. The plaintiffs claimed two injuries. (1) One was that they and their children were stigmatized by government financial aid to schools that discriminate. The Court held that this injury was too abstract to confer standing. (2) The plaintiffs also claimed that their children's chances to receive an integrated education were diminished by the continued tax breaks to discriminatory schools. The parents argued that if the IRS enforced the law, the schools either would stop discriminating or have to charge more money because of the loss of the tax breaks. Either way, more white students likely would attend the public schools.
    • o RULE: Standing requires a plaintiff to allege a personal injury fairly traceable to the defendant’s allegedly unlawful conduct and likely to be redressed by the requested relief.
      • To have standing to bring a lawsuit, plaintiffs must sufficiently allege that they have personally suffered a distinct injury, and the chain of causation linking that injury to the actions of a defendant must not be attenuated.
    • o ANALYSIS: Standing doctrine prohibits litigants from raising another person’s rights, bars courts from adjudicating general grievances more appropriately addressed in the representative branches, and requires that a complaint fall within the zone of interests protected by the law invoked. The injury must be “distinct and palpable,” and not “abstract,” “conjectural” or “hypothetical.” The dispute must be one “traditionally thought to be capable of resolution through the judicial process,” and its adjudication must be consistent with the separation of powers. In this case the parents (P) allege two injuries: (1) direct harm from the mere fact of federal aid to discriminatory private schools, and (2) the impairment of their ability to have their schools desegregated.
      • The Court stated that “respondents' second claim of injury cannot support standing because the injury alleged is not fairly traceable to the Government conduct respondents challenge as unlawful.… From the perspective of the IRS, the injury to respondents is highly indirect and results from the independent action of some third party not before the court.”
    • o Justice O’Connor: “Cases such as this, in which the relief requested goes well beyond the violation of law alleged, illustrate why it is important to keep the inquiries separate if the redressability component is to focus on the requested relief.”
  • Massachusetts v. EPA (2007) [Page 53]

PARENS PATRIAE: The state regarded as a sovereign; the state in its capacity as provider of protection to those unable to care for themselves; a doctrine by which a government has standing to prosecute a lawsuit on behalf of a citizen, especially on behalf of someone who is under a legal disability to prosecute the suit. The state ordinarily has no standing to sue on behalf of its citizens, unless a separate, sovereign interest will be served by the suit.

  • o TOPIC: Standing – special consideration for states.
  • o FACTS: After the EPA declined several private petitions to issue regulations governing greenhouse gas emissions from new automobiles, a group of states sued the EPA seeking declaratory relief on the issue of whether the EPA had the statutory authority to regulate greenhouse gas emissions under the Clean Air Act; and if so, whether its stated reasons for refusing to do so were consistent with the Act. Massachusetts alleged inter alia that the EPA’s failure to regulate these emissions would ultimately result in loss of its coastal lands due to increased global warming from the emissions. The EPA claimed that the Act did not authorize the agency to issue regulations to address global climate change and, moreover, that Congress had not yet finished investigating the scientific merits of climate change. The EPA further argued that it was not wise to regulate such emissions at that time.
  • o RULE: When a party has been vested with a procedural right, that party has standing if there is some possibility that the requested relief will prompt the party causing the injury to reconsider the decision that allegedly harmed the other party.
    • (1) The Clean Air Act provides the Environmental Protection Agency with the statutory authority to regulate new motor vehicle emissions greenhouse gases as an “air pollutant.” (2) For standing to be appropriate, an actual case or controversy must be present, which is characterized by a truly adversarial relationship. (3) Although an agency’s refusal to initiate rulemaking is subject to judicial review, such review is extremely limited and highly deferential.
      • A plaintiff can show the existence of a truly adversarial relationship by demonstrating that he has suffered a concrete and particularized injury that is either actual or imminent; that the injury is fairly traceable to the defendant; and that it is likely that a favorable decision would redress that injury. In a class action suit, only one member of a class of petitioners must meet the required elements of standing to sufficiently demonstrate an adversarial relationship.
    • o ANALYSIS: According to the majority, Massachusetts (P) has standing because of the threatened injury to its quasi-sovereign rights. Threatened injury to the citizens of Massachusetts (P) does not figure greatly in the majority’s opinion. This would appear to take standing in this case out of the traditional view of theparens patriae Parts of the majority opinion not reproduced in the casebook mentionparens patriaeonly twice, and those mentions are in quotations from other authority.
      • Chief Justice Roberts's dissent argued that global warming is too general a problem affecting the entire world to meet the requirement for a particularized injury. The dissent, though, focused especially on the redressability requirement and argued that it was not met.

Standing Requirements

  • City of Los Angeles v. Lyons (1983) [Page 59]
    • o TOPIC: Standing – injunctive relief
    • o FACTS: Involved a suit to enjoin as unconstitutional the use of chokeholds by the LAPD in instances where the police were not threatened with death or serious bodily injury. Adolph Lyons, a 24-year-old black man, was stopped by the police for having a burnt-out taillight on his car. A LAPD officer used a chokehold on Lyons to apply excessive force. At the time of the suit, 16 people in Los Angeles had died from the chokehold—12 of them black men. Lyons's complaint alleged that it was the official policy of the LAPD to use the chokeholds in situations where officers were not faced with a threat of bodily injury or death.
    • o ANALYSIS: Although Lyons could bring a suit seeking damages for his injuries, he did not have standing to enjoin the police because he could not demonstrate a substantial likelihood that he, personally, would be choked again in the future. “Lyons' standing to seek the injunction requested depended on whether he was likely to suffer future injury from the use of the chokeholds by police officers.” The Court concluded that “absent a sufficient likelihood that he will again be wronged in a similar way, Lyons is no more entitled to an injunction than any other citizen of Los Angeles; and a federal court may not entertain a claim by any or all citizens who no more than assert that certain practices of law enforcement officers are unconstitutional.”
    • o RULE: Lyons thus establishes that in order for a person to have standing to seek an injunction, the individual must allege a substantial likelihood that he or she will be subjected in the future to the allegedly illegal policy.
  • Lujan v. Defenders of Wildlife (1992) [Page 61]
    • o TOPIC: Standing – congressional statutes can’t grant standing to citizens not actually injured in fact
    • o FACTS: When Congress passed a statute protecting endangered animals, it authorized any person to sue the administrative agency for violating it in a “citizen suit” [provides any person can commence a civil suit on his own behalf] provision. When wildlife activists sued, the agency claimed they lacked standing. First, they alleged “injury in fact” because individual plaintiffs had personally visited potentially impacted foreign sites to view endangered animals and intended to do so again. Second, they proposed an “ecosystem nexus” that confers standing on any person who uses part of a “contiguous ecosystem” even if the damage done is quite distant from that person. Third, they proposed an “animal nexus” that confers standing on any person who wants to view or study endangered animals anywhere on the planet. Finally, fourth, they proposed a “vocational nexus” that confers standing whose profession is linked to endangered species affected by the Secretary’s (D) decisions. On appeal, the Court of Appeals held Defenders (P) had standing because they suffered a “procedural injury,” sinceESA’s “citizen suit” provision grants all “persons” a “procedural right” to the consultation.
    • o RULE: Congressional statutes cannot confer standing to plaintiffs who suffered no “actual” “injury in fact.”
      • Under Article III of the Constitution, a party does not have standing to litigate a generalized grievance against the government in federal court if she suffered no personal injury other than the harm suffered by all citizens.''''
    • o ANALYSIS: The Court expressly applied Lyons and held that the plaintiffs lacked standing because they could not show a sufficient likelihood that they would be injured in the future by a destruction of the endangered species abroad.
      • Two of the plaintiffs had submitted detailed affidavits describing their trips abroad and their viewing of endangered animals such as the Nile crocodile, the elephant, and the leopard. The Court said that the fact that the women had visited the areas in the past “proves nothing,” and their desire to return in the future—”some day”—is insufficient for standing “without any description of concrete plans or indeed any specification of when the some day will be.”
      • Justice Blackmun wrote a vehement dissent and lamented that the requirement that a plaintiff have specific plans to return to a foreign country created only a silly formality that a plaintiff must purchase a plane ticket in order to sue. Moreover, the dissent challenged the majority's assumption that a person is harmed by the destruction of the environment only if the individual has concrete plans to visit the harmed place. Justice Blackmun stated: “It cannot be seriously contended that a litigant's failure to use the precise or exact site where animals are slaughtered or where toxic waste is dumped into a river means that he or she cannot show injury.”
  • Clapper v. Amnesty International USA (2013) [Page 65]
    • o TOPIC: Standing – injury requirement
    • o FACTS: Section 702 of the Foreign Intelligence Surveillance Act of 1978 (FISA), added by the FISA Amendments Act of 2008, permits the attorney general and the Director of National Intelligence to acquire foreign intelligence information by intercepting communications between those in the United States and those in foreign countries. A lawsuit was brought by attorneys, journalists, and business people who said that their communications were chilled by the possibility that their communications with individuals in foreign countries might be intercepted.
    • o RULE: Threatened injury must be certainly impending to constitute injury in fact for purposes of Article III standing.
    • o ANALYSIS: The Court stressed that none of the plaintiffs could show that their communications had been intercepted or were likely to be intercepted. Thus, they lacked the requisite injury required for standing. A key difference between the majority and the dissent was how the test for future likelihood should be articulated. Justice Alito, writing for the Court, said that the injury has to be “certainly impending,” while for the dissent what is required is a “reasonable probability” or “high probability.”

RULE: In the cases Linda R., Warth v. Seldin, and Simon v. Welfare Rights, the Supreme Court determined that standing will likely not exist in cases where there are purely speculative remedies, as in, remedies that are not guaranteed to resolve the problem.RULE: In Duke Power Co., the Supreme Court determined that if a Plaintiff can prove that but for a specific event happening, the Plaintiff will not have been injured or at the risk of being injured, the Plaintiff likely has standing.

  • Linda R. S. v. Richard''''' D. [Page 66]
    • o TOPIC: Standing – unmet (speculative)
    • o FACTS: An unwed mother sued to have the father of her child prosecuted for failure to pay child support. The mother challenged the Texas policy of prosecuting fathers of legitimate children for not paying required child support but not prosecuting fathers of illegitimate children.
    • o ANALYSIS: Dismissed the case for lack of standing, explaining that an injunction commanding state prosecutions would not ensure that the mother would receive any additional child support money.
      • “[I]f appellant were granted the requested relief, it would result only in the jailing of the child’s father. The prospect that prosecution, at least in the future, will result in payment of support can, at best, be termed only speculative.”
  • Warth v. Seldin [Page 66]
    • o TOPIC: Standing – unmet (speculative)
    • o FACTS: Several plaintiffs challenged the constitutionality of zoning practices in Penfield, New York, a suburb of Rochester. The plaintiffs included Rochester residents who wanted to live in Penfield but claimed that they could not because of the zoning practices that prevented construction of multifamily dwellings and low-income housing. Also, an association of homebuilders that wanted to construct such housing joined as plaintiffs in the suit.
    • o ANALYSIS: Held that these plaintiffs lacked standing because they could not demonstrate that appropriate housing would be constructed without the exclusionary zoning ordinances.
      • The Court felt that the low-income residents seeking to live in Penfield might not be able to afford to live there even if the town’s zoning ordinances were invalidated. Also, the builders might not choose to construct new housing in Penfield, regardless of the outcome of the lawsuit.
  • Simon v. Eastern Kentucky Welfare Rights Organization [Page 67]
    • o TOPIC: Standing – unmet (speculative)
    • o FACTS: The Court felt that the low-income residents seeking to live in Penfield might not be able to afford to live there even if the town’s zoning ordinances were invalidated. Also, the builders might not choose to construct new housing in Penfield, regardless of the outcome of the lawsuit.
    • o ANALYSIS: Denied standing, concluding that it was “purely speculative” whether the new Revenue Ruling was responsible for the denial of medical services to the plaintiffs and that “the complaint suggests no substantial likelihood that victory in this suit would result in respondents’ receiving the hospital treatment they desire.”
  • Duke Power Co. v. Carolina Environmental Study Group, Inc. [Page 67]
    • o TOPIC: Standing – met (But for)
    • o FACTS: Forty individuals and two organizations challenged the constitutionality of the Price-Anderson Act, which limits the liability of utility companies in the event of a nuclear reactor accident. The claim was that the Act violated the Due Process Clause because it allowed injuries to occur without compensation. The defendant moved to dismiss the case on the ground that the injury was purely speculative: No catastrophic nuclear accident had occurred, no one had been denied compensation, and perhaps no one ever would suffer the injury.
    • o ANALYSIS: Found standing to exist because the construction of a nuclear reactor in the plaintiffs’ area subjected them to many injuries, including exposure to radiation, thermal pollution, and fear of a major nuclear accident.
      • The Court concluded that the causation and redressability tests were met because but for the Price-Anderson Act the reactor would not be built and the plaintiffs would not suffer these harms. After finding standing, the Court held that the Price-Anderson Act was constitutional.
ii. Prudential Standing Requirements
  • v There are two major prudential standing requirements:
    • o the prohibition of third-party standing and
    • o the prohibition of generalized grievances.
  • v Like the constitutional standing requirements of injury, causation, and redressability, the prudential requirements are judicially created. The difference, though, is that Congress, by statute, can overrule the prudential requirements because they are derived not from the Constitution but instead from the Court’s view of prudent judicial administration.
  • Singleton v. Wulff (1976) [Page 68]
    • o TOPIC: Standing – met (third party)
    • o FACTS: Missouri passed a statute that would provide benefits to a needy woman seeking an abortion only when the abortion was “medically indicated.” Two Missouri doctors (P) who had performed such abortions, sued in federal court to have the law declared unconstitutional. The state Medicaid official (D) moved to dismiss the case. The doctors (P) opposed the motion, stating that they had performed non-medically indicated abortions, that they anticipated performing more such abortions, and that the Medicaid official (D) turned down all Medicaid applications associated with the abortions, citing as authority, the statute at issue in the case. The dismissal motion was appealed, eventually to the U.S. Supreme Court. The primary challenge on appeal did not concern the content of the statute, but rather the right of the physicians to file the suit. The Medicaid official (D) claimed that only the abortion patients had the right to challenge the abortion statute.
    • o RULE: A plaintiff has standing to bring a lawsuit on behalf of a third party’s right when that right is inextricably bound up with the activity the litigant wishes to pursue, and when it is unlikely that the third party can or will sue on his or her own behalf.
    • o ANALYSIS: Normally, the Courts will not allow a person to sue to protect the rights of a third party. The Court has adopted this rule for two reasons. First, the affected third party may choose not to assert her right or may conclude that she can have the benefit of the right without litigating. In such cases, the courts should not litigate unnecessarily. Second, courts want the best possible advocate for a position and, in general, that would be the third party right-holder. When these two reasons do not apply, however, it may be proper for a person to assert the rights of a third party. This is such a case. First, the closeness of the relationship— the relationship between the physicians (P) and the patients is sufficiently close to make the physicians (P) effective proponents of the challenge to the Missouri statute. An indigent woman who might seek an abortion could not safely exercise her right to an abortion without access to her physician and Medicaid assistance to pay for that access. Second, the likelihood that the third party can bring the suit on his or her own behalf— genuine obstacles exist in this case to the woman bringing the case herself. For example, the woman might be scared off from the lawsuit because it would compromise her privacy. Furthermore, rights concerning pregnancy and abortion fit into the category of “capable of repetition yet evading review” since the case becomes “moot” once the pregnancy has advanced beyond the point where abortion is safe or practical.
      • The Court emphasized the closeness of the doctors' relationship to the patient and that “the constitutionally protected abortion decision is one in which the physician is intimately involved.” The Court concluded that “it generally is appropriate to allow a physician to assert the rights of women patients as against governmental interference with the abortion decision.”
  • Barrows v. Jackson [Page 70]
    • o TOPIC: Standing – met (third party)
    • o FACTS: Barrows, a white person who had signed a racially restrictive covenant, was sued for breach of contract for allowing nonwhites to occupy the property. As a defense, Barrows raised the rights of blacks, who were not parties to the lawsuit, to be free from discrimination.
    • o ANALYSIS: The Court allowed third-party standing, permitting the white defendant to raise the interests of blacks to rent and own property in the community. The Court stated that “it would be difficult if not impossible for the persons whose rights are asserted to present their grievance before any court.” Because blacks were not parties to the covenant, they had no legal basis for participating in the breach of contract suit. (The suit occurred before open housing laws were enacted that would have allowed African Americans to challenge the covenants as impermissible discrimination.)
  • Craig v. Bore [Page 71]
    • o TOPIC: Standing – met (third party)
    • o FACTS: Oklahoma law permitted women to buy 3.2 percent beer at age 18, but said that men could not do so until age 21. A bartender challenged the law on behalf of male customers between the ages of 18 and 21. The bartender suffered economic loss from the law, thus fulfilling the injury requirement.
    • o ANALYSIS: The Court allowed the bartender standing to assert the rights of his customers and explained “that vendors and those in like positions have been uniformly permitted to resist efforts at restricting their operations by acting as advocates for the rights of third parties who seek access to their market or function.”
  • Gilmore v. Utah [Page 71]
    • o TOPIC: Standing – unmet (third party)
    • o FACTS: Gilmore was sentenced to death in Utah, but chose not to pursue collateral challenges in federal court. His mother sought a stay of execution on his behalf.
    • o ANALYSIS: The Gilmore case might be read as supporting the proposition that a close relationship is not enough for third-party standing; the advocate also must be part of the third party's exercise of the protected right. On the other hand, Gilmore might be thought of as a narrow decision in a unique factual context.
  • United States v. Richardson (1974) [Page 72]
    • o TOPIC: Standing – unmet
    • o FACTS: A taxpayer sued to compel the CIA to release details of its expenditures pursuant to Article I, § 9 of the U.S. Constitution.
    • o RULE: Taxpayer status is not sufficient to confer standing to challenge the constitutionality of federal action unless the taxpayer alleges direct injury from the practice and not generalized grievances common to all members of the public.
    • o ANALYSIS: No. Because his allegations do not fit the two-part test for taxpayer standing articulated inFlast
      • Richardson is an interesting case to consider when reviewing the prohibition against generalized grievances. The majority opinion begins by simply citing to two precedents: the broad rule against taxpayer standing from Frothingham, and the narrow exception from Flast. Here the reasoning is straightforward; the opinion applies the facts of Richardson’s claim to the Flast test and concludes that this case does not fit the exception. Then, however, the majority elaborates on the fact that Richardson’s claim is a generalized grievance. The opinion states that when the generalized nature of the grievance makes it impossible to grant any plaintiff standing to bring the case, it is because the subject matter of the case was “committed” to a legislative or political resolution.

Schlesinger v. Reservists Committee to Stop the War

  • v The Court also denied citizen and taxpayer standing. The plaintiffs sued to enjoin members of Congress from serving in the military reserves. Article 1, §6 of the Constitution prevents a senator or representative from holding civil office.
  • v As in Richardson, standing was denied because the plaintiff alleged injury only as a citizen or taxpayer with an interest in having the government follow the law and not a violation of a specific constitutional right.
    • o “Respondents seek to have the Judicial Branch compel the Executive Branch to act in conformity with the Incompatibility Clause, an interest shared by all citizens. . . . Our system of government leaves many crucial decisions to the political processes. The assumption that if respondents have no standing to sue, no one would have standing, is not a reason to find standing.”
  • Flast v. Cohen (1968) [Page 75]
    • o TOPIC: Standing – met (generalized grievance)
    • o FACTS: Several individual taxpayers [the “Taxpayers”] (P) filed suit in federal District Court claiming that federal expenditures under the Elementary and Secondary Education Act of 1965 violated the First Amendment’s Establishment and Free Exercise Clauses. The challenged expenditures were the purchase of books and materials and the funding of instruction in academic subjects for religious schools. The Taxpayers (P) claimed standing to bring their claim exclusively on their taxpayer status. Several individual taxpayers [the “Taxpayers”] (P) filed suit in federal District Court claiming that federal expenditures under the Elementary and Secondary Education Act of 1965 violated the First Amendment’s Establishment and Free Exercise Clauses. The challenged expenditures were the purchase of books and materials and the funding of instruction in academic subjects for religious schools. The Taxpayers (P) claimed standing to bring their claim exclusively on their taxpayer status. The lower court dismissed the plaintiff's challenge to the Act based on Frothingham (holding that a federal taxpayer is without standing to challenge the constitutionality of a federal statute), concluding that the plaintiff's only claim was as a taxpayer and that such standing was not permitted.
    • o RULE: Taxpayer status is sufficient to confer standing on an individual to bring suit in federal court to challenge the constitutionality of federal spending in violation of the First Amendment Free Exercise and Establishment clauses.
    • o ANALYSIS: The purpose of the standing requirement is to assure that the litigant has a sufficient personal stake to bring the case in a traditionally adversarial posture so that the courts may decide the controversy. Whether the nexus between a person’s taxpayer status and his interest in a controversy will be sufficiently close to grant him standing can be determined through a two-part test. First, the taxpayer must be challenging an action taken by Congress under its Article I, § 8 taxing and spending power. In this case, the Taxpayers (P) challenged an exercise by Congress of its power under Article I, § 8 to spend for the general welfare, and the challenged program involves a significant expenditure of federal funds. Second, the taxpayer must allege that the challenged levies or expenditures violate specific constitutional limits on government power. Here, the Taxpayers (P) allege violations of the First Amendment Establishment and Free Exercise clauses. Here, the taxpayers sued alleging that the expenditures violate the Free Exercise and Establishment clauses of the Constitution; both of which were designed to limit Congress’ ability to tax and spend for the purposes of promoting or furthering religion.
      • The Framers saw the connection between the power to tax and spend and religious liberty. They intended to prevent the government from using its power to favor a given religion or religious institution. The Framers intended the Establishment Clause as a limit on Congress’s power to tax and spend.
    • o The Court distinguished Flast from Frothingham because although both involved challenges to government spending programs, the First Amendment is a limit on Congress's taxing and spending authority, whereas the Tenth Amendment, at issue in Frothingham, is not. Flast raised speculation that the Court had substantially expanded the availability of taxpayer standing.

Valley Forge Christian College v. Americans United for Separation of Church and State, Inc.

  • v Held that taxpayers lacked standing to challenge the federal government’s decision to grant property or its power to dispose of property under Article IV, §3. The Court held that taxpayers have standing only to challenge expenditures under Congress’s Article I, §8 spending power as violating the Establishment Clause.

Hein v. Freedom from Religion Foundation

  • v FACTS: President George W. Bush created a White House office and several centers in executive branch offices to provide government funds to “faith-based institutions.” This was designed to facilitate churches, synagogues, and mosques directly receiving federal money to provide social services. The White House Office of Faith-Based and Community Initiatives was funded entirely through general executive revenue. Taxpayers brought a suit challenging this as violating the establishment clause.
  • v HOLDING: The Court ruled, without a majority opinion, that the taxpayers lacked standing.
    • o The Court distinguished Flast v. Cohen as involving expenditures under a specific federal statute, whereas the money for the White House Office of Faith-Based and Community Initiatives came from general executive revenue.
  • v In other words, the plurality concludes that taxpayers lack standing to challenge expenditures of funds from general executive revenue as violating the establishment clause.
  • v The other six Justices criticized the plurality's distinction of Flast. The executive branch, no less than Congress, must comply with the First Amendment. All federal spending is pursuant to a federal statute; all executive branch revenues are authorized by Congress.
  • v Justices Scalia and Thomas concurred in the judgment and called for the overruling of Flast. Justice Scalia lamented the “meaningless distinctions” and wrote: “Flast is wholly irreconcilable with the Article III restrictions on federal-court jurisdiction that this Court has repeatedly confirmed are embodied in the doctrine of standing.”
  • v By contrast, Justice Souter, in a dissent joined by Justices Stevens, Ginsburg, and Breyer, agreed that there was no meaningful distinction between money from general executive revenue and spending under a specific federal statute, but they would have followed Flast and allowed taxpayer standing.

c). Ripeness - bars consideration of claims before they have fully developedRIPENESS: The point in a dispute where the facts and adversity of the parties allow a court to render a meaningful resolution to the conflict; many courts decline to hear issues that are not “ripe” for their resolution.Rule: Generally, a court may not review or grant a declaratory judgment of a state law before it is enforced or when there is ''no real threat the statute will ever be enforced (Poe v. Ullman).

  • v Ripeness, like mootness, is a justiciability doctrine determining when review is appropriate. The ripeness doctrine seeks to separate matters that are premature for review because the injury is speculative and never may occur, from those cases that are appropriate for federal court action.

Ripeness'''''''Issue: The issue is whether [x]’s complaint is ripe for review.Rule: A complaint is ripe if is too early and there isn’t a concrete dispute (Mitchell). A controversy that is merely speculated or is requesting anticipatory relief will be too ripe (Laird). It is [likely/unlikely] that this case is ripe.Application': Here, [x] is facing a concrete disputebecause [x]. However, this may not be considered sufficient for ripeness because [x].

  • Cases must be ready for litigation, court can’t consider claims that haven’t yet developed as it would be too premature.

'Conclusion': It is [likely/unlikely] that [x]’s complaint will be ripe.

  • Poe v. Ullman (1961) [Page 81]
    • o TOPIC: Ripeness
    • o FACTS: Married women for whom pregnancy was medically unadvisable and their doctors filed a lawsuit challenging a Connecticut law preventing the distribution or use of contraceptives.
    • o RULE: For a lawsuit to be ripe for adjudication, the injury threatened must be relatively immediate and certain to occur without court intervention.
      • Absent state prosecution, the existence of a statute does not make the statute’s constitutionality ripe for federal court review.
    • o ANALYSIS: The Court noted that because the state has not chosen to enforce its anti-conception statute (and appears unlikely to do so), the issue of that statute’s constitutionality is not ripe for determination 34 by the federal courts. Even if the parties were to stipulate facts concerning possible prosecutions, it would not make the case ripe for federal court resolution because the reality of the situation is clearly different. For example, contraceptives are sold openly in Connecticut stores, yet no prosecutions have resulted. It is unlikely, therefore, that a prosecution would result from the giving of private medical advice.
      • Justice Douglas, in dissent, argued that there was sufficient hardship to justify judicial review of the Connecticut statute: “What are these people—doctors and patients—to do? Flout the law and go to prison? Violate the law surreptitiously and hope they will not get caught? By today's decision we leave them no other alternatives. It is not the choice that they need have under the regime of the declaratory judgment and our constitutional system.”
      • The severity of the facts in this case serves to underscore how seriously the Court takes the issue of ripeness. The majority concludes that review of the Connecticut statute would be premature since there has been no enforcement. Should a court lower its ripeness threshold in cases, like this one, where the threat of harm from the law is significant? The dissent inPoeimplies that the court could consider such facts. The majority opinion, on the other hand, seems to look exclusively to the likelihood of prosecution to determine whether it will decide the case. But neither the dissent nor the majority opinion outlines a clear test for determining when a controversy is ripe.
  • Abbott Laboratories v. Gardner (1967) [Page 84]
    • o TOPIC: Ripeness
    • o FACTS: The FDA promulgated a regulation requiring the inclusion of generic names for prescription drugs on all labels and other printed materials. Violations of the regulation were punishable by civil and criminal sanctions. Thirty-seven drug companies, accounting for 90 percent of the supply of prescription drugs in the country, challenged the regulation as exceeding the scope of the FDA's authority under the pertinent statutes. The government argued that the case was not ripe until a drug company was prosecuted for violating the regulation.
    • o RULE: (1) In determining whether a case or controversy is ripe for adjudication, a court must evaluate the fitness of the issues for judicial decision and the hardship to the parties of withholding court consideration. (2)Judicial review of a final agency action by an aggrieved person will not be cut off unless Congress clearly intended to prevent such review.
    • o ANALYSIS: The Court emphasized the substantial hardship to denying preenforcement review.
      • “If petitioners wish to comply they must change all their labels, advertisements and promotional materials; they must destroy stocks of printed matter; and they must invest heavily in new printing type and new supplies. The alternative to compliance … would risk serious criminal and civil penalties for the unlawful distribution of ‘misbranded’ drugs.”
    • o This case is an interesting companion to Poe. In Abbott, the Court sets out a two-part test to determine ripeness. If the issue in the case is appropriate for judicial resolution, the Court will 36 look at whether the parties will face a hardship if the courts cannot decide the case. After examining the rule in Abbott, it is tempting to go back to Poe and see where that case would have failed the Abbott The main difference between the cases seems to be that in Poe the law in question had been on the books a long time without much enforcement.
  • United Public Workers v. Mitchell [Page 85]
    • o TOPIC: Ripeness
    • o FACTS: This case is an interesting companion to Poe. In Abbott, the Court sets out a two-part test to determine ripeness. If the issue in the case is appropriate for judicial resolution, the Court will 36 look at whether the parties will face a hardship if the courts cannot decide the case. After examining the rule in Abbott, it is tempting to go back to Poe and see where that case would have failed the Abbott test. The main difference between the cases seems to be that in Poe the law in question had been on the books a long time without much enforcement.
    • o RULE: Pursuant to Article III of the Constitution, federal courts may not render advisory opinions on constitutional issues in the absence of an actual interference with the parties’ interests.
    • o ANALYSIS: The Court found their claims to be not ripe. The Court said that the plaintiffs “seem clearly to seek advisory opinions upon broad claims. . . . A hypothetical threat is not enough. We can only speculate as to the kinds of political activity the appellants desire to engage in or as to the contents of their proposed public statements or the circumstances of their publication.”

RULE: Where the inevitably of the enforcement of a statute is impending, a justiciable controversy is irrelevant and the case can be considered ripe.

  • Regional Rail Reorganization Act Cases
    • o TOPIC: Ripeness – inevitability
    • o FACTS: Eight major railroads brought a lawsuit challenging the conveyance of their property to Conrail. The district court found the case not justiciable on ripeness grounds because the reorganization plan had not yet been formulated and a special court had not yet ordered the reconveyances.
    • o ANALYSIS: But the Supreme Court held that the case was ripe, concluding, “Where the inevitability of the operation of a statute against certain individuals is patent, it is irrelevant to the existence of a justiciable controversy that there will be a time delay before the disputed provisions will come into effect.”
  • Lake Carriers Assn. v. MacMullan [Page 85]
    • o TOPIC: Ripeness – inevitability
    • o FACTS: A state law prohibited discharge of sewage from boats. Plaintiffs challenged the statute’s validity. State officials had announced that they would not enforce the law until land-based pumpout facilities would be available, a construction process that would take a substantial amount of time.
    • o ANALYSIS: Although enforcement was many years in the future, the Court found that the suit was ripe because it was inevitable that the law would be enforced and that as a result the boat owners had to begin installing new facilities on their boats in anticipation of the time when the law was implemented.
  • Susan B. Anthony List v. Driehaus [Page 86]
    • o TOPIC: Ripeness
    • o FACTS: The Court considered whether a group had standing to challenge an Ohio statute that prohibits “false statements” during the course of a political campaign. The Court explained that the “question in this case is whether their preenforcement challenge to that law is justiciable—and in particular, whether they have alleged a sufficiently imminent injury for the purposes of Article III.” A member of Congress initiated proceedings against the Susan B. Anthony List in the Ohio Elections Commission, which found probable cause to proceed, but the matter was dismissed after the Congressman lost his reelection bid. The Susan B. Anthony List had brought a challenge to the Ohio law in federal court, and the issue before the Supreme Court was whether its challenge to the Ohio law could continue after the Ohio Elections Commission proceedings were dismissed.
    • o ANALYSIS: The Supreme Court ruled unanimously in favor of the Susan B. Anthony List and found that the matter was justiciable. Justice Thomas, writing for a unanimous Court, noted that “[w]hen an individual is subject to such a threat, an actual arrest, prosecution, or other enforcement action is not a prerequisite to challenging the law.” The Court found that the matter was ripe for review because the plaintiffs “have alleged a credible threat of enforcement.” The Court stressed that “the threat of future enforcement of the false statement statute is substantial.”

d). Mootness

  • v MOOTNESS DOCTRINE: Prohibits courts from deciding issues that are only abstract and do not involve a real dispute, or which have already been resolved.
  • v A plaintiff must present a live controversy at all stages of federal court litigation. If anything occurs while a lawsuit is pending to end the plaintiff’s injury, the case is to be dismissed as moot.
  • v For example, a case is moot if a criminal defendant dies during the appeals process or if a civil plaintiff dies where the cause of action does not survive death. Also, if the parties settle the matter, a live controversy obviously no longer exists. If a challenged law is repealed or expires, the case is moot.
  • v One exception to the mootness doctrine is for “wrongs capable of repetition but evading review.” Some injuries are of such short duration that inevitably they are over before the federal court proceedings are completed. A case is not dismissed, even though it is moot, if there is an injury likely to recur in the future and it is possible that it could happen to the plaintiff again, and it is of such a short duration that it likely always will evade review. (Carolene Products, Footnote #4)

Mootness'Issue: The issue is whether [x]’s complaint is moot.Rule: An issue is moot if it isn’t live and actual at each stage of review. The mootness doctrine requires than an actual, ongoing controversy exist at all stages of the controversy, not just at the time of filing (DeFunnis). It is [likely/unlikely] that [x]’s complaint is moot.Application': In this case, [x]’s [will/will not] be deprived of a concrete stake in the outcome because there [have/have not] been changes in the facts or in the law occurring after lawsuit begins. Here, it would not be impossible to grant [x] was effectual relief, because [x]. This is [analogous/distinguishable] to the plaintiff in DeFunnis because [x].

    • Exceptions:''''''
      • Cases that are capable of repetition yet evading review (Roe v. Wade) (Moore v. Ogilvie)'
      • Cases where D voluntarily ceases' illegal or wrongful action when litigation commences, however, court must be assured that “there is no reasonable expectation that the wrong will be repeated” (Laidlaw)'

RULE: In sum, a case is not dismissed, although the plaintiff's claim is moot, if the injury is one likely to recur and if the injury is of an inherently short duration that would make complete federal court review impossible. Courts have substantial discretion in deciding what is a sufficient likelihood of future injury or a sufficiently short time span for the injury to justify invoking this exception.

  • Moore v. Ogilvie [Page 87]
    • o TOPIC: Mootness – capable of repetition, yet evading review
    • o FACTS: Illinois law required that a petition to nominate candidates for the general election for a new political party be signed by at least 25,000 qualified voters, including 200 qualified voters from each of at least 50 counties. In 1968, the plaintiffs filed petitions for inclusion on the ballot but were denied this because they did not meet the requirement for the number of signatures in each county (which was unconstitutional based on the idea of one man one vote, some counties are more concentrated than others). They immediately filed suit, but, of course, the election was over by the time the Supreme Court heard the case.
    • o ANALYSIS: Illinois law required that a petition to nominate candidates for the general election for a new political party be signed by at least 25,000 qualified voters, including 200 qualified voters from each of at least 50 counties. In 1968, the plaintiffs filed petitions for inclusion on the ballot but were denied this because they did not meet the requirement for the number of signatures in each county. They immediately filed suit, but, of course, the election was over by the time the Supreme Court heard the case.
  • Roe v. Wade [Page 87]
    • o TOPIC: Mootness – capable of repetition, yet evading review
    • o FACTS: The plaintiff was pregnant when she filed her complaint challenging the constitutionality of a state law prohibiting abortion. However, obviously, by the time the case reached the Supreme Court, her pregnancy was completed and she no longer sought an abortion. Hence, her case was moot; intervening circumstances meant that there no longer was a live controversy between the plaintiff and the state.
    • o ANALYSIS: But the Supreme Court refused a request to dismiss the case on mootness grounds. The Court explained that the duration of pregnancy was inherently likely to be shorter than the time required for federal court litigation. The Court concluded that the challenge to the state laws prohibiting abortions “truly could be ‘capable of repetition yet evading review.’”
  • DeFunis v. Odegaard [Page 88]
    • o FACTS: The plaintiff, a white male, applied for admission to the University of Washington Law School and was denied acceptance. He sued the school, contending that he was discriminated against because of the school's preferential treatment of minority candidates. The trial court issued a preliminary injunction admitting the plaintiff to law school while the case was pending. By the time the case reached SCOTUS, the plaintiff was in his final year of school and the school stipulated that the plaintiff would be allowed to complete his studies regardless of the outcome of the litigation.
    • o ANALYSIS: The Supreme Court held that the case was moot because “the controversy between the parties has thus clearly ceased to be definite and concrete and no longer touches the legal relations of parties having adverse legal interests.”
  • Friends of the Earth, Inc. v. Laidlaw Environmental Services (2000) [Page 89]
    • o TOPIC: Mootness
    • o FACTS: The environmental group Friends of the Earth (P) brought a citizen suit under the Clean Water Act alleging that Laidlaw Environmental Services [“Laidlaw”] (D) had violated the mercury discharge limits established by its National Pollutant Discharge Elimination System [“NPDES”] permit. During the course of the lawsuit (which went on for several years), Laidlaw (D) voluntarily achieved compliance with its NPDES permit and also closed its Roebuck facility. However, Laidlaw (D) retained its NPDES permit.
    • o RULE: Defendant’s voluntary cessation of actions that are the subject of a Complaint does not make the lawsuit moot unless there is no reasonable chance that the defendant can return to the actions.
    • o ANALYSIS: This case is an example of the “voluntary cessation” exception to the mootness doctrine. It applies when a defendant stops doing the activity that produced the complaint in the case. The exception states that even though the challenged activity has ceased to exist, the court can still hear the case unless it is clear that the defendant cannot resume the activity. The standard of proof is very high for a defendant who wants to argue that his voluntary action has mooted the case. The defendant must show that there is “no reasonable chance” that he could start up the challenged behavior or activity again. Thus, shutting off a valve would not be sufficient, but creating a permanent barrier to the pipe’s ability to flow probably would. This exception is especially important to the courts because it helps avoid the type of on-again/off-again dispute that ties up court resources without bringing finality to the case. In this sense, the voluntary cessation exception is similar to “wrongs capable of repetition but evading review.” Although that category generally covers situations that have a short time-span, the practical effect is the same as with the defendant who stops long enough to make the lawsuit go away but then repeats the same behavior.
      • This is quite different from standing where the burden is on the plaintiff to show that the constitutional and prudential requirements are met. The Court, in Laidlaw, found that the defendant failed to meet its heavy burden and refused to dismiss the case based on its voluntary changes in behavior.
  • United States Parole Commission v. Geraghty (1980) [Page 90]
    • o TOPIC: Mootness
    • o FACTS: Geraghty (P), a federal prisoner, filed suit in federal court to challenge the federal parole release guidelines. Geraghty (P) sought to be the named plaintiff to represent the class of all federal prisoners eligible for parole now or in the future. The District Court denied Geraghty’s (P) request for class certification. The District Court granted summary judgment in favor of the federal parole board (D). Geraghty (P) appealed both decisions, but was released from prison while the appeal was pending. Other prisoners that would have been in the class moved to have themselves substituted as the named plaintiffs in the case.
    • o RULE: A properly certified class action suit may continue even if the named plaintiff’s individual claims are rendered moot.
    • o ANALYSIS: United States Parole Commission v. Geraghtyillustrates a third exception to the mootness doctrine in federal courts, namely, an exception for many class action lawsuits. The general rule is that when class members continue to have “live” controversies, the fact that the named plaintiff’s case becomes moot does not make the entire class action moot. The specific rule inGeraghty—that an appeal of a denial of class certification is not mooted because the named plaintiff’s case was mooted—is a logical extension of this general rule.

e). The Political Question Doctrine i. Political Question Doctrine Defined

  • v The political question doctrine refers to allegations of constitutional violations that federal courts will not adjudicate, and that the Supreme Court deems to be inappropriate for judicial review.
    • o The Court has held that some constitutional provisions are left to the political branches of government to interpret and enforce. Although there is an allegation that the Constitution has been violated, cases brought under these provisions are dismissed as nonjusticiable political questions.

Issue': The issue is whether [x]’s claim presents a non-justiciable political question.Rule': The political question doctrine excludes controversies from review that derive from policy choices and determinations that are committed for resolution to Congress or the Executive (Baker). A complaint will be a political question if it meets any one of the six elements enumerated in Baker. These factors are (1) a textually demonstrable constitutional commitment of the issue to coordinate political department; or (2) a lack of judicially discoverable and manageable standards; or (3) the impossibility of deciding without an initial policy determination of a kind clearly for nonjudicial discretion; or (4) the impossibility of a court’s undertaking independent resolution without expressing lack of the respect due coordinate branches of government; or (5) an unusual need for unquestioning adherence to a political decision already made; or (6) the potentiality of embarrassment from multifarious pronouncements by various departments on one question (Baker). A tangential relationship between the merits of a case and one of these six factors is insufficient for the Court to find a political question, although certain cases have demonstrated that first two factors to be the most important (Powell).Application

  • Federal courts won’t rule on controversy if the matter is a political question that is to be resolved by the other branches of gov’t'
    • '1) A textually demonstrable constitutional commitment of the issue to a coordinate political dept. '
      • Nixon: Impeachment trying left to Senate, as long as they don’t exceed scope of their authority this is political question'
      • Powell: Congress has constitutional ability to exclude members based on age, citizenship and residence w/o judicial review, nothing more'
    • '2) Lack of judicially discoverable and manageable standards for resolving issue'
      • Guarantee of republican gov’t'
      • War power disputes'
      • President and Legislature have alternative methods of dispute resolution w/treaties'
    • '3) Impossibility of deciding w/initial policy determination of a kind clearly for non-judicial discretion (unsuitable policy determination)'
    • '4) Impossibility of court’s undertaking indep. resolution w/o expressing lack of respect due co-ordinate branches of gov’t (Lack of respect for other branches)'
    • '5) Unusual need for unquestioning adherence to political decision already made (there has already been a political decision)'
    • '6) Potential for “embarrassment from pronouncements made by other departments on same question'
    • Baker’s 6 Political Question Test'
  • Typical issues for Political Question Doctrine to be present''''''
    • Goldwater'': no standards in Constitution governing rescission of treaties and matter was “dispute b/w coequal branches of gov’t each of which has resources available to protect and assert their interests” ' case was a political Q'''
    • Zivotofsky'': case dealt w/ examining a statute and of passport and recognition powers ' typical for court to review; not a political Q'''
    • As long as Senate functioning w/in their scope of power ' case will be political Q'''
    • Actions totally committed to executive branch'''
    • Some, but not all, foreign policy issues'''
    • Questions arising under Guaranty Clause (Luther; all cases under Guaranty Clause = PQ)'''
    • Most Questions of Impeachment (Nixon)'''
    • Qs if Constitutional amendment has been ratified (Colemen)'''
Case Name ODF Political Question?

Baker v. Carr

Plaintiffs sued for loss of meaningful vote in Tennessee

No, this dealt with a state issue, not a coordinate government branch, embarrassing policy, and there was a lot of precedent

Nixon (Senator) v. United States

Senator Nixon argued impeachment by Senate committee was violation of Art. I standard of “trial by senate”

Yes, Senate has sole power to try impeachments and decide policies

Powell v. McCormack

House refused to seat representative based on corrupt findings

No, House can’t refuse to seat a member

Goldwater v. Carter

Carter pulled a treaty with China to enter a treaty with Taiwan

Yes, this was a dispute between coequal branches and the Court did not want to be involved with partisan climate

Before Baker v. Carr,

  • v In Colegrove v. Green, in 1946, the Court held that challenges to malapportionment under the “Guaranty Clause” are a nonjusticiable political question.
    • o This clause is found in Article IV, §4 of the Constitution: “The United States shall guarantee to every State in this Union a Republican Form of Government, and shall protect each of them against Invasion; and on Application of the Legislature, or of the Executive (when the Legislature cannot be convened) against domestic Violence.”
  • v In Luther v. Borden, in 1849, the Court refused to hear a challenge to the Rhode Island government under the Guaranty Clause after the voters of Rhode Island adopted a new state constitution, in part in response to significant malapportionment in the state legislature.
    • o After this enactment, the existing government, which was sure to lose power under the new document, enacted a law prohibiting the constitution from going into effect. Nonetheless, elections were held—even though the existing government had declared voting in them to be a crime. Relatively few people participated, but a new government was chosen, led by Dorr, who was elected governor. Dorr’s government met for two days in an abandoned foundry and then disbanded.
    • o In April 1842, a sheriff, L. Borden, broke into the house of one of the election commissioners, Martin Luther, to search for evidence of illegal participation in the prohibited election. Luther sued Borden for trespass. Borden claimed that the search was a lawful exercise of government power. Luther, however, contended that Borden acted pursuant to an unconstitutional government’s orders; he maintained that the Rhode Island government violated the Republican Form of Government Clause.
    • o The Supreme Court held that the case posed a political question that could not be decided by a federal court.
      • “Under this article of the Constitution it rests with Congress to decide what government is the established one in a State. For as the United States guarantee to each State a republican government, Congress must necessarily decide what government is established in the State before it can determine whether it is republican or not.” The Court also explained that the case posed a political question because if the state’s government was declared unconstitutional, then all of its actions would be invalidated, creating chaos in Rhode Island.

RULE: The Supreme Court never has varied from this holding: Cases under the Guaranty Clause are nonjusticiable.

Baker v. Carr (1962)[edit | edit source]

[Page 92]
    • o TOPIC: Political Question
    • o FACTS: Voters in Tennessee claimed that the apportionment of the Tennessee General Assembly violated their equal protection rights “by virtue of the debasement of their votes.” The state constitution allocated representation based on population, but the assembly had not been reapportioned since 1901. Because of the malapportioned Assembly, the voters’ group (P) contends that redress through changes in state law was difficult or impossible. They sought to enjoin any further elections under the 1901 apportionment, and asked a federal court either to direct elections at large or to decree an apportionment according to the most recent census figures. The lower court denied relief.
    • o RULE: Claims that state apportionment violates the Guarantee Clause are nonjusticiable ‘political questions,’ but claims based on violation of equal protection are justiciable.
    • o ANALYSIS: This challenge to apportionment presents no non-justiciable “political question.” The fact that the suit seeks protection of a political right does not mean that it is necessarily a political question. It has been argued that apportionment cases involve no federal constitutional right except one resting on Article IV, § 4, which guarantees a republican form of government. Based on a review of precedent, we hold that the case here neither rests upon nor implicates the Guaranty Clause. In Guaranty Clause and “political question” cases, it is the relationship between the judiciary and the coordinate branches of the Federal Government, and not the federal judiciary’s relationship to the States, which gives rise to the “political question.”
      • Justice Brnnan: In any case involving a political question is found (1) a constitutionally assigned duty or power to a branch of government; or (2) a lack of judicially manageable standards for resolving the question; or (3) the impossibility of a court’s deciding the issue without an initial policy determination of a kind clearly for non-judicial discretion; or (4) the impossibility of a court’s undertaking independent resolution without expressing a lack of respect to other branches of government; or (5) an unusual need for adherence to a political decision already made; or (6) the potential for embarrassment from various pronouncements on a single issue by different departments of government. If none of these circumstances is present, the case should not be dismissed on the ground that it is a political question.
  • Davis. Bandemer [Page somewhere from 92-96]
    • o TOPIC: Political Question – Gerrymandering
    • o FACTS: Plaintiffs contended that the Republican-controlled Indiana legislature gerrymandered the drawing of election districts to maximize the election of Republican representatives. While careful to preserve one person, one vote and to avoid racial discrimination, the state legislature tried to divide the Democrats into separate districts where possible and to combine Republican voters into districts where they would be the majority. The result was that Democrats obtained a majority of the popular vote in the state in legislative elections but only won a minority of the seats in the legislature. The plaintiff claimed that this was a violation of equal protection.
    • o RULE: The Court held that “political gerrymandering cases are properly justiciable under the Equal Protection Clause''.”
    • o ANALYSIS: The Supreme Court held that the claim was justiciable. The Court explained that “the standards that we set forth here for adjudicating this political gerrymandering claim are [no] less manageable than the standards that have been developed for racial gerrymandering claims.”
      • [A]s here, the mere lack of proportional representation will not be sufficient to prove unconstitutional discrimination. Without specific supporting evidence, a court cannot presume in such a case that those who are elected will disregard the disproportionally underrepresented group. Rather, unconstitutional discrimination occurs only when the electoral system is arranged in a manner that will consistently degrade a voter's or a group of voters' influence on the political process as a whole. Justice White, for the plurality.
  • Vieth v. Jubelirer (YEAR) [Page 96]
    • o TOPIC: Gerrymandering IS a political question!
    • o FACTS: Vieth (P) challenged state electoral districts established by the Pennsylvania General Assembly as unconstitutional political gerrymandering. Following the 2000 census, Pennsylvania was entitled to nineteen Representatives in Congress, down two members from the previous election. The Republican-controlled General Assembly drew a new districting map under pressure from national Republicans to create Republican-favored districts, as retaliation for Democrat-favored districts created in other states. After the plan was adopted, Vieth (P) and other Pennsylvania Democrats challenged the plan as unconstitutional political gerrymandering.
    • o RULE: Political gerrymandering claims are not justiciable.
    • o ANALYSIS: InDavis v. Bandemer, 478 U.S. 109 (1986), the Supreme Court held that political gerrymandering claims were justiciable but defined no standards under which such claims should be adjudicated. However, because political gerrymandering claims involve nonjusticiable political questions,Bandemeris overruled. Although there are several tests for determining whether a political question exists, the Court has stated that “a lack of judicially discoverable and manageable standards for resolving” an issue calls for judicial abstention.
      • The stark differences among the justices illustrate the difficult questions often presented in cases deciding the justiciability of a claim. While the various dissenting justices appear willing to keep up the fight to develop a justiciable standard, the Court’s plurality decision appears to foreclose that possibility. Just as Justice Souter pointed out, no manageable standard arose followingBandemerbecause the Court’s decision prevented lower courts from exploring different standards. The Court’s decision inViethseems to forever foreclose the possibility that such a standard will emerge.

Gill v. Whitford (pending, US Supreme Court).

  • v The trial provided a key initial test for theefficiency gap, a proposed standard for determining discriminatory effect that counts the number of votes each party wastes in an election to determine whether either party enjoyed a systematic advantage in turning votes into seats.

League of Women Voters v. Rucho (3-judge panel, North Carolina)

  • v Partisan gerrymander favoring Republicans in congressional districts struck down as violative of associational rights and equal protection.

League of Women Voters v. Pennsylvania (PA Supreme Court)

  • v Court struck down Pennsylvania congressional districts on state constitutional grounds, after federal panel upheld districts in face of federal constitutional challenge.
ii. The Political Question Doctrine Applied: Congressional Self-Governance
  • Powell v. McCormack (1969) [Page 103]
    • o TOPIC:
    • o FACTS: The 90th Congress voted not to seat member-elect Powell due to his past improprieties. Congress (D) argued that the federal courts were barred from hearing the case due to the political question doctrine. Congress (D) argued that Article I, § 5 of the Constitution gives Congress (D) the sole authority to determine the qualifications for membership. Powell (P) argued that Article I, § 5 permitted Congress (D) to judge whether members-elect meet the membership qualifications outlined in the Constitution, but not the power to exclude those who met the stated qualifications.
    • o RULE: The political question doctrine does not bar the federal courts from deciding a case concerning Congress’s powers to determine its membership when the text of the Constitution does not specifically commit the issue in the case to Congressional resolution.
    • o ANALYSIS: Because the text of the Constitution does not commit to Congress a blanket authority to determine whether to seat a member-elect, the courts are not barred from deciding the issue. The rule to apply in this case is that the courts should refuse to decide cases in which the Constitution has explicitly committed the issue to the discretion of one of the other branches of government. Review of the documents and debates of the Framers of the Constitution leads to the conclusion that the phrase “be the Judge of the Qualifications of its own Members” in Article I, § 5 was not intended to give Congress authority to deny a seat to a duly elected member through a majority vote. This conclusion is consistent with the Framers’ requirement that the House have a super-majority in order to expel a member.
      • The federal courts consider issues of congressional self-governance to be political questions generally inappropriate for judicial resolution. Nevertheless, inPowellthe Court was willing to step in and enjoin Congress from refusing to seat the plaintiff, a member-elect of the House.
iii. The Political Question Doctrine Applied: Foreign Policy'Oetjen v. Central Leather Co.
  • v “The conduct of the foreign relations of our Government is committed by the Constitution to the Executive and Legislature ‘the political’ Departments of the Government, and the propriety of what may be done in the exercise of this political power is not subject to judicial inquiry or decision.”
  • Goldwater v. Carter (1979) [Page 106]
    • o FACTS: In conjunction with actions recognizing the People’s Republic of China, President Jimmy Carter terminated the United States treaty with Taiwan. The treaty was concerned with defense and other issues. The Senate debated a resolution that would have stated that Senate approval was required in the termination of this type of treaty. The Senate resolution never came to a vote. Some Members of Congress, led by Senator Barry Goldwater (P) sued, claiming that the President’s actions were unconstitutional, because Article II, Section 2 requires the Senate to ratify treaties.
    • o RULE: The Senate’s role in the termination of treaties is a nonjusticiable political question.
    • o ANALYSIS: The issue of whether the Senate is required to rescind foreign treaties or to ratify the President’s rescission is a political question beyond the scope of the federal courts. The issue is political because it involves a foreign policy decision of the President and a treaty involving potential commitments of military troops. The question of whether Congress is authorized to halt the President’s decision is likewise political. Although the Constitution directs the Senate to ratify treaties, it says nothing as to the rescission or termination of treaties. The President and Congress should resolve the dispute among themselves.
      • Dissent: 3 Baker Questions
        • The first question fromBakeris whether the text of the Constitution explicitly makes another branch of government responsible for resolving the dispute.
        • The secondBakerquestion is whether the dispute is beyond the court’s judicial expertise.
        • Finally, the thirdBakerquestion is whether the court should decline to address an issue for prudential reasons.
  • Zivotofsky v. Clinton (2012) [Page 108]
    • o FACTS: Congress enacted a statute that allowed Americans born in Jerusalem to list Israel as the place of birth on their passports. The State Department (D) declined to follow the law, citing its policy of not taking a position on the political status of Jerusalem. Zivotofsky (P) brought suit to enforce the statute, but the suit was dismissed as presenting a political question.
    • o RULE: Determining the constitutionality of a statute is not a political question.
    • o ANALYSIS: Congress enacted a statute that allowed Americans born in Jerusalem to list Israel as the place of birth on their passports. The State Department (D) declined to follow the law, citing its policy of not taking a position on the political status of Jerusalem. Zivotofsky (P) brought suit to enforce the statute, but the suit was dismissed as presenting a political question.
      • Dissent: Four prudential questions, taken together, lead to the conclusion that this case poses a political question: First, the issue arises in the field of foreign affairs. Second, answering the constitutional question may well require the courts to evaluate the foreign policy implications of foreign policy decisions. Third, the countervailing interests in obtaining judicial resolution of the constitutional determination are not particularly strong. Fourth, insofar as the case reflects different foreign policy views among the political branches of government, those branches have nonjudicial methods of working out their differences. This case has a minimal need for judicial intervention and poses a more serious risk of potentially disrupting sound foreign policy decision making.
iv. The Political Question Doctrine Applied: Impeachment and Removal
  • Nixon v. United States (1993) [Page 110]
    • o FACTS: A federal judge, impeached by the Senate, challenged the constitutionality of the Senate’s impeachment procedure.
    • o RULE: Federal courts are not authorized to review impeachments, which the Constitution explicitly confers to congress.
    • o ANALYSIS: A controversy is nonjusticiable if the Constitution commits the issue to another branch of government. Article I, section 3, clause 6 of the Constitution gives the Senate sole power to try all impeachments.Judicial review would be inconsistent with checks and balances because impeachment is the legislature’s only check on the judicial branch. Nixon (P) argues that without judicial review, the Senate could usurp judicial power.But, the judiciary is usurping the Senate’s power if it reviews this case
      • Justice Souter’s Concurrence: Whether an issue is a political question requires a case-by-case inquiry. The Senate has broad boundaries for determining the procedures for trying someone for impeachment. This case is nonjusticiable because we should adhere to a political decision that was already made and because we should avoid the potential embarrassment of multiple decisions on a single question. However, judicial review may be necessary if the Senate acted in a way that seriously threatened the integrity of its decision, convicting, say, upon a coin toss.

THE EXECUTIVE POWER[edit | edit source]

Foreign Affairs[edit | edit source]

'Treaty Power'Issue: The issue here is whether Congress may enact [statute] under their Treaty Power.Rule: Under the treaty power, the President has the power to enter into treaties with the advice and consent of the Senate (Art. II), and Congress will not be limited to their enumerated powers in passing legislature provided it doesn’t violate any prohibitory provisions of the Constitution (Reid). When the treaty in question is non-self-executing, then Congress’ legislature may effectuate the treaty’s obligations under the necessary and proper clause, provided the law is reasonably related to the end that is itself within power (McCulloch), but with wide latitude given (Kebadeaux). Here, it is [un/likely] that this law will be constitutional under Congress’ treaty power.Analysis': 'First: Because we do not have the facts necessary to determine that this treaty was entered into in a valid manner, there will be a cloud of uncertainty in regards to the ultimate validity of the legislature. However, in the event that this treaty was enacted appropriately, then it’s necessary to establish whether the treaty contradicts prohibitory constitutional provisions (Reid, Missouri). Here, the law in question seems as though it [does/does not] violate prohibitory provisions [if it does, explain which provision and why]. Further, it [does/does not] appear that the law here is clearly appropriate and adapted to enforcing the terms of this treaty (Kebadeaux).

  • Prohibitory provisions: It doesn’t violate the Constitution
    • The only areas really off limits are when the statute is against express prohibitive clauses of the Constitution
    • Congress will still have the power to enact if it isn’t listed under their enumerated powers
    • Self-Executing Treaty: requires no addt’l fed legislation to become enforceable in domestic courts
    • Non-Self-Executive Treaty: requires addt’l legislation to become valid in domestic court

Conclusion': Therefore, it is [un/likely] that Congress will be able to pass this law under the treaty power.Necessary and Proper Clause'Issue: Does the Necessary and Proper Clause allow Congress to enact [statute]?Rule: Congress may use the Necessary and Proper Clause to enact [statute] if it the statute “hooks” on to another power that Congress is able to exercise.

  • Necessary:
    • Comstock'': Congress may prescribe sanctions for crimes that they created,
    • NFIB'': Must be used in conjunction with valid exercise of another power of Congress; Cong can’t just create the necessity for a certain statute
    • Is a means appropriate and adapted to a legitimate end;
    • Hooks on to another valid power of Congress (not necessarily powers that are enumerated – Comstock);
  • Proper:
    • Doesn’t make constitution redundant
    • Is consistent with the letter and spirit of the Constitution
Case Name ODF Outcome

McCulloch v. Maryland

United States made a second bank, Maryland tried to tax it

Constitutional, the means were appropriate and adapted to the ends of an enumerated power

United States v. Comstock

Granted district courts the ability to order civil commitment of mentally ill, sexually dangerous federal prisoners past their release dates

Constitutional, look to the 5 factors provided for necessary and is proper

NFIB v. Sebelius

OBAMACARE

No, it does not latch on to an enumerated power and it undermined the structure of the government and tramples individual autonomy

10th Amendment Constraint on Federal Legislative Power'Issue': The issue is whether Congress has the power to regulate [state or local gov’t] in pursuant to the Tenth Amendment.Rule': Under the Tenth Amendment, any power not delegated to the federal government is retained by the states. So, Congress may not place an affirmative duty on states to regulate in a particular way unless the law is one that is generally applicable (Garcia). It is [likely/unlikely] that Congress will be able to do [x].Application': 'First (unconst’al): Here, this act seems to be placing an affirmative duty to [state] in order to [x]. The type of duty that Congress is imposing on [x] is similar to [New York/Printz] because [x]. This infringes on state sovereignty as Congress is essentially taking control of state legislatures and commandeering them to [x] to carry out a federal scheme (New York). In contrast, it could be said that this legislature instead prohibits conduct as opposed to placing any affirmative duties on [x] because [x].Second: Further, unlike the Fair Labor Standards Act in Garcia that was equally applicable to both state and private entities, this act here only seems to apply to the states because [x].First (const’al): Here, this act does not seem to place any duty on [x]. Unless it is shown that Congress is commandeering the legislature to [x] to carry out a federal scheme, it is unlikely that there is a violation of the state’s sovereignty (New York). Further, like the Fair Labor Standards Act in Garcia that applied to both state and private entities, this act functions similarly because [x].Second: Further, like the statute in [Garcia/Reno], this statute seems to be generally applicable because [x].Garcia: Like the Fair Labor Standards Act in Garcia that could apply to both state and private entities, this act here will likely be constitutional for because [x]. '''New York''': The law will be held unconstitutional if the federal government is attempting to use the state political subdivisions and is commandeering states to enforce a federal regulatory program. The federal government can shift the blame of a federal mistake to the statesPrintz''': federal law was found unconstitutional because it was commandeering state executive officials (gun background checks). Commandeering local law enforcement officers.Reno''': law for personal date into motor vehicle department database constitutional because federal government can regulate commercial activity that is generally applicable (Reno).Conclusion': Therefore, Congress [did/did not] have the power to regulate state governments to [x] under the commandeering doctrine of the Tenth Amendment.

Case Name Case Explanation Constitutional?

National League of Cities v. Usery

Fair Labor Standards Act was amended to remove exemption to employees of state institutions.

Unconstitutional. Federal government does not have the power to force states to give up traditional government functions.

Garcia v. San Antonio Metro. Transit Authority

US Dept of Labor asserted that the FLSA covered the Transit authority’s operations

Constitutional. Overruled Usery. The “Traditional Government Functions” test was unworkable.

New York v. United States

Federal law required state legislatures to enact laws or take possession and liability for nuclear waste

Unconstitutional. Commandeering of state government and legislature.No accountability.

Printz v. United States

Federal law required state police officers to conduct background checks for guns. Unconstitutional. Pushes costs on to states. Against original intent of the constitution.

Reno v. Condon

Federal law prohibited state DMVs from giving out personal information without consent of individuals.

Constitution. Statute is generally applicable to all institutions with such information. Not an affirmative duty, but a negative prohibition.

  • Ø Article II – Outlines the role of the executive branch
    • o Vesting Clause of Article II does not contain the limiting language seen in the Legislative Vesting Clause of Article I.
      • Some argue that the omission of this language means that the president is vested with ALL executive power, while others argue that the clause is merely the formal naming of the President as the holder of the executive powers enumerated in the Constitution.
    • Ø Even though there is a great deal over overlap, the President's powers can be broadly classified into three categories: foreign affairs, war powers, and domestic authority.
      • o Foreign Affairs: As the head of foreign affairs for the U.S. government, the President can, among other things, negotiate and enter into agreements with other countries, appoint ambassadors and ministers, and receive foreign ambassadors.
      • o War Powers: Also, though the Constitution divides war-making powers between the President and Congress, the President is the Commander-in-Chief of the armed forces.
      • o Domestic Authority: Domestically, the President is charged with 'taking care that the laws are faithfully executed,' appointing federal judges and other government officials, and signing or vetoing legislation passed by Congress.
    • Ø President’s powers are not unlimited—s/he can be held accountable for misconduct
      • o Impeachment; President can be sued, subject to privileges and immunities
      • o In United States v. Nixon, the Supreme Court held that executive privilege exists, but is not absolute. Specifically, the Court made clear that while a claim of executive privilege in order to protect national security or diplomatic secrets or to safeguard executive communications from exposure during a civil trial might be upheld, a generalized claim of confidentiality will not outweigh the compelling public interest in securing evidence for a criminal trial.
      • o The President is absolutely immune from legal liability for conduct taken while in office related to his presidential duties.
      • o But, the President's immunity is not absolute in all cases. The President may be held liable for conduct taken before or after his presidency, even if he is in office at the time of litigation. (Clinton v. Jones)

EXECUTIVE ORDER: An order from the President having the force and effect of law and commanding certain actions to be taken.Foreign Affairs

  • Ø The Constitution divides the power to manage relationships with other nations between Congress and the President, but the bulk of these rights and responsibilities are given to the President.
  • Ø Article II of the Constitution gives the President the authority to appoint ambassadors and foreign ministers, with the advice and consent of the Senate, as well as the power to receive ambassadors from other countries. Implied in this power is the right to refuse to receive foreign ambassadors.
    • o This means that the President effectively has the authority to determine whether or not to recognize a foreign government, which is critically important under international law.
  • Ø The President also has the primary authority for negotiating agreements with other countries.
  • Ø Although agreements between nations are called many things, for our purposes there are two types of international agreements: treaties and executive agreements.
    • o Treaties are agreements with other countries that are negotiated by the President and approved by two-thirds of the Senate, following the procedure set out in Article II, Section 2 of the Constitution.
    • o Executive Agreements are agreements with other countries made by the President acting alone.
  • Ø Treaties and executive agreements that violate the Constitution are unenforceable
    • o And just like a treaty, an executive agreement will supersede an inconsistent state law. This happened in U.S. v. Belmont.
    • o A treaty is on equal footing with federal law, and when a treaty comes into conflict with a federal statute, the last one enacted will prevail.
      • An executive agreement will never prevail over a federal statute.
    • Ø Curtiss-Wrightis probably the case most often used to justify the President's broad authority when it comes to foreign affairs.
      • o Justice Sutherland's opinion took the position that the national government's power to conduct foreign affairs is inherent, rather than enumerated or delegated. Because the President is the head, or sole organ, of the government with respect to foreign relations, Sutherland's expansive view of foreign affairs power means that the President has sweeping authority to control the United States' relations with other countries.
      • o The Supreme Court's opinion has been used since then as authority for the President to act without congressional approval under his foreign affairs power.
    • Ø The President is also named Commander in Chief of the armed forces.

War Powers

  • Ø The bulk of the country's war powers are given to Congress, including the power to declare war, raise and supply an army, make rules for the armed forces, and suppress rebellions.
  • Ø The Federalist Papers took the view that making the President Commander in Chief of the Armed forces was largely about ensuring civilian control over the military.
  • Ø It is at least clear that the President has authority to act to repel a sudden attack and the right to command the armed forces.
    • o What is less clear is how much Congress may limit or check the President's authority as Commander in Chief.
  • Ø The War Powers Resolution is a law designed to place some constraints on the President's exercise of war powers; specifically, the act requires the President only send U.S. troops into action with congressional approval unless there is a national emergency caused by an attack on the United States. The Resolution goes on to require that the President give Congress notice if troops are sent into battle and to limit the amount of time that troops can remain in action, without a declaration of war, to sixty days.
    • o The Resolution passed only by a congressional override of a presidential veto and has been violated by past presidents. Numerous questions have been raised about the law's constitutionality, and the debate continues today.
  • Ø War on Terror
    • o Following 9/11, Congress passed the Authorization for Use of Military Force Against Terrorists, (AUMF)
      • Gave the President sweeping authority to use all "necessary and appropriate force" against those he determined planned, assisted, or committed the attacks.
    • o A plurality of the Court inHamdi v. Rumsfeldagreed that the President may detain a U.S. citizen as an enemy combatant for the duration of hostilities, but that a citizen is still entitled to some due process in order to challenge his or her classification. Justice Scalia's dissent asserted that a U.S. citizen can never be detained without charges unless the writ of habeas corpus has been suspended and suggested that the government's only option in the case was to bring criminal charges against Hamdi.
    • o InHamdan, the Court held that the Executive Branch did not have the authority to establish military commissions that are contrary to the Constitution, federal law, the laws of war, and the Geneva Conventions. In footnote 23 of the majority opinion, Justice Stephens includes an important statement about executive power: "Whether or not the President has independent power, absent congressional authorization, to convene military commissions, he may not disregard limitations that Congress has, in proper exercise of its own war powers, placed on his powers. SeeYoungstown Sheet & Tube Co. v. Sawyer, 343 U. S. 579, 637 (1952) (Jackson, J., concurring)."
    • o InBoumediene, the Supreme Court held that the protections of habeas corpus applied to detainees held at Guantanamo Bay.

Domestic Authority[edit | edit source]

  • Ø The Presentment Clause of Article I
    • o The Presentment Clause of Article I requires that every bill passed by Congress be presented to the President for signature. If the President signs the bill, it becomes a law. The President is not obligated to sign a bill, however. Under the terms of the Presentment Clause, the President may return the bill with objections for reconsideration.
      • President can sign, veto, send back, or not sign while Congress is in session (bill automatically takes effect).
      • President cannot exercise Line-Item Veto (the power of a president, governor, or other elected executive to reject individual provisions of a bill).
      • The Supreme Court inClintonconcluded that the Line-Item Veto violates the formal requirements of the Presentment Clause and the separation of powers.
      • Using a similarly formalistic approach, the Court inChadhaheld that the legislature could not reserve for itself a legislative veto on the grounds that it violated requirements of bicameralism and presentment.
      • Pocket veto: an indirect veto of a legislative bill by the president or a governor by retaining the bill unsigned until it is too late for it to be dealt with during the legislative session.
    • Ø Appointment Power
      • o The Constitution gives the President the authority to appoint ambassadors and foreign ministers, federal judges, and other government officials, with the advice and consent of the Senate.
      • o 3 categories of government officials: principal officers, inferior officers, and employees.
        • Principal actors must be nominated by the President and confirmed by the Senate; these are high-level officials with broad discretion who report directly to the President.
        • Congress may grant authority to appoint inferior officers to the President, the courts, or department heads, and there is no need for confirmation by the Senate; these are lower-ranking officials with less discretion, like undersecretaries.
        • Employees and other nonofficers are not mentioned in the Constitution; fulfilling these roles is pretty much left up to Congress.
      • o How to determine if principal officer or inferior officer?
        • How are they removed?
        • Constraints on their jx, responsibilities, tenure?
        • Degree of supervision?
      • o Courts have recognized the President's inherent power as head of the executive branch to remove principal officers, without seeking approval from the Senate. SeeMyers v. United States
        • Congress may not usurp the President's authority to remove executive officers.
      • o But with respect to certain types of interbranch appointments, like the appointment of a special prosecutor, Congress can place some constraints on the President's removal authority, provided those restrictions don't impede the President's ability to carry out his duties. SeeMorrison v. Olson(1988).
    • Ø Take Care Clause
      • o Article II commands the President to "take care that the Laws be faithfully executed."
        • Broad interpretation (grant of power) vs. narrow interpretation (limit on executive authority)
      • o In any event, inNeagle, the Court found that the Take Care clause gave executive branch officials to act to protect national interests, even without statutory authority.
      • o The Take Care clause is often given as partial justification, along with delegated authority from Congress, for the President's authority to issue executive orders, which have the force of law.
    • Ø Steel Seizures case (Youngstown Sheet)
      • o President Truman issued an executive order to his Secretary of Commerce instructing him to seize the country's steel mills, because the steel workers were threatening to strike in the middle of the (undeclared) Korean War.
      • o Justice Black, writing for the majority, said that President Truman had unconstitutionally exceeded the bounds of his authority and engaged in lawmaking when he ordered the seizure. Justices Frankfurter and Jackson placed considerable importance on the fact that Congress had considered authorizing the President to make seizures of the type in this case but elected not to. Justice Vinson's dissent concluded that the seizure was within the President's emergency powers and made much of the fact that President Truman had gone to Congress before the seizure and Congress made no effort to stop him.
      • o BUT: J. Jackson’s concurrence! He said there were 3 categories into which executive action could fall:
        • First, when a President acts with Congress's express or implied approval, his authority is at its maximum because he is exercising all of his own power as well as all that Congress can delegate.
        • At the other end of the spectrum, when the President is acting contrary to the express or implied will of Congress, his power is at its lowest ebb, because he can act only on his own authority less any power Congress can take.
        • Things get difficult in the middle, in Justice Jackson's "zone of twilight." When Congress has said nothing, the President is acting solely under his own authority.

Inherent Presidential Power

  • Youngstown Sheet & Tube Co. v. Sawyer (1952) [Page 322]
    • o TOPIC: President acting w/o congressional authority
    • o FACTS: During the Korean War, a strike by workers at the nation’s largest steel mills was imminent. In order to prevent the strike, which the President feared would severely hamper the war effort, President Truman issued an Executive Order 10340 directing the Secretary of Commerce, Charles Sawyer, (“Secretary”) (D) to take possession of most of the steel mills and keep them running. Truman believed that the steel strike could endanger the national defense and the war effort in Korea because steel was indispensable for all weapons. The Secretary (D) then ordered the presidents of the various steel companies to act as his operating managers for the steel mills, which they did under protest. Congress was twice informed of the President’s and Secretary’s (D) actions, but Congress remained silent on the matter. The companies (P) brought suit in federal court, arguing that the President acted outside of the scope of his constitutional authority. The Secretary (D) defended the President’s actions, arguing that the President was acting within his power as chief executive and commander-in-chief of the nation’s armed forces. The lower courts granted an injunction against the Secretary (D).
    • o RULE: The President of the United States does not have the inherent authority to order the involuntary surrender of private property to the government.
    • o ANALYSIS
      • Justice Black writing for the majority:
        • “The President’s power, if any, to issue the order must stem either from an act of Congress or from the Constitution itself.”
      • Justice Jackson three-part test:
        • 1. When the President acts pursuant to an express or implied authorization of Congress, his authority is at its maximum, for it includes all that he possesses in his own right plus all that Congress can delegate.
          • o In these circumstances, and in these only, may he be said (for what it may be worth), to personify the federal sovereignty. If his act is held unconstitutional under these circumstances, it usually means that the Federal Government as an undivided whole lacks power. A seizure executed by the President pursuant to an Act of Congress would be supported by the strongest of presumptions and the widest latitude of judicial interpretation, and the burden of persuasion would rest heavily upon any who might attack it.
        • 2. When the President acts in absence of either a congressional grant or denial of authority, he can only rely upon his own independent powers, but there is a zone of twilight in which he and Congress may have concurrent authority, or in which its distribution is uncertain.
          • o Therefore, congressional inertia, indifference or quiescence may sometimes, at least as a practical matters, enable, if not invite, measures of independent presidential responsibility. In this area, any actual test of power is likely to depend on the imperatives of events and contemporary imponderables rather than on abstract theories of law.
        • 3. When the President takes measures incompatible with the expressed or implied will of Congress, his power is at its lowest ebb, for then he can rely only upon his own constitutional powers minus any constitutional powers of Congress over the matter.
          • o Courts can sustain exclusive Presidential control in such a case only by disabling the Congress from acting upon the subject. Presidential claim to a power at once so conclusive and preclusive must be scrutinized with caution, for what is at stake is the equilibrium established by our constitutional system.
        • (1) where the president acts with congressional approval, (2) where the president acts in light of congressional silence, and (3) where the president acts in light of express or implied congressional disapproval. Under the first category, the president’s power is at its highest and the president can act as long as Congress uses its power properly (which means Congress cannot give the president any type of power, for example, the ability to make laws). This area includes the president acting pursuant to a self-executing treaty. The second category is a grey area, which requires a proper factual evaluation. The president is walking a fine line when it is unclear whether Congress approves, or disapproves of his actions. Under the last category (clear congressional disapproval), the president can only properly use his independent powersgranted to him in the Constitution.
      • Four different approaches can be identified in the opinions in Youngstown; these varying approaches also are reflected in numerous other cases.
        • There is no inherent presidential power; the president may act only if there is express constitutional or statutory authority. (Majority opinion)
        • The president has inherent authority unless the president interferes with the functioning of another branch of government or usurps the powers of another branch.
        • The president may exercise powers not mentioned in the Constitution so long as the president does not violate a statute or the Constitution.
        • The president has inherent powers that may not be restricted by Congress and may act unless the Constitution is violated.
        • The president may exercise powers and __ in the Constitution so long as the president does not violate a statute or the Constitution (this is the model that has been adopted by the majority of courts.)
      • *Note also that this case involved the President acting in the context of a domestic dispute. In the field of foreign affairs, the President is much less restricted.

Executive Privilege: the privilege, claimed by the president for the executive branch of the US government, of withholding information in the public interest.

  • U.S. v. Nixon (1974) [Page 332]
    • o TOPIC: President is not beyond the scope of the judicial process
    • o FACTS: [Several of President Nixon’s (P) associates were indicted on charges of conspiracy and obstruction of justice in the Watergate investigation, though the President (P) himself was not indicted. Formal criminal proceedings followed.] The District Court issued a subpoena duces tecum ordering the President (P) to produce the tape recordings of his conversations in one of the criminal cases. The President (P) brought this motion to quash the subpoena on the grounds of executive privilege and separation of powers. The District Court denied the President’s (P) motion to quash, and the Supreme Court granted review.
    • o RULE: Conversations between the President and his advisors are generally privileged, but the privilege is not absolute.
    • o ANALYSIS: The court made 3 major points
      • First, the Court held that it is the role of the Court to decide whether the president has executive privilege and, if so, its scope.
        • Nixon claimed that the Constitution gave the president executive privilege and that the president alone determined its reach. The Court rejected this contention: “The President's counsel, as we have noted, reads the Constitution as providing an absolute privilege of confidentiality for all Presidential communications. Many decisions of this Court, however, have unequivocally reaffirmed the holding of Marbury v. Madison, that ‘[i]t is emphatically the province and duty of the judicial department to say what the law is.’ ” One can question, though, whether Marbury really stands for this proposition. Marbury obviously establishes the power of judicial review of executive actions, but as Professor Gerald Gunther argued, “there is nothing in Marbury v. Madison that precludes a constitutional interpretation which gives final authority to another branch.”
      • Second, the Court recognized the existence of executive privilege.
        • The Court recognized that the need for candor in communications with advisors justified executive privilege; indeed, the Court said that a need for confidentiality was “too plain to require further discussion.” Although Article II of the Constitution does not expressly grant this power to the president, the Court said that “the privilege can be said to derive from the supremacy of each branch within its own assigned area of constitutional duties. Certain powers and privileges flow from the nature of enumerated powers; the protection of the confidentiality of Presidential communications has similar constitutional underpinnings.”
        • United States v. Nixon thus recognizes executive privilege as an inherent presidential power. Compare to Youngstown.
      • Third, the Court held that executive privilege is not absolute, but rather must yield when there are important countervailing interests.
        • The Court explained that “neither the doctrine of separation of powers, nor the need for confidentiality of high-level communications, without more, can sustain an absolute, unqualified presidential privilege of immunity from judicial process under all circumstances.”
        • More specifically, the Court said that an absolute privilege would interfere with the ability of the judiciary to perform its constitutional function. The Court explained: “The impediment that an absolute, unqualified privilege would place in the way of the primary constitutional duty of the Judicial Branch to do justice in criminal prosecutions would plainly conflict with the function of the courts under Article III.”
        • The Court thus concluded that the need for evidence at a criminal trial outweighed executive privilege. The Court said that allowing “the privilege to withhold evidence that is demonstrably relevant in a criminal trial would cut deeply into the guarantee of due process of law and gravely impair the basic function of the courts.”

The Authority of Congress to Increase Executive Power

  • Clinton v. City of New York (1998)
    • o TOPIC: Increasing veto power
    • o FACTS: The Line Item Veto Act (“the Act”) of 1996 purported to give the President (D) the power to cancel discrete provisions of certain spending and taxing bills after they have been signed into law. It sets out specific procedures that the President (D) must follow in making cancellations under the Act. Included is a procedure whereby the President (D) must notify the Congress of the cancellation, and the Congress will have the opportunity to resuscitate the cancelled measures by a majority vote. The President (D) used his line-item veto power to cancel an item of spending that would benefit only the State of New York (P) and also to cancel a tax break to farmers’ cooperatives (P). It is undisputed that the President (D) complied with the Act, that the Congress properly enacted the Act, and that the taxing and spending provisions at issue fall within the Act.
    • o RULE: The Line Item Veto Act is unconstitutional. If the president wishes to exercise his veto power he must veto a bill in its entirety before signing it.
    • o ANALYSIS: The President’s (D) role in lawmaking is limited to initiating, influencing, and vetoing legislation. The President’s (D) cancellation authority in a “line-item veto” differs from that of a constitutional veto. The constitutional veto takes place before the bill becomes law, whereas the statutory cancellation occurs after the bill becomes law. Therefore, it is a repeal, rather than a veto. In addition, the constitutional veto is of the entire bill, whereas the statutory cancellation is of only discrete parts. The power to enact statutes may only be exercised in accord with a single, finely wrought and exhaustively considered process.
      • Separation of powers concerns normally arise when one branch attempts to take over another branch’s power for itself, thus aggrandizing itself at the other’s expense. Here, however, the opposite is true; the question is whether the legislature was inappropriately giving power to the executive.

Checks on the President – Suing/Prosecuting + Impeachment[edit | edit source]

  • Nixon v. Fitzgerald (1982) [Page 439]
    • o TOPIC: President is immune from suit from his official acts.
    • o FACTS: Fitzgerald, an analyst in the Air Force, alleged that his job was eliminated in unconstitutional retaliation for his exposing cost overruns in the Defense Department in testimony to Congress.
    • o RULE: The President of the United States is shielded by absolute immunity from civil damages liability for acts done in his official capacity as President.
    • o ANALYSIS: In a 5-to-4 decision, the Court held that the president's “unique status under the Constitution” and the “singular importance” of the duties of the office justify absolute immunity from money damages.
      • The Court emphasized that it feared that frequent suits against the president would detract from his or her ability to perform effectively.
      • The Court concluded not only that a sitting president was immune from suit, but also that “a former President … is entitled to absolute immunity from damages liability predicated on his official acts.”
      • The Court explained that there are other checks against the president, ranging from formal ones, such as impeachment, to more informal ones, such as political pressure.
      • The dissent responded that these checks do not provide compensation to an individual injured by unconstitutional presidential actions. The dissent lamented that the decision “places the President above the law,” and that granting absolute immunity to the President for official acts is an abandonment of the most basic principle that the United States is a government of laws, and not of men.
  • Clinton v. Jones (1997) [Page 442]
    • o TOPIC: A sitting president is not immune from suit for unofficial acts.
    • o FACTS: President Clinton (D), accused of making inappropriate sexual advances toward a subordinate while Governor of Arkansas, sought to postpone the proceeding of a civil lawsuit until he left office. Bill Clinton was sued for sexual harassment that allegedly occurred while he was governor of Arkansas. Clinton moved in federal district court to dismiss the suit, or at least to have it stayed until he was no longer the president. The federal district court ruled that a president does not have absolute immunity to suits for conduct prior to becoming president, but the court used its discretion to stay the trial until after the completion of Clinton's tenure in the White House. The Eighth Circuit affirmed and the Supreme Court granted certiorari.
    • o RULE: A sitting President does not enjoy temporary immunity from all civil lawsuits based on his unofficial acts taken prior to becoming President.
    • o ANALYSIS:
      • The Court explained that immunity exists to safeguard the exercise of discretion by an officeholder; thus, there is no basis for “immunity for unofficial conduct.”
      • The Court expressly rejected President Clinton's claim that civil suits would unduly interfere with the president's carrying out the important and unique constitutional functions of the office.
        • “[I]n the more than 200-year history of the Republic, only three sitting Presidents have been subjected to suits for their private actions. If the past is any indicator, it seems unlikely that a deluge of such litigation will ever engulf the Presidency. As for the case at hand, if properly managed by the District Court, it appears to us highly unlikely to occupy any substantial amount of petitioner's time.”
      • On the one hand, Justice Stevens was hardly prescient; the civil suit against Clinton took a huge amount of the president's time and ultimately led to impeachment. Some believe that it is for exactly this reason, the inevitable distraction and burden of civil suits, that litigation against the president should be stayed until he is out of office.
      • Clinton v. Jones is a powerful statement that no one, not even the president, is above the law and delaying suits for as much as eight years often may preclude meritorious suits from going forward. The Supreme Court, though, unanimously and unequivocally held that presidents may be sued for acts that allegedly occurred prior to taking office.
      • The Court was not persuaded by Clinton’s (D) claims that allowing this suit to go forward will result in harassing and frivolous litigation against future presidents, or that national security concerns might prevent the President from explaining a legitimate need for a continuance. The courts have tools and methods for dealing with both of these concerns.
      • Note the Court’s unwillingness to extend executive immunity any further than necessary. This case is an example of the narrowness of the Court’s decision inNixon v. Fitzgerald.Immunity of any kind applies only to official acts of the President. The immunity ends when the rationale for immunity ends. Immunity is extended to official acts in order to encourage the President to act fearlessly for the public good. However, no such encouragement is needed for unofficial acts, and thus there is no immunity of any type in that regard.

Impeachment and Removal

  • v Article II, §4: “The President, Vice President and all civil Officers of the United States, shall be removed from Office on Impeachment for, and Conviction of, Treason, Bribery, or other high Crimes and Misdemeanors.”
  • v Article I, §2 provides that the House of Representatives has the sole power to impeach. If there is an impeachment by the House, then a trial is held in the Senate. Article I, §3 gives the Senate the sole power to try impeachments and prescribes that “no Person shall be convicted without the Concurrence of two thirds of the Members present.”
  • v Two major issues remain unresolved concerning these provisions.
    • o First, what are “high Crimes and Misdemeanors”?
    • o Second, what procedures must be followed when there is an impeachment and removal proceeding?

The Constitutional Problems of the Administrative State

  • v Although federal agencies and departments have existed in some form since the beginning of American history, it is only in the last century that Congress has routinely delegated its legislative power to executive agencies.
  • v Administrative agencies generally have legislative power, as they possess the authority to promulgate rules that have the force of law. Administrative agencies also have executive power, because they are responsible for bringing enforcement actions against those who violate the relevant federal laws and regulations. Finally, administrative agencies frequently have judicial power in that they employ administrative law judges who hear cases brought by agency officials against those accused of violating the agency’s regulations.
  • v The combination of legislative, executive, and judicial power in the same hands is troubling. James Madison wrote, “The accumulation of all powers, legislative, executive, and judiciary, in the same hands . . . may justly be pronounced the very definition of tyranny.” The Federalist No. 47.

1). The Nondelegation Doctrine & Its Demise

  • ALA Schechter Poultry Corp v. US (1935) [Page 343]
    • o TOPIC: Legislature cannot delegate to the executive branch the unrestrained authority to make law
    • o FACTS: The National Industrial Recovery Act, a key piece of New Deal legislation, authorized the president to approve “codes of fair competition” developed by boards of various industries. Pursuant to this law, the president approved a Live Poultry Code for New York City. In part, the Code required sellers to sell only entire coops of chickens or half coops of chickens and made it illegal for buyers to reject individual chickens. The Code also regulated employment by requiring collective bargaining, prohibiting child labor, and establishing a 40-hour workweek and a minimum wage.
    • o RULE: Congress may not delegate legislative power to the executive without outlining strict standards for how the executive is to exercise that power.
    • o ANALYSIS: In short, the NIRA sets up no specific standards for the President to apply in determining whether to accept or reject proposed codes, aside from the general aims of rehabilitation, correction, and expansion. This leaves the discretion of the President virtually unfettered. Thus, the code-making authority granted to the President is an unconstitutional delegation of power.
      • “Congress cannot delegate legislative power to the President to exercise an unfettered discretion to make whatever laws he thinks may be needed or advisable for the rehabilitation and expansion of trade or industry.”
      • Notice that the Schechter Poultry and Panama Oil cases, other cases invalidating legislation on the basis of the non-delegation doctrine, both involve New Deal legislation. Much New Deal legislation was heavily scrutinized and vulnerable to attack on constitutional grounds. The country was undergoing vast economic, political, and social change at this time in history. It was also a time when the federal government was growing tremendously, both in its size and in its scope. The non-delegation doctrine, which worked well to preserve checks and balances in an era of limited government power and activity, would eventually give way to the modern administrative state.
  • Panama Refining Co. v. Ryan (1935) [Page 344]
    • o TOPIC: Congressional delegation of power to the executive branch must be specific and limited
    • o FACTS: Congress enacted a provision in the National Industrial Recovery Act that gave the President the power to prohibit the transportation of petroleum products in excess of the amount permitted by state law. Based on this provision, the President promulgated an Executive Order making such a prohibition. Congress delegated to the President the power to restrict or prohibit the interstate and foreign transport of petroleum.
    • o RULE: It is a violation of the separation of powers for Congress to delegate law-making authority to the President without imposing standards or rules limiting that authority.
    • o ANALYSIS: Congress is not permitted to abdicate to others the essential legislative functions with which it is vested. The Constitution is flexible enough to allow the Congress to assign to certain administrative instrumentalities the ability to make subordinate rules—within prescribed limits—and to make certain factual determinations to which legislative enactments shall apply. However, if our constitutional system is to be maintained, the necessity of administrative authority cannot override the limitations on congressional authority to delegate. If we were to hold the legislation in question valid, Congress would be free to delegate authority at will to the President, another officer, or an administrative body. The delegation of authority was unlawful and invalid.
    • o The non-delegation doctrine is an attempt by the Court to keep the constitutional structure of checks and balances solid by restricting the amount of lawmaking that an administrative agency is allowed to effect.
  • Whitman v. American Trucking Associations, Inc. [Page 345]
    • o The Court unanimously rejected a challenge to the federal Clean Air Act as an impermissible delegation of legislative power.
    • o The District of Columbia Circuit declared unconstitutional the EPA’s air quality regulations, particularly as to ozone levels, on the ground that there was an impermissible delegation of power.
    • o The Court unanimously upheld the delegation and the constitutionality of the EPA's air quality regulations.
      • “In a delegation challenge, the constitutional question is whether the statute has delegated legislative power to the agency. Article I, §1, of the Constitution vests “[a]ll legislative Powers herein granted . . . in a Congress of the United States.” This text permits no delegation of those powers, and so we repeatedly have said that when Congress confers decision-making authority upon agencies Congress must “lay down by legislative act an intelligible principle to which the person or body authorized to [act] is directed to conform.””
      • “In the history of the Court we have found the requisite “intelligible principle” lacking in only two statutes, one of which provided literally no guidance for the exercise of discretion, and the other of which conferred authority to regulate the entire economy on the basis of no more precise a standard than stimulating the economy by assuring “fair competition.” See Panama Refining Co. v. Ryan (1935); A.L.A. Schechter Poultry Corp. v. United States (1935)”
      • “EPA … set air quality standards at the level that is ‘requisite’—that is, not lower or higher than is necessary—to protect the public health with an adequate margin of safety, fits comfortably within the scope of discretion permitted by our precedent.”

2). The Legislative Veto & Its Demise

  • INS v. Chadha (1983) [Page 347]
    • o TOPIC: Legislative Veto is unconstitutional.
    • o FACTS: Chadha (P) was an alien who was lawfully admitted to the United States on a nonimmigrant student visa. He overstayed his visa, and the Immigration and Naturalization Service (INS) (D) initiated deportation proceedings against Chadha (P) in immigration court. [Both the INS (D) and the immigration court are within the Department of Justice, over which the Attorney General has ultimate authority.] The immigration judge found that Chadha (P) met the requirements set out in the Immigration and Nationality Act (the Act) for suspension of deportation. Pursuant to the Act, the Attorney General reported the suspension of deportation to Congress. Under the Act, either house of Congress had the option to pass a resolution to “veto” the Attorney General’s determination that Chadha’s (P) deportation should be suspended. The House of Representatives passed such a resolution, finding that Chadha (P) (along with five others) did not meet the statutory requirements for suspension of deportation. The House action was not submitted to the Senate or presented to the President.
    • o RULE: Legislative action is not legitimate unless there is bicameral approval and presentment to the President.
    • o ANALYSIS
      • Chief Justice Burger: Congress may only exercise its legislative power of the federal government in accord with a single, finely wrought and exhaustively considered procedure. Nearly every legislative act, In order to have the force and effect of law, must be considered and passed by both houses of Congress and then presented to the President for his signature. The framers of the Constitution considered these bicameralism and presentment requirements to be essential. The President’s participation in the legislative process was to protect the Executive Branch from Congress and to protect the people from congressional enactment of oppressive, improvident, and ill-considered laws. The division of Congress into two distinct bodies assured that laws would be enacted only after an opportunity for study and debate in separate settings.
      • Chief Justice Burger expressly rejected the position that the legislative veto was necessary to ensure adequate checks and balances.
        • “The choices we discern as having been made in the Constitutional Convention impose burdens on governmental processes that often seem clumsy, inefficient, even unworkable, but those hard choices were consciously made by men who had lived under a form of government that permitted arbitrary governmental acts to go unchecked. There is no support in the Constitution or decisions of this Court for the proposition that the cumbersomeness and delays often encountered in complying with explicit Constitutional standards may be avoided, either by the Congress or the President.”
      • While Chief Justice Burger's majority opinion was highly formalistic, Justice White's dissent was functional.
        • Burger emphasized the formal structure prescribed in the Constitution for adopting laws and dismissed the functional concern that the legislative veto was essential to check administrative power. White, in contrast, stressed the fact that over 200 federal laws contained legislative vetoes reflecting Congress's judgment that this was an essential tool for checking the exercise of delegated powers.
      • The dispute among the Justices in Chadha was over the proper form of analysis in separation of powers cases. Should the evaluation of the constitutionality of the legislative veto rest entirely on the text of the Constitution and the framers' intent, or should the Court consider the functional justification for legislative vetoes? Neither the majority nor the dissent addressed whether the legislative veto is actually an effective tool for checking administrative agencies. Chadha involved a legislative veto of an adjudicatory proceeding; Congress, by resolution of the House of Representatives, overturned an immigration judge's decision to allow Chadha to remain in the country.
      • Almost immediately after Chadha, the Court extended its holding to preclude legislative vetoes of agency rules. It is thus clearly established that if Congress wants to overturn an executive action, there must be bicameralism, passage by both houses of Congress, and presentment, giving the bill to the president for signature or veto. Anything less is a legislative veto, and legislative vetoes are unconstitutional.

3). Checking Administrative Power

  • v What other mechanisms exist to check administrative agencies and are they sufficient?
    • o Congress can control administrative agencies through statutes. For instance, laws can be enacted directing agencies to perform certain tasks or denying them authority in particular areas. Also, Congress can overturn agency decisions by statute, following the prescribed procedures for bicameralism and presentment. The president, of course, can veto such statutes, requiring that Congress act by a two-thirds vote to effectuate such a check.
    • o Furthermore, Congress controls the budget of administrative agencies and can use this to exercise an important check on their work. Congressional committees that oversee particular agencies often play a key role in monitoring and controlling agency actions.
    • o Another important check on agencies is the appointment and removal power. The president’s authority to select members of agencies, subject to confirmation by the Senate, often directs the conduct of the agencies. Also, the president’s power to remove agency officials is another check. The appointment and removal power have generated many constitutional issues concerning separation of powers and are worth examining.

The Appointment Power Article II, § 2

  • Alexa Morrison v. Theodore B. Olson (1988)
    • o TOPIC: President does not have exclusive authority to appoint executive officers
    • o FACTS: Title VI of the Ethics in Government Act allows for the appointment of an “independent counsel” to investigate and, if appropriate, prosecute certain high-ranking Government officials for violations of federal criminal laws. Title VI provided that he or she could be removed by the attorney general only for cause. If an independent counsel were removed, the attorney general would have to file a report with the panel of judges who made the appointment and with the House and Senate Judiciary Committees.
    • o RULE: A law vesting the judiciary with the power to appoint an inferior executive officer (an independent counsel) and prohibiting removal without cause does not violate separation of powers principles.
    • o ANALYSIS: The Appointments Clause, Art. II, § 2, cl. 2, divides executive officers into two classes: principal officers and inferior officers. The appointment of principal officers is for the President with the advice and consent of the Senate. However, for inferior officers the Congress may provide for appointment by the President alone, by the heads of departments, or by the Courts. Thus, this case turns upon whether the independent counsel is a “principal” or “inferior” officer. The independent counsel is clearly an inferior officer for several reasons. First, she is subject to removal by a higher executive branch official. Second, she is empowered by the Act to perform only certain, limited duties—namely the investigation and prosecution of specific crimes by specific government officials. Third, the office of independent is limited to the jurisdiction granted by the Special Division. Finally, her office is limited in tenure—it is temporary and for the limited purpose of accomplishing a single task.
    • o Limitations on the appointment power of the President are a check on the power of administrative agencies. The Appointments Clause limits presidential appointments of principal officers only with the approval of the Senate. It also allows Congress to determine the method of appointment of inferior officers—Congress can leave that job up to the President alone, the heads of the Departments, or to the Judiciary. InMorrison v. Olson,note that it did not seem to bother the Court that Congress used a combination—both the Attorney General (Executive Officer) and the Judiciary (the Special Division made up of three judges of the D.C. Circuit Court of Appeals). However, the one limit is that Congress may not give appointment power to itself—the Constitution limits the grant of appointment power to the executive or judicial branches.
    • o The Court noted that earlier cases, such as Humphrey’s Executor and Weiner, had drawn a distinction between purely executive tasks and those that were quasi-legislative or quasi-judicial. The Court said that while it did “not mean to suggest that an analysis of the functions served by the officials at issue is irrelevant … the real question is whether the removal restrictions are of such a nature that they impede the president's ability to perform his constitutional duty.” The Court stressed that the independent counsel, who exists to investigate and prosecute alleged wrongdoing in the executive branch of government, ideally should be independent of the president.
    • o The Court also emphasized that the statute does not prohibit all removal; rather, it allows the attorney general to fire an independent counsel for “good cause.” Hence, the Court concluded that the limits on the removal of the independent counsel did not violate the Constitution.
  • NLRB v. Noel Canning (2014)
    • o RULE: The Recess Appointment power allows the President to make appointments to fill any vacancy whenever the Senate declares that it is in recess for ten days or more.

Myers, Humphrey’s Executor, and Bowsher all use the analysis for removal of administrative officers. If it’s an executive function, then executive should remove. If it’s a legislative function, then legislature should remove.REMOVAL

Officer Appointment Limitations on Removal
Superior Officers President Only(Art II)(Buckley) For cause restrictions OK when function is quasi-legislative or quasi-judicial (Humphreys). Even officials exercising executive functions may be protected from at will removal where restrictions do not impede president from performing constitutionally assigned powers and duties (Morrison).Congress cannot itself maintain control over removal of officer exercising executive functions (Bowsher).President may be restricted over removal of quasi-judicial officers even in absence of legislation limited removal only for cause (Weiner).Congressional involvement in removal decisions particularly suspect (Myers, Bowsher).
Inferior Officers Courts, Heads of Departments or President (Art II) For cause restrictions usually acceptable, even for officials performing executive functions (Morrison), unless double layer of protection excessively precludes presidential control (PCAOB).

Separation of Powers and Foreign PolicyAre Foreign Policy and Domestic Affairs Different?

  • US v. Curtiss-Wright Export Corp. (1936)
    • o TOPIC: The president has broad authority to conduct foreign affairs.
    • o FACTS: A weapons manufacturer (D) was convicted of selling arms to warring nations in South America in violation of an Executive Order that was promulgated pursuant to a Joint Resolution of Congress (which authorized the president to ban the sales of arms to countries involved in the Chaco border dispute [Bolivia and Paraguay]).
    • o RULE: The non-delegation doctrine does not bar Congress from delegating great authority and discretion to the President in the conduct of foreign affairs.
    • o ANALYSIS: This case demonstrates that there is a strong rationale for allowing the President wide latitude in the conduct of foreign affairs. So far, the other branches of government have acquiesced in the President’s increasing power in the conduct of foreign affairs. As this case underscores, the President has broad authority—even lawmaking authority—when it comes to foreign affairs.
  • Zivotofsky v. Kerry (2015)
    • o FACTS: The U.S. State Department (D) declined to follow a law allowing Americans born in Jerusalem to have their place of birth listed as “Israel,” and Zivotofsky (P) brought suit.
    • o RULE: The President has the exclusive power to recognize foreign nations and governments.
    • o ANALYSIS: In 1995, Congress passed the Jerusalem Embassy Act, P.L. 104–45, which required moving the U.S. Embassy in Israel from Tel Aviv to Jerusalem. The bill became law without Presidential approval, and every President has declined to implement the law, saying that it is an unconstitutional interference with Presidential powers.

Treaties and Executive Agreements

  • Dames & Moore v. Regan, Secretary of the Treasury (YEAR) [Page 00]
    • o TOPIC: An executive agreement has the same force and effect as a treaty and can alter the rights of U.S. citizens
    • o FACTS: The President ordered the dismissal of pending litigation against the government of Iran in U.S. courts and forced the claims into arbitration pursuant to an “executive agreement.”
    • o RULE: The President has the power to settle claims by U.S. citizens against foreign governments, even without the consent of the U.S. citizens whose claims are compromised.
    • o ANALYSIS: The debate in this case is whether the President can enter into “executive agreements” that are binding and have the force and effect of law. When the President acted unilaterally to take the claims of Dames & Moore (P) out of the hands of the U.S. courts and into binding arbitration, he clearly enacted an enforceable law (through his Executive Orders) that changed the parties’ rights and relationships. As demonstrated inINS v. Chadha, all legislation is normally required to go through the process of enactment and presentment, not to mention that the Constitution requires the legislation start in Congress. This case, approving such a radical departure from the normal course of events, seems to establish quite clearly that the President had extraordinary power in matters of foreign affairs.
    • o EXCEPTION TO CHADHA!

War Powers[edit | edit source]

Presidential Power and the War on Terrorism''''

  • Hamdi v. Rumsfeld (2004): Combatants may be detained throughout duration of conflict but must have opportunity to challenge combatant status.
  • Rasul v. Bush (2004): Statutory grant of habeas jurisdiction to federal courts extends to Guantanamo Bay (28 U. S. C. §2241, which authorizes district courts, “within their respective jurisdictions,” to entertain habeas applications).
  • Detainee Treatment Act of 2005: stripped courts of jurisdiction over Guantanamo detainees, provided limited review in DC Circuit as to whether Combatant Status Review Tribunal decision justified.
  • Hamdan v. Rumsfeld (2006): DTA did not strip habeas jurisdiction in pending cases and military tribunals violate Geneva convention and laws of war.
  • Military Commissions Act of 2006: responded to Hamdan by stripping all habeas in any case pending or not, and corrected some of the due process problems of military tribunals identified in Hamdan.

Detentions

  • Hamdi v. Rumsfield (YEAR) [Page 00]
    • o TOPIC:
    • o FACTS: Hamdi (P) was an American citizen in Afghanistan. In 2001, the Northern Alliance detained Hamdi (P) for allegedly cooperating with the Taliban and surrendered him to the U.S. military. Hamdi (P) was transferred to a U.S. military brig in South Carolina, where he was indefinitely detained as an “enemy combatant” without formal charges or proceedings. Hamdi (P) filed a petition for a writ of habeas corpus, demonstrating that he had arrived in Afghanistan two months before the attacks on September 11, had received no military training, and was trapped there by the Northern Alliance’s military invasion. The district court found the Government’s (D) evidence insufficient to support Hamdi’s (P) detention. On appeal, the Fourth Circuit Court of Appeals reversed, concluding that the evidence, if correct, supported Hamdi’s (P) detention under a valid exercise of the President’s war powers.
    • o RULE: A citizen-detainee seeking to challenge his classification as an enemy combatant must receive notice of the factual basis for his classification and a fair opportunity to rebut the government’s factual assertions before a neutral decisionmaker.
    • o ANALYSIS: In 2001, Congress enacted the Authorization of Use of Military Force Resolution, enabling the President to use “all necessary and appropriate force” against those persons he determines to have assisted in the terrorist attacks of September 11. On its face, the Resolution authorizes Hamdi’s (P) detention. However, 18 U.S.C. § 4001(a) precludes the detention of any citizen except by an Act of Congress. But because the detention of suspected combatants falls within the meaning of “necessary and appropriate force” under the Resolution, the requirements of § 4001(a) are satisfied by an Act of Congress as an important incident of war. So long as American troops are at battle in Afghanistan, Hamdi’s (P) continued detention is lawfully authorized by the Resolution.
    • o The war on terror following the attacks of September 11 brought the debate between presidential war powers and individual civil rights to a head. It has been argued that, in times of international crisis threatening the safety of the country, the sacrifice of some individual rights for the benefit of the country as a whole is justified. The contrary view is that the value of national security is uniquely related to the individual liberties enjoyed under the law. Is the country better off as a safe police state than as a more vulnerable nation that upholds individual interests?

Suspension Clause: Writ may be suspended in times of rebellion or invasion; where right to writ is protected, Supreme Court has held that Congress may provide adequate and effective substitute to the writ if it chooses.

  • Ø Article One, Section 9, clause 2, which demands that "The privilege of the writ of habeas corpus shall not be suspended, unless when in cases of rebellion or invasion the public safety may require it."
  • Boumediene v. Bush (YEAR) [Page 00]
    • o TOPIC: Suspending habeas corpus
    • o FACTS: Boumediene (P) was one of several alien nationals designated as enemy combatants. They were detained at the U.S. Naval Station at Guantanamo Bay, Cuba. The detainees (P) petitioned for a writ of habeas corpus, and two district judges reached opposite conclusions on whether that remedy was available. While appeals were pending, Congress passed the Military Commissions Act (MCA), which provided in part that courts would have no jurisdiction to consider petitions for habeas corpus or any other lawsuits from prisoners determined to be detained as enemy combatants, or who were awaiting such a determination. The MCA was made applicable to any pending case relating to the detention of an alien detained by the U.S. (D) since September 11, 2001.
    • o RULE: Courts must provide detainees held as unlawful alien enemy combatants a writ of habeas corpus to challenge their detention, or, if a writ of habeas corpus is not available, provide an adequate substitute process to detainees that includes the same procedural protections and opportunities that would be provided in a writ of habeas corpus.
    • o ANALYSIS: Kenendy: If the privilege of habeas corpus is to be denied, Congress must act in accordance with the requirements of the Suspension Clause. The privilege—not “right”—of habeas corpus entitles a prisoner to a meaningful opportunity to demonstrate that he or she is being held unlawfully. The habeas court must also have the power to release the person whose detention is unlawful, although release need not be the only remedy available. Detention pursuant to an executive order, rather than after a criminal trial and conviction, makes the need for collateral review more pressing.
      • InJohnson v. Eisentrager,339 U.S. 763 (1950), the Court denied the writ to enemy aliens, captured outside U.S. territory, who were tried and convicted by a military tribunal for offenses committed outside the United States and who were at all times imprisoned outside the United States. In light ofEisentrager, three factors are relevant in determining the scope of the Suspension Clause with respect to detainees: (1) the citizenship and status of the detainee and the adequacy of the process that determines the status; (2) the nature of the sites where apprehension and then detention took place; and (3) the practical obstacles inherent in resolving the prisoner’s entitlement to the writ.

Military Tribunals

  • Ex Parte Quirin (1942)
    • o FACTS: During World War II, a group of German military personnel attempted to sabotage the American government by secretly landing German submarines on American shores. The saboteurs removed their uniforms and carried with them a supply of explosives, fuses, and incendiary and timing devices. All had been instructed by the German government to destroy war industries and facilities in the United States. The saboteurs were captured and held in detention for trial by military commission, which was appointed by an Executive Order of the President in 1942 to try them for violations of the law of war and the Articles of War. The saboteurs (defendants) petitioned for habeas corpus in federal district court, which was denied. The saboteurs appealed to the court of appeals but petitioned the United States Supreme Court for certiorari prior to judgment. The writ of certiorari was granted.
    • o RULE: The United States Congress and President, through the Articles of War and Executive Orders, may constitutionally place unlawful combatants on trial before a military commission for offenses against the law of war.
    • o Citizens can be tried in a military tribunal, citizens don’t have to pay …. [25:00]
    • o Seemingly allowed treating citizens and noncitizens alike


CONGRESSIONAL POWER[edit | edit source]

Majority of Congress’s powers are enumerated in Article I, Section 8.Tax and Spend

  • Ø Congress has broad powers to tax and spend for the general welfare. (Not a substitute for police powers).
  • Ø Congress may not pass regulations for the general welfare unrelated to taxation or federal spending. Further, although Congress cannot use its power to tax and spend in a way that would otherwise violate the constitution (for example, a tax levied only on people of a particular race would violate the Equal Protection Clause),
  • Ø Congress can use its power to tax to accomplish its policy objectives.
    • o US v. Kahriger: Court upheld a tax that was designed to penalize gambling.
    • o South Dakota v. Dole: Supreme Court validated a law that made federal highway finding contingent upon raising the legal drinking age to 21. This conditional spending was valid so long
      • (1) the spending was for the general welfare, (2) Congress clearly defined the condition, (3) the condition furthered "the federal interest in the national project or program," (4) the Constitution did not forbid the condition, and (5) the condition was not unduly coercive on the states.

The Civil War Amendments

  • Ø The Thirteenth Amendment abolished slavery; the Fourteenth Amendment defines citizenship and guarantees due process and equal protection for all; and the Fifteenth Amendment guarantees the right to vote regardless of color or previous enslavement. Collectively referred to as The Civil War Amendments, these amendments empower Congress to pass the legislation needed for enforcement.
  • Ø However, only the Thirteenth Amendment gives Congress the right to regulate private actors or organizations.
  • Ø The Civil Rights Cases make clear that some state action is required in order to invoke Congress’s enforcement authority under the Fourteenth and Fifteenth Amendments.

Article IV

  • Ø Article IV focuses largely on the relationships between states.
  • Ø The Full Faith and Credit Clause requires each state to recognize and give effect to the public acts of other states.
  • Ø The Privileges and Immunities Clause of Article IV forbids states from discriminating against citizens of other states.
    • o The protections of this clause are available to residents of another state that have been discriminated against, but do not extend to corporations or aliens. In order to show a denial of privileges and immunities, an individual must show that a right fundamental to national unity, such as the right to practice his or her profession, has been denied. Courts have generally used the Privileges and Immunities Clause to vindicate economic rights of out-of-state residents.
    • o For example, an Alaskan rule favoring state residents for employment on the oil pipeline was struck down under the Privileges and Immunities Clause. But, a Montana provision charging out-of-state hunters higher licensing fees was upheld. If a right is considered protected, courts will strike down acts that discriminate against citizens of other states unless it can be shown that those citizens are a “peculiar source of evil” and there is a substantial relationship between the law and the evil. This means there must be no less discriminatory means of addressing the issue.
  • Ø Section 3 sets out the rules for the entry of new states into the union and gives Congress authority over public lands and U.S. territories.
  • Ø Section 4 guarantees every state a “republican form of government” and protection from invasion and domestic violence. These are obligations of the federal government to provide security and protection for republican principles to each of the states.

Article V

  • Ø Procedures for amending the Constitution

Article VI

  • Ø The Supremacy Clause unequivocally states that the Constitution, federal laws, and U.S. treaties are “the supreme law of the land” and supersede all state and municipal laws. The Oaths Clause requires officials in all branches of government at both the state and federal levels to make an oath or affirmation that they will uphold the Constitution. Further, this clause guarantees that public officials will never have to take a religious test in order to hold office.

Article VII

  • Ø Ratification

Intro: Congress + the States

  • McCulloch v. Maryland (1819) [Page 117]
    • o Established three crucial aspects about constitutional law:
      • First, by rejecting “compact federalism,” McCulloch emphatically declares that the federal government is supreme over the states and that the states have no authority to negate federal actions.
      • Second, the Court expansively defines the scope of Congress's powers.
      • Finally, the Court limits the ability of states to interfere with federal activities, such as by imposing taxes or regulations on the federal government.
    • o FACTS: Maryland (P) attempted to impose a tax on the federal bank. The bank’s cashier, McCulloch (D) refused to pay the tax. Maryland (P) sued McCulloch (D), arguing that (1) the establishment of the bank is unconstitutional, and (2) the bank may be forced to pay state taxes.
    • o RULE: Under the Necessary and Proper Clause, Congress may enact legislation so long as its ends are legitimate under the Constitution and the legislation is appropriate and plainly adapted to those ends.
    • o ANALYSIS
      • (1) Did Congress have the authority to create the Bank of the U.S.? YES
        • Marshall’s first point: historical practice established the power of Congress to create the bank. Marshall's contention was that historical experience justifies the constitutionality of a practice.
        • Marshall's second major point was to refute the argument that states retain ultimate sovereignty because they ratified the Constitution. (compact federalism). Marshall emphatically rejected this view and contended that it was the people who ratified the Constitution, and thus the people are sovereign, not the states. “[t]he government of the Union … is, emphatically, and truly, a government of the people.”
        • Third point addressed the scope of powers. Marshall's ultimate conclusion is that Congress is not limited only to those acts specified in the Constitution; Congress may choose any means, not prohibited by the Constitution, to carry out its lawful authority. Even though the Constitution does not mention a power to create a Bank of the United States, Congress can create one as a means to carrying out many of its other powers. This is a dramatic expansion in the scope of congressional authority.
        • The fourth point concerns the meaning of the necessary and proper clause. Chief Justice Marshall said that this provision makes it clear that Congress may choose any means not prohibited by the Constitution to carry out its express authority. Marshall rejects that restrictive interpretation of necessary and proper clause. “provision is made in a constitution, intended to endure for ages to come, and consequently, to be adapted to the various crises of human affairs.” Also, the necessary and proper clause is placed in Article I, §8, which expands Congress's powers, and not in Article I, §9, which limits them.
      • (2) Can Maryland tax the Bank of the U.S.? NO
      • The Court concluded that the state may not tax the Bank of the United States because such exactions could greatly impede its operation and potentially even tax it out of existence.
      • An alternative approach would have been for the Court to allow state taxation up to the point of its interfering with or endangering the bank. But the Court rejected this argument, in part, because it did not want to embark on assessing the impact of each and every tax.
      • Also, the Court noted that a state tax on the Bank of the United States essentially was a state tax on those in other states. Those who were being taxed therefore were not represented in the state imposing the tax, and the tax was thus illegitimate.
  • National Federation of Independent Business v. Sebelius (2012) [Page]
    • o FACTS: The National Federation of Independent Business (P) brought suit against Sebelius (D), claiming that the mandate in the Patient Protection and Affordable Care Act that individuals purchase health insurance exceeded Congress’s power under the Constitution, and that the requirement that states expand their Medicaid programs or lose all federal funding was unconstitutional as well.
    • o RULE: The legitimacy of Spending Clause legislation depends on whether a state voluntarily and knowingly accepts the terms of such programs, and when Congress threatens to terminate other grants as a means of pressuring the states to accept a Spending Clause program, the legislation runs counter to this nation’s system of federalism.
    • o ANALYSIS
      • The Taxing Power': The Court upheld the individual mandate in the Patient Protection and Affordable Care Act as a valid exercise of Congress's taxing power.
        • Those defending the Court's conclusion argue that it rests on the well-established principle that Congress has broad power to tax for the general welfare and that it may tax behavior that it wants to discourage, such as going without health insurance. Those criticizing the Court's decision emphasize that the individual mandate was not adopted under the taxing power and should not be upheld on that basis.
      • The Spending Power': The Court declared unconstitutional provisions of the Patient Protection and Affordable Care Act that required that states receiving federal Medicaid funds cover within their Medicaid programs those within 133 percent of the federal poverty level.
        • Those defending the Court's conclusion argue that it rests on the well-established principle that Congress has broad power to tax for the general welfare and that it may tax behavior that it wants to discourage, such as going without health insurance. Those criticizing the Court's decision emphasize that the individual mandate was not adopted under the taxing power and should not be upheld on that basis.
      • Limits on the Commerce Clause: A limit on Congress's commerce power: It may not regulate economic inactivity.
        • Congress under the commerce clause may regulate economic activity that taken cumulatively has a substantial effect on interstate commerce. They saw the individual mandate as regulating inactivity, regulating those not engaged in commerce, and thus exceeding the scope of Congress's power.
      • The Necessary and Proper Clause: The necessary and proper clause must be used in conjunction with a valid exercise of another power of Congress.
        • “Each of our prior cases upholding laws under that Clause involved exercises of authority derivative of, and in service to, a granted power.… The individual mandate, by contrast, vests Congress with the extraordinary ability to create the necessary predicate to the exercise of an enumerated power.” In other words, since the individual mandate was not a valid exercise of Congress's commerce power, the necessary and proper clause could not be used as the basis for enacting it.

Necessary + Proper Clause

  • Ø Often called the “elastic clause,” the necessary and proper clause simply states that Congress has the power, “To make all Laws which shall be necessary and proper for carrying into Execution the foregoing Powers, and all other Powers vested by this Constitution in the Government of the United States
  • United States v. Comstock (2010) [Page 00]
    • o TOPIC: Necessary and Proper Clause – reaffirms McCulloch
    • o FACTS: A federal statute, the Adam Walsh Child Protection Act of 2006, authorized federal courts to order the indefinite confinement of individuals in the custody of the federal Bureau of Prisons who are deemed to be “sexually dangerous.” Earlier, in Kansas v. Hendricks, the Court ruled that it does not violate due process for a state to indefinitely imprison such individuals even after they have completed their prison sentences.
    • o RULE: The Constitution grants Congress the authority to enact such legislation as is necessary and proper for carrying into execution the powers vested by the Constitution in the Government of the United States.
    • o ANALYSIS: Upheld the federal law and stressed that this was permissible as an exercise of Congress's power under the necessary and proper clause. “We have since made clear that, in determining whether the Necessary and Proper Clause grants Congress the legislative authority to enact a particular federal statute, we look to see whether the statute constitutes a means that is rationally related to the implementation of a constitutionally enumerated power.”
      • The Necessary and Proper Clause grants Congress broad authority to enact federal legislation. The civil-commitment statute before us is a modest addition to a set of federal prison-related mental health statutes that have existed for decades. Congress reasonably extended its longstanding civil-commitment system to cover mentally ill and sexually dangerous persons. Moreover, the statute is reasonably adapted to Congress’s power to act as a responsible federal custodian.

The Commerce Power[edit | edit source]

  • Ø Congress may regulate commerce under Article I: “[t]o regulate Commerce with foreign Nations, and among the several States, and with the Indian Tribes.”
    • o First, Congress has the authority to make regulations regarding commerce and trade with foreign countries; this is sometimes referred to as the Foreign Commerce Clause.
    • o Second, Congress is vested with power to regulate commerce between states; this is sometimes called the Interstate Commerce Clause. (This is the part of the Commerce Clause that has been the subject of the most scrutiny, litigation, and debate over the years.)
    • o Third, Congress is granted power to regulate commerce with "Indian Tribes"; this is called the "Indian Commerce Clause.”
  • Ø The Dormant Commerce Clause or Negative Commerce Clause is the legal doctrine implicit in the Commerce Clause that forbids states from taking action that burdens or otherwise discriminates against commerce between states.
  • Ø On the other hand, the Tenth Amendment makes clear that any power not specifically delegated to the federal government is reserved for the states and the people. Particularly in the early years of Commerce Clause jurisprudence, the Supreme Court saw the Tenth Amendment as an important limit on congressional power to issue economic regulations.

Issue': The issue is whether [x]’s claim presents a non-justiciable political question.Rule': The political question doctrine excludes controversies from review that derive from policy choices and determinations that are committed for resolution to Congress or the Executive (Baker). A complaint will be a political question if it meets any one of the six elements enumerated in Baker. These factors are (1) a textually demonstrable constitutional commitment of the issue to coordinate political department; or (2) a lack of judicially discoverable and manageable standards; or (3) the impossibility of deciding without an initial policy determination of a kind clearly for nonjudicial discretion; or (4) the impossibility of a court’s undertaking independent resolution without expressing lack of the respect due coordinate branches of government; or (5) an unusual need for unquestioning adherence to a political decision already made; or (6) the potentiality of embarrassment from multifarious pronouncements by various departments on one question (Baker). A tangential relationship between the merits of a case and one of these six factors is insufficient for the Court to find a political question, although certain cases have demonstrated that first two factors to be the most important (Powell).Application

  • Federal courts won’t rule on controversy if the matter is a political question that is to be resolved by the other branches of gov’t'
    • '1) A textually demonstrable constitutional commitment of the issue to a coordinate political dept. '
      • Nixon: Impeachment trying left to Senate, as long as they don’t exceed scope of their authority this is political question'
      • Powell: Congress has constitutional ability to exclude members based on age, citizenship and residence w/o judicial review, nothing more'
    • '2) Lack of judicially discoverable and manageable standards for resolving issue'
      • Guarantee of republican gov’t'
      • War power disputes'
      • President and Legislature have alternative methods of dispute resolution w/treaties'
    • '3) Impossibility of deciding w/initial policy determination of a kind clearly for non-judicial discretion (unsuitable policy determination)'
    • '4) Impossibility of court’s undertaking indep. resolution w/o expressing lack of respect due co-ordinate branches of gov’t (Lack of respect for other branches)'
    • '5) Unusual need for unquestioning adherence to political decision already made (there has already been a political decision)'
    • '6) Potential for “embarrassment from pronouncements made by other departments on same question'
    • Baker’s 6 Political Question Test'
  • Typical issues for Political Question Doctrine to be present''''''
    • Goldwater'': no standards in Constitution governing rescission of treaties and matter was “dispute b/w coequal branches of gov’t each of which has resources available to protect and assert their interests” ' case was a political Q'''
    • Zivotofsky'': case dealt w/ examining a statute and of passport and recognition powers ' typical for court to review; not a political Q'''
    • As long as Senate functioning w/in their scope of power ' case will be political Q'''
    • Actions totally committed to executive branch'''
    • Some, but not all, foreign policy issues'''
    • Questions arising under Guaranty Clause (Luther; all cases under Guaranty Clause = PQ)'''
    • Most Questions of Impeachment (Nixon)'''
    • Qs if Constitutional amendment has been ratified (Colemen)'''
Case Name ODF Political Question?

Baker v. Carr

Plaintiffs sued for loss of meaningful vote in Tennessee

No, this dealt with a state issue, not a coordinate government branch, embarrassing policy, and there was a lot of precedent

Nixon (Senator) v. United States

Senator Nixon argued impeachment by Senate committee was violation of Art. I standard of “trial by senate”

Yes, Senate has sole power to try impeachments and decide policies

Powell v. McCormack

House refused to seat representative based on corrupt findings

No, House can’t refuse to seat a member

Goldwater v. Carter

Carter pulled a treaty with China to enter a treaty with Taiwan

Yes, this was a dispute between coequal branches and the Court did not want to be involved with partisan climate

Spending Power'''''''Issue: The issue is whether the Spending Clause give Congress the power to enact ['statute].Rule: Congress may grant conditional spending if it is for the general welfare, the conditions are unambiguous, the conditions related to the purposes of the overall spending program, recipients don’t have to violate constitutional rights to receive the funds, and the spending isn’t coercive (Dole, Butler). It is [likely/unlikely] that this statute will be constitutional under Congress’ spending power.Application': 'First: In this case, the [spending scheme] likely promotes the general welfare because [x]. Under this requirement, the Court will typically defer to Congress over whether the program promotes the general welfare. Second, the [spending scheme] is [un/likely] to meet the requirements of the clear statement rule because [x]. It could also be argued that this statute is instead ambiguous because [x]. If the Court determines the statute is unclear, then the statute will instead be read to not contain any conditions because having a clear statement isn’t a constitutional requirement. Third, these conditions are [un/likely] germane as they relate to [x]. The germaneness requirement isn’t as strict as what is required by the commerce clause which may also reinforce the relationship between the conditions and the goal of the statute. Further, this statute does not force the states to do anything illegal because [x]. Lastly, it is [un/likely] that this statute constitutes coercion to the states because the conditions resemble a [gun to the head (NFIB)/mere mild encouragement (Dole)]. This is similar to [case] where [x]. Here, the government is [doing x]. In contrast, it’s possible that this scheme is more similar to [Case] because [x].

  • Dole (germane, purpose, coercion): where the Court upheld a statute to withhold federal highway funding if the state drinking age was not increased because [x].
    • Clear Statement Rule: not a constitutional req., if Cong. doesn’t make it clear, then statute will be read as if condition is not there and the conditions won’t be required for the funding
    • Germaneness Requirement: requirement is not as strict as it is with the Commerce Clause
    • Condition doesn’t require the recipients to violate any constitutional provisions
    • Court doesn’t provide a guideline for what constitutes as coercive; “mild encouragement” is appropriate, however
      • Temptation: motivating states is not coercive (Dole)
      • Gun to the head: Dole considered the 5% to be okay, but the state’s budgets in NFIB was a gun to the head with the amount.
    • (1) Spending program must promote the general welfare
    • (2) Any condition on state’s receipt of federal funds must be unambiguous must be clear statement rule
    • (3) The spending condition must be related to the purposes of the overall spending program
    • (4) The condition does not require the recipients to violate any individual rights
    • (5) The spending cannot be coercive
  • Butler: The Court will invalidate a law enacted with under the spending clause if the spending scheme is coercive, unless Congress can validate the law under another enumerated power (agricultural adjustment act invalid). Shows coercion.
  • NFIB: The condition for spending must relatively mild encouragement similar to Dole; whereas in NFIB the spending condition was a gun to the head if the states did not comply.
  • Steward: Distinguish Butler (1) NOT earmarked for special group; (2) States approval; (3) NOT an irrevocable agreement; (4) Directed at lawful ends.

'Conclusion': Therefore, Congress [will likely/is unlikely to be able to] enact [x] under their Spending power.

Case Name ODF Outcome

United States v. Butler

Statute to stabilize farm prices by pay farmers not to use land

Unconstitutional

Steward Machine Co. v. Davis

Statute requiring unemployment insurance for the states

Constitutional

Helvering v. Davis

Statute enacting old age provision of SSA

Constitutional

South Dakota v. Dole

National Minimum Age Drinking Act, Secretary of Transportation would withhold 5% highway funding for not having 21+

Constitutional

NFIB

Medicaid Expansion, 20% of 50-80% of the state budget (gun to the head)

Unconstitutional

'Treaty Power'''''''Issue: The issue here is whether Congress may enact [statute] under their Treaty Power.Rule: Under the treaty power, the President has the power to enter into treaties with the advice and consent of the Senate (Art. II), and Congress will not be limited to their enumerated powers in passing legislature provided it doesn’t violate any prohibitory provisions of the Constitution (Reid). When the treaty in question is non-self-executing, then Congress’ legislature may effectuate the treaty’s obligations under the necessary and proper clause, provided the law is reasonably related to the end that is itself within power (McCulloch), but with wide latitude given (Kebadeaux). Here, it is [un/likely] that this law will be constitutional under Congress’ treaty power.Analysis': 'First: Because we do not have the facts necessary to determine that this treaty was entered into in a valid manner, there will be a cloud of uncertainty in regards to the ultimate validity of the legislature. However, in the event that this treaty was enacted appropriately, then it’s necessary to establish whether the treaty contradicts prohibitory constitutional provisions (Reid, Missouri). Here, the law in question seems as though it [does/does not] violate prohibitory provisions [if it does, explain which provision and why]. Further, it [does/does not] appear that the law here is clearly appropriate and adapted to enforcing the terms of this treaty (Kebadeaux).

  • Prohibitory provisions: It doesn’t violate the Constitution
    • The only areas really off limits are when the statute is against express prohibitive clauses of the Constitution
    • Congress will still have the power to enact if it isn’t listed under their enumerated powers
    • Self-Executing Treaty: requires no addt’l fed legislation to become enforceable in domestic courts
    • Non-Self-Executive Treaty: requires addt’l legislation to become valid in domestic court

Conclusion': Therefore, it is [un/likely] that Congress will be able to pass this law under the treaty power.Necessary and Proper Clause'''''''Issue: Does the Necessary and Proper Clause allow Congress to enact [statute]?Rule: Congress may use the Necessary and Proper Clause to enact [statute] if it the statute “hooks” on to another power that Congress is able to exercise.

  • Necessary:
    • Comstock'': Congress may prescribe sanctions for crimes that they created,
    • NFIB'': Must be used in conjunction with valid exercise of another power of Congress; Cong can’t just create the necessity for a certain statute
    • Is a means appropriate and adapted to a legitimate end;
    • Hooks on to another valid power of Congress (not necessarily powers that are enumerated – Comstock);
  • Proper:
    • Doesn’t make constitution redundant
    • Is consistent with the letter and spirit of the Constitution
Case Name ODF Outcome

McCulloch v. Maryland

United States made a second bank, Maryland tried to tax it

Constitutional, the means were appropriate and adapted to the ends of an enumerated power

United States v. Comstock

Granted district courts the ability to order civil commitment of mentally ill, sexually dangerous federal prisoners past their release dates

Constitutional, look to the 5 factors provided for necessary and is proper

NFIB v. Sebellius

OBAMACARE

No, it does not latch on to an enumerated power and it undermined the structure of the government and tramples individual autonomy

10th Amendment Constraint on Federal Legislative Power'''''''Issue': The issue is whether Congress has the power to regulate [state or local gov’t] in pursuant to the Tenth Amendment.Rule': Under the Tenth Amendment, any power not delegated to the federal government is retained by the states. So, Congress may not place an affirmative duty on states to regulate in a particular way unless the law is one that is generally applicable (Garcia). It is [likely/unlikely] that Congress will be able to do [x].Application': 'First (unconst’al): Here, this act seems to be placing an affirmative duty to [state] in order to [x]. The type of duty that Congress is imposing on [x] is similar to [New York/Printz] because [x]. This infringes on state sovereignty as Congress is essentially taking control of state legislatures and commandeering them to [x] to carry out a federal scheme (New York). In contrast, it could be said that this legislature instead prohibits conduct as opposed to placing any affirmative duties on [x] because [x].Second: Further, unlike the Fair Labor Standards Act in Garcia that was equally applicable to both state and private entities, this act here only seems to apply to the states because [x].First (const’al): Here, this act does not seem to place any duty on [x]. Unless it is shown that Congress is commandeering the legislature to [x] to carry out a federal scheme, it is unlikely that there is a violation of the state’s sovereignty (New York). Further, like the Fair Labor Standards Act in Garcia that applied to both state and private entities, this act functions similarly because [x].Second: Further, like the statute in [Garcia/Reno], this statute seems to be generally applicable because [x].Garcia: Like the Fair Labor Standards Act in Garcia that could apply to both state and private entities, this act here will likely be constitutional for because [x]. '''New York''': The law will be held unconstitutional if the federal government is attempting to use the state political subdivisions and is commandeering states to enforce a federal regulatory program. The federal government can shift the blame of a federal mistake to the statesPrintz''': federal law was found unconstitutional because it was commandeering state executive officials (gun background checks). Commandeering local law enforcement officers.Reno''': law for personal date into motor vehicle department database constitutional because federal government can regulate commercial activity that is generally applicable (Reno).Conclusion': Therefore, Congress [did/did not] have the power to regulate state governments to [x] under the commandeering doctrine of the Tenth Amendment.

Case Name Case Explanation Constitutional?

National League of Cities v. Usery

Fair Labor Standards Act was amended to remove exemption to employees of state institutions.

Unconstitutional. Federal government does not have the power to force states to give up traditional government functions.

Garcia v. San Antonio Metro. Transit Authority

US Dept of Labor asserted that the FLSA covered the Transit authority’s operations

Constitutional. Overruled Usery. The “Traditional Government Functions” test was unworkable.

New York v. United States

Federal law required state legislatures to enact laws or take possession and liability for nuclear waste

Unconstitutional. Commandeering of state government and legislature.No accountability.

Printz v. United States

Federal law required state police officers to conduct background checks for guns. Unconstitutional. Pushes costs on to states. Against original intent of the constitution.

Reno v. Condon

Federal law prohibited state DMVs from giving out personal information without consent of individuals.

Constitution. Statute is generally applicable to all institutions with such information. Not an affirmative duty, but a negative prohibition.

What Can Congress Regulate?

  • Ø Based on the case law, we can generally say that any activity that falls under one of three broad headings may be constitutionally regulated by Congress under the Commerce Clause:
  • Ø (1) The channels of commerce,
    • o Channels of Interstate Commerce allow for the movement of goods, people, or communications in interstate commerce; channels include railroads, highways and roadways, waterways, airspace, phone lines, the internet, and the like.
  • Ø (2) The instrumentalities of commerce, and
    • o The line between a channel and an instrumentality of interstate commerce is somewhat blurry, but instrumentalities include trains, vehicles, planes, boats, and anything else used to carry out interstate commerce.
  • Ø (3) Anything that has a substantial effect on commerce.
    • o The substantial effects doctrine is the biggest and most complicated of the commerce clause theories.
    • o In U.S. v. Lopez, the Supreme Court held that the connection between gun possession on school property to commerce was not strong enough for a federal law barring firearms in schools to be justified by the Commerce Clause.
    • o Based on that case, we have to consider a number of factors to help us decide whether an activity has a substantial effect on interstate commerce.
      • First, we must ask whether the activity is commercial in nature. If the activity is commercial, then the entire category of activities that it falls into may be considered collectively to determine whether there is, in fact, a substantial effect on interstate commerce.
        • If yes, then may regulate activity category
        • May regulate noneconomic parts of the economic activity
      • Does it involve intrastate commerce?
      • Does it have noneconomic aspects?
        • May regulate if collective substantial effect
        • If entirely noneconomic activity, there probably needs to be a fairly obvious connection to interstate commerce for that regulation to be considered a valid exercise of commerce authority.
        • But in all such cases involving regulation of noneconomic activity, a court will show much less deference to Congress's determination that such a substantial effect exists.
      • Court will independently determine if there is a substantial effect on commerce
      • Does the activity being regulated fall under the police power of the state?
        • May be overcome if there is a clear need for a national solution.
      • Congress can also regulation articles moving in commerce

There have been roughly four eras of Commerce Clause jurisprudence.

  • v From early in American history until the 1890s, the commerce power was broadly defined but minimally used.
  • v From the 1890s until 1937, the Court narrowly defined the scope of Congress’s commerce power and used the Tenth Amendment as a limit. The era
  • v From 1937 until the 1990s was a time when the Court expansively defined the scope of the commerce power and refused to apply the Tenth Amendment as a limit.
  • v Since the 1990s, the Court has again narrowed the scope of the commerce power and revived the Tenth Amendment as an independent, judicially enforceable limit on federal actions.

Throughout these eras, the Court is considering three questions.

  • v First: What is “commerce”? Is it one stage of business, or does it include all aspects of business—even life in the United States?
  • v Second: What does “among the several states” mean? Is it limited to instances where there is a direct effect on interstate commerce, or does any effect on interstate activities suffice?
  • v Third: Does the Tenth Amendment limit Congress? If Congress is acting within the scope of its commerce power, can a law be declared unconstitutional as violating the Tenth Amendment?

1). Initial Era

  • Gibbons v. Ogden (1824) [Page 157]
    • o TOPIC: Supreme Court interprets federal commerce power broadly.
    • o FACTS: Ogden (D) operated a ferry boat service between New York and New Jersey under an exclusive license granted by the New York legislature. Gibbons (P), holding a similar license granted by Congress, began competing with Ogden (D). Ogden (D) sought and received an injunction against Gibbons’ (P) competing ferry service. Gibbons (P) appeals to the Supreme Court.
    • o RULE: The federal commerce power extends to all commerce among and between the states and foreign nations, with only commerce having connections solely within a single state being unreachable under the commerce power.
    • o ANALYSIS: The New York monopoly was an impermissible restriction of interstate commerce. “All America understands, and has uniformly understood, the word “commerce,” to comprehend navigation.” According to Gibbons, commerce includes all phases of business, including navigation.
      • “The subject to which the power is next applied, is to commerce “among the several States.” The word “among” means intermingled with. A thing which is among others, is intermingled with them. Commerce among the States, cannot stop at the external boundary line of each State, but may be introduced into the interior.”
    • o The Commerce Clause has two general purposes. First, it provides Congress with the authority to affirmatively regulate all commerce that is not exclusively founded and finished within the borders of a single state. Second, the commerce clause, without any affirmative declarations by Congress, acts as a limit on the exercise of state power in the interstate commerce arena. This is often called the dormant commerce clause. Both of these aspects are present inGibbons v. Ogden.Though Chief Justice Marshall does not expressly state that Congress has the authority to license ferryboat operators in interstate commerce, this conclusion is surely implied by stating that Congress has plenary power in this arena. Possibly more importantly, the Chief Justice holds that by granting Ogden (D) an exclusive license to ferry passengers and cargo between New York and New Jersey, New York Interferes with or burdens interstate commerce.Gibbonsis one of the very first steps in giving “commerce” a broad definition, a trend that continues long after this decision.
      • Congress has complete authority to regulate all commerce among the states. When acting under Commerce Clause authority, Congress can regulate in the same way it could if no state governments existed. The Court said that the sole check on Congress is the political process, not judicially enforced limits to protect states.

The Daniel Ball

  • v The Court accorded Congress broad authority to license ships, even those operating entirely intrastate, as long as the boats were carrying goods that had come from another state or that ultimately would go to another state. The Court explained that unsafe ships in intrastate commerce could affect and harm ships in interstate commerce.

United States v. Dewitt

  • v A federal law outlawed the sale of naphtha and other illuminating oils that could ignite at less than 110 degrees Fahrenheit. The Court held that the law was “a police regulation, relating exclusively to the internal trade of the States.” In a precursor to decisions that followed after the 1890s, the Court declared the law unconstitutional because it was “a virtual denial of any power to interfere with the internal trade and business of the separate States.”

The Trademark Cases

  • v The Court invalidated the federal law that established a federal system for registering trademarks. The Court concluded that the law was unconstitutional because it applied to wholly intrastate businesses and business transactions and therefore “is obviously the exercise of a power not confided to Congress.”

Congress rarely exercised its commerce authority in the years that followed Gibbons v. Ogden, and it wasn't until the Interstate Commerce Act of 1887 and the Sherman Antitrust Act of 1890 that we see a significant uptick in Supreme Court scrutiny of the commerce power.2). 1890s-1937: Limited Federal Commerce PowerThis era of constitutional law is extremely important. It was the first time that the Supreme Court aggressively used its power of judicial review to invalidate federal and state laws. Constitutional law since 1937 has very much been a reaction to this earlier era.Between the late nineteenth century and 1937, the Court espoused a philosophy often termed “dual federalism.”

  • v Dual federalism was the view that the federal and state governments were separate sovereigns, that each had separate zones of authority, and that it was the judicial role to protect the states by interpreting and enforcing the Constitution to protect the zone of activities reserved to the states.
  • v Dual federalism was embodied in three important doctrines that the Court developed and followed during this period.
    • o First, the Court narrowly defined the meaning of commerce so as to leave a zone of power to the states.
    • o Second, the Court restrictively defined among the states as allowing Congress to regulate only when there was a substantial effect on interstate commerce.
    • o Finally, the Court held that the Tenth Amendment reserved a zone of activities to the states and that even federal laws within the scope of the commerce clause were unconstitutional if they invaded that zone.
  • v However, it should be noted at the outset that the Court was not completely consistent in applying these principles. The Court was most likely to follow them when considering federal economic regulations; the Court was least likely to adhere to them, and most willing to uphold federal laws, when they concerned federal morals regulation.
  1. What is “Commerce?”
  • v The Court held that commerce was to be narrowly defined as one stage of business, separate and distinct from earlier phases such as mining, manufacturing, and production
  • v United States v. E.C. Knight
    • o HOLDING: The Sherman Antitrust Act could not be used to stop a monopoly in the sugar refining industry because the Constitution did not allow Congress to regulate manufacturing.
    • o FACTS: The U.S. government attempted to use the Sherman Antitrust Act to block the American Sugar Refining Company from acquiring four competing refineries. The acquisition would have given the company control of over 98 percent of the sugar refining industry.
    • o The Court held that federal law could not be applied because the monopoly was in the production of sugar, not in its commerce.
      • The Court was clear that this rigid distinction was based on a need for preserving a zone of activities to the states.
        • The Court explained that although the commerce power was one of the “strongest bond[s] of the union, . . . the preservation of the autonomy of the States [w]as required by our dual form of government.”
      • This distinction between manufacturing and commerce seems arbitrary; a company would desire a monopoly in production because it would benefit from monopoly profits in commerce. The Court acknowledged this but said that the relationship was too indirect to allow federal regulation under the commerce power. The Court said that it would be “far-reaching” to allow Congress to act “whenever interstate or international commerce may be ultimately affected.”
    • v Carter v. Carter Coal Co.
      • o The Court declared unconstitutional the Bituminous Coal Conservation Act of 1935. The law contained detailed findings as to the relationship between coal and the national economy and declared that the production of coal directly affected interstate commerce. The law provided for local coal boards to be established to determine prices for coal and to determine, after collective bargaining by unions and employers, wages and hours for employees.
      • o “[C]ommerce is the equivalent of the phrase “intercourse for the purposes of trade.” Plainly, the incidents leading up to and culminating in the mining of coal do not constitute such intercourse. The employment of men, the fixing of their wages, hours of labor and working conditions, the bargaining in respect of these things—whether carried on separately or collectively—each and all constitute intercourse for the purposes of production, not of trade.”

'

  1. What Does “Among the States” Mean?

The Court interpreted “among the states” as requiring a direct effect on interstate commerce. Yet the Court never formulated a clear or consistent way to distinguish direct from indirect effects. The stream of commerce approach was sometimes used during this era to evaluate whether an activity was among the states. Yet the Court was no more consistent in applying this test.

  • v Shreveport Rate Cases
    • o The Court upheld the ability of the Interstate Commerce Commission to set intrastate railroad rates because of their direct impact on interstate commerce. Specifically, a railroad was ordered to charge the same rates for shipments to Marshall, Texas, whether from Shreveport, Louisiana, or from Dallas, Texas.
    • o “Congress in the exercise of its paramount power may prevent the common instrumentalities of interstate and intrastate commercial intercourse from being used in their intrastate operations to the injury of interstate commerce . . . [Congress] does possess the power to foster and protect interstate commerce, and to take all measures necessary or appropriate to that end, although intrastate transactions of interstate carriers may thereby be controlled.”
  • v L.A. Schecter Poultry Corp. v. United States
    • o The National Industrial Recovery Act (New Deal legislation) authorized the president to approve “codes of fair competition” developed by boards of various industries. The president approved a Live Poultry Code for New York City. In part, the code was designed to ensure quality poultry by preventing sellers from requiring buyers to purchase the entire coop of chickens, including sick ones. The code also regulated employment by requiring collective bargaining, prohibiting child labor, and establishing a 40-hour workweek and a minimum wage.
    • o SCOTUS declared the code unconstitutional because there was not a sufficiently “direct” relationship to interstate commerce. Although the Court acknowledged that virtually all the poultry in New York was shipped from other states, the Court said that the code was not regulating the interstate transactions; rather, the code concerned the operation of businesses within New York. The Court emphasized that Congress only could regulate when there was a direct effect on interstate commerce.
    • o “In determining how far the federal government may go in controlling intrastate transactions upon the ground that they ‘affect’ interstate commerce, there is a necessary and well-established distinction between direct and indirect effects . . . where the effect of intrastate transactions upon interstate commerce is merely indirect, such transactions remain within the domain of state power.”
      • Enforcing the distinction between direct and indirect effects on commerce “must be recognized as . . . essential to the maintenance of our constitutional system.”

The Supreme Court soon adopted another theory of congressional authority under the Commerce Clause: stream of commerce.

  • Ø In its landmark 1905 decision, the Court in Swift & Co. v. United States held that Congress had the authority to enact legislation regarding seemingly intrastate monopolies that were part of the stream or current of commerce.
    • o The fact that goods had once been transported across state lines was enough to put an item under Congress's authority to regulate interstate commerce.
  • Ø Likewise, the Supreme Court generally upheld congressional regulations on activities that had "substantial economic effects" on interstate commerce. In the Shreveport Rate Case, the Supreme Court upheld Congress's authority to regulate a wholly intrastate railway line because there was a close, substantial relationship to interstate commerce.
  • v Swift & Co. v. United States
    • o In light of the ambiguous direct and indirect effects standard, the Court allowed Congress to regulate to protect the stream of commerce
    • o Upheld the application of the Sherman Antitrust Act to an agreement among meat dealers to fix the price at which they would purchase meat from stockyards. Although the stockyard was intrastate, the Court stressed how it was only a temporary stop for the cattle. Justice Holmes, writing for the Court, explained that the stockyards were in “a current of commerce among the States, and the purchase of the cattle is a part and incident of such commerce.”
  • v Stafford v. Wallace
    • o Upheld the Packers and Stockyards Act of 1921, which authorized the secretary of commerce to regulate rates and prescribe standards for the operation of stockyards where livestock was kept. The law was designed to protect consumers by lessening collusion between stockyard managers and packers and by decreasing the ability of packers to set prices for livestock. SCOTUS emphasized that the stockyards are in the stream of commerce.
      • “stockyards are but a throat through which the current flows, and the transactions which occur therein are only incident to this current from the West to the East, and from one State to another. Such transactions can not be separated from the movement to which they contribute and necessarily take on its character.”
    • v Railroad Retirement Board v. Alton R.R. Co.
      • o The Court declared unconstitutional the Railroad Retirement Act of 1934, which provided a pension system for railroad workers. Railroads obviously were part of the stream of interstate commerce, and the Court had upheld other federal regulations of railroads. '
        • In Southern Railway v. United States, the Court upheld the Federal Safety Appliance Acts, which regulated couplers on railroad cars. In Baltimore & Ohio Railroad Co. v. Interstate Commerce Commission, the Court upheld a federal law that set maximum hours for railroad workers.'
      • o Here, the Court struck down the requirement for a pension for railroad workers and distinguished the other cases as concerning the safety or efficiency of the railroads. The Court said that Congress could not use its commerce power to require a pension program for railroad employees because the law was only to help “the social welfare of the worker, and therefore [was] remote from any regulation of commerce.”
  1. Does State Sovereignty Limit Congressional Power?

The Court held that even if an activity was commerce and was among the states, Congress still could not regulate if it was intruding into the zone of activities reserved to the states. The Court concluded that the Tenth Amendment reserved control of activities such as mining, manufacturing, and production to the states. Even federal laws regulating commerce among the states were unconstitutional if they sought to control mining, manufacturing, and production. But, the Court did not consistently define the zone of activities reserved to the states.

The Child Labor Case[edit | edit source]

  • v (Hammer v. Dagenhart)
    • o A federal law prohibited the shipment in interstate commerce of goods produced in factories that employed children under age 14 or employed children between the ages of 14 and 16 for more than eight hours per day or six days a week.
    • o Although the law only regulated goods in interstate commerce, the Court declared it unconstitutional because it controlled production.
    • o The decision was close, five-to-four, but ultimately, the Court concluded the harm Congress was trying to remedy was only in the manufacturing of the goods and not the goods themselves, and therefore the regulation was unconstitutional.
      • “[t]he grant of power to Congress over the subject of interstate commerce was to enable it to regulate such commerce, and not to give it authority to control the States in their exercise of the police power over local trade and manufacture.”
      • The Court said that regulating the hours of labor of children was entrusted “purely [to] state authority.”
      • The Court expressly rejected the argument that federal legislation was necessary to prevent unfair competition; states that wanted to outlaw child labor would find it difficult to do so as long as other states allowed child labor.
      • “if Congress can thus regulate matters entrusted to local authority by prohibition of the movement of commodities in interstate commerce, all freedom of commerce will be at an end, and the power of the States over local matters may be eliminated, and thus our system of government be practically destroyed.”
    • v The Lottery Case (Champion v. Ames)
      • o Upheld a federal law prohibiting the interstate shipment of lottery tickets.
      • o In both The Child Labor Case and The Lottery Case, the federal law prohibited the shipment of a specified item—goods made by child labor or lottery tickets—in interstate commerce. In both, Congress obviously was seeking to stop intrastate activities: the use of child labor and gambling in lotteries. Yet in the former the Court declared the federal law unconstitutional, whereas in the latter the Court upheld the federal law.
      • o The Court made it clear that the power to regulate interstate commerce includes the ability to prohibit items from being in interstate commerce. It was within Congress’s Commerce Clause power to stop lottery tickets from being a part of interstate commerce.
        • “If a State, when considering legislation for the suppression of lotteries within its own limits, may properly take into view the evils that inhere in the raising of money, in that mode, why may not Congress, invested with power to regulate commerce among the several States, provide that such commerce shall not be polluted by the carrying of lottery tickets from one State to another?”
      • o The Court rejected the argument that according Congress such power would give Congress seemingly limitless authority and would endanger the constitutional structure.
        • “[T]he possible abuse of a power is not an argument against its existence.”

3). 1937-1990s: Broad Federal Commerce PowerThese three cases adopt broad definitions of “commerce” and “among the states” and reject the Tenth Amendment as a limit on Congress’s Commerce Clause power.

  • v Commerce includes all stages of business; no longer is a distinction drawn between commerce and other stages of business such as mining, manufacture, and production.
  • v Congress can regulate any activity, intrastate or interstate, that has a substantial effect on interstate commerce.
  • v Congress can regulate activities that themselves have little effect on interstate commerce if the activity, looked at cumulatively throughout the country, has a substantial effect on commerce.
  • v The Tenth Amendment, as Darby states, is simply a reminder that for Congress to legislate it must point to express or implied power. The Tenth Amendment is no longer seen as reserving a zone of activities for exclusive state control.
  • NLRB v. Jones (1937) [Page 168]
    • o FACTS: In 1935, Congress passed the National Labor Relations Act which created the National Labor Relations Board (NLRB) to enforce federal fair labor practice standards, including the right of employees to unionize. After Jones & Laughlin Steel Corp. fired ten employees that attempted to unionize at one of its Pennsylvania plants, the NLRB sanctioned the company for engaging in discriminatory employment practices in violation of federal standards. JLSC sued alleging that the Act was an unconstitutional exercise of Congress’s interstate commerce power, and the lower courts agreed. The NLRB appealed to SCOTUS.
    • o RULE: Congressional power to regulate interstate commerce extends to the regulation of intrastate activities that may burden or obstruct interstate commerce.
    • o ANALYSIS: Jones & Laughlin Steel Corp. was clearly a part of interstate commerce because of its expansive nationwide operations and it was the fourth largest producer of steel with factories in Pennsylvania. The steel business was part of the stream of commerce and labor relations within it had a direct effect on commerce.
    • o The Act is challenged in its entirety as an attempt to regulate all industry, thus invading the reserved powers of the States over their local concerns. We think it clear that the Act may be construed so as to operate within the sphere of constitutional authority. It covers interstate and foreign commerce, subjects left exclusively to the federal government.
    • o However, congressional authority to protect interstate commerce is not limited to transactions deemed to be an essential part of the “flow” of interstate commerce. Burdens and obstructions may come from many sources. Congress’ authority to deal with such burdens is plenary, and reaches to the source of the burden wherever it may lie. Thus, although the activities may be intrastate in character when separately considered, if they have such a close and substantial relationship to interstate commerce that their control is essential or appropriate to protect that commerce, Congress cannot be denied the power to exercise that control. In other words, the fact that the employees here concerned were engaged in production is not determinative. The question remains as to the effect upon interstate commerce of the labor practice involved. Given the breadth of J&L’s (D) operations, a stoppage of those operations by industrial strife would have a most serious effect upon interstate commerce.

InN.L.R.B. v. Friedman-Harry Marks Clothing Co.,the Court upheld the application of the NLRA to even a relatively small clothing manufacturer that shipped clothing in interstate commerce. The Court noted that a strike in the New York clothing industry would have a severe effect on interstate commerce. This shows that an economic effect on interstate commerce, even if slight, gives Congress authority under the Commerce clause to regulate the activity.

  • U.S. v Darby (1941) [Page 171]
    • o FACTS: Congress passed the Fair Labor Standards Act to prevent the introduction and shipment of goods produced under labor conditions that failed to meet federal standards from entering the stream of interstate commerce. Through the Act, Congress seeks to prevent the production of goods under conditions harmful to the maintenance of the minimum standards of living necessary for health and general well-being; and to prevent the use of interstate commerce as a means of competition in the distribution of goods so produced, and as the means of spreading and perpetuating such substandard labor conditions in the several states (i.e. Congress is exercising police powers). Darby (D), a Georgia lumber producer shipping lumber in interstate commerce, was indicted for violating the wage and hour standards of the Act. The district court quashed the indictment of Darby, and the court of appeals affirmed. The U.S. government appealed to SCOTUS.
    • o RULE: Congress has the authority, under the Commerce Clause, to exclude any article from interstate commerce, in judgment that they are injurious to the public health, morals or welfare.
    • o ANALYSIS: “Congress, having by the present Act adopted the policy of excluding from interstate commerce all goods produced for the commerce which do not conform to the specified labor standards, it may choose the means reasonably adapted to the attainment of the permitted end, even though they involve control of intrastate activities.”
      • “Whatever their motive and purpose, regulations of commerce which do not infringe some constitutional prohibition are within the plenary power conferred on Congress by the Commerce Clause.” It is not a valid objection to such an assertion of congressional power to regulate commerce that its exercise is attended by the same incidents which attend the exercise of the police powers of the states.
      • “The power of Congress over interstate commerce is not confined to the regulation of commerce among the states. It extends to those activities intrastate which so affect interstate commerce or the exercise of the power of Congress over it as to make regulation of them appropriate means to the attainment of a legitimate end, the exercise of the granted power of Congress to regulate interstate commerce.”
      • The [Tenth] amendment states but a truism that all is retained which has not been surrendered.” In other words, a law is constitutional so long as it is within the scope of Congress's power; the Tenth Amendment would not be used by the judiciary as a basis for invalidating federal laws.
    • o The Court’s conclusion inHammer v. Dagenhart, that only those articles that are harmful or deleterious in and of themselves may be banned from interstate commerce, was not supported by any provision in the Constitution. It was novel then and has not been followed since. Overruled.

Remember the Court’s ruling inHammer v. Dagenhart, that Congress did not have the power, under the Commerce Clause, to prohibit child labor through a general ban on the shipment of its fruits. WasHammertruly a ruling without support in prior and subsequent case law? In theLottery Casethe Court ruled that only those articles adjudged to be deleterious in and of themselves could be banned from interstate commerce. InSchechter Poultry(the Sick Chicken Case), the Court ruled that the commerce power did not extend past the initial sale once an article entered a state. InE.C. Knightthe Court ruled that Congress could not regulate manufacturing because it is a local concern. The holding inCarter v. Carter Coalis directly on point forDarby.The Court held therein that Congress could not set wages and working conditions for miners, yet, afterDarby, it has power to do so.

  • Wickard v. Filburn (1942) [Page 173]
    • o FACTS: During the Great Depression, FDR and the Democratic-controlled Congress passed many “New Deal” programs designed to improve the poor economic climate. The Agricultural Adjustment Act of 1938 limited the area that farmers could devote to wheat production in an effort to stabilize the national price of wheat. Filburn, a small farmer, was penalized pursuant to the Act for producing wheat in excess of the Act's quotas. Filburn sued Secretary of Agriculture Wickard, seeking to enjoin enforcement against himself of the penalties. Filburn argued that because the excess wheat was produced for his own private consumption and never entered the stream of commerce, his activities could not be regulated by Congress under the Commerce Clause. The district court agreed with Filburn and the circuit court affirmed. Wickard appealed to SCOTUS.
    • o RULE: Congress’ commerce authority extends to all activities having a substantial effect on interstates commerce, including those that do not have such a substantial effect individually, but do when judged by their national aggregate effects.
    • o ANALYSIS: Even if an activity is local in nature, and therefore not regarded as commerce, it may still be reached by Congress if it exerts a substantial economic effect on interstate commerce, whether such an effect be “indirect” or “direct.”
      • “That appellee’s own contribution to the demand for wheat may be trivial by itself is not enough to remove him from the scope of federal regulation where, as here, his contribution, taken together with that of many others similarly situated, is far from trivial.”
      • It is clear afterWickard, if it wasn’t clear afterDarby, that the Tenth Amendment is ineffective as a check on the federal commerce power.Wickardalso set a new, broader standard for the exercise of the commerce power. How difficult is it to come up with an intrastate activity that doesn’t, in some way, substantially affect interstate commerce when viewed in the national aggregate? As demonstrated in the 1960s civil rights cases, Congress has taken advantage of this vast broadening of its commerce authority. AfterWickard, the only significant checks on the commerce power were specific guarantees, such as those in the Bill of Rights.

The Meaning of “commerce among the states”[edit | edit source]

Civil Rights Laws

  • v The Civil Rights Act of 1964 prohibits private employment discrimination based on race, gender, or religion, and forbids racial discrimination by places of public accommodation such as hotels and restaurants. Congress enacted this legislation under its Commerce Clause power.
  • v SCOTUS, in 1883, had held that Congress only could regulate government conduct and could not regulate private behavior under the Fourteenth Amendments. Therefore, in 1964, it was uncertain whether Congress could use its Fourteenth Amendment power to outlaw private discrimination in employment and public accommodations. Congress thus chose the commerce clause as the authority for this landmark legislation.
  • Heart of Atlanta Motel, Inc. v. US (1964) [Page 176]
    • o TOPIC:
    • o FACTS: Title II of the Civil Rights Act of 1964 forbids racial discrimination by places of public accommodation such as hotels and restaurants. The Heart of Atlanta Motel, Inc. advertises to and hosts primarily out-of-state guests. The motel practices a policy of refusing to rent rooms to African Americans and sued the U.S. government to challenge the Act as an unconstitutional extension of Congress’s power to regulate interstate commerce. The district court upheld the Act as constitutional. The court of appeals affirmed. Heart of Atlanta appealed to SCOTUS.
    • o RULE: Congress has the power, under the Commerce Clause, to regulate local activities that could reasonably be seen as exerting a substantial and harmful effect upon interstate commerce.
    • o ANALYSIS: The record of the Act’s passage in each house is replete with evidence of the burdens that racial discrimination places upon interstate commerce: that our people have become increasingly mobile, with millions of people traveling from state to state; that Negroes in particular have been the subject of discrimination in transient accommodations; that these conditions had become so acute as to require the listing of available lodging for Negroes in a special guidebook. These exclusionary practices were found to be nationwide. The testimony indicated a qualitative as well as quantitative effect on interstate travel by Negroes.
      • Here, Congress was dealing with a moral problem. But this does not detract from the overwhelming evidence of the harmful effects of racial discrimination upon interstate commerce. It was this burden which empowered Congress to enact appropriate legislation.
      • Douglas Concurrence: Reluctant to rely solely on the Commerce Clause. The result reached by the Court is much more obvious when the facts are considered under the Fourteenth Amendment, for this Amendment deals with the constitutional status of the individual, not with the impact on commerce of local activities or vice versa.
      • The Act’s constitutionality here flows from the fact that an inability to rely on finding accommodations while traveling from state to state is a substantial obstruction to the undertaking of such travel. When a significant percentage of the population is much less likely to travel interstate, even the purely economic ramifications are easy to recognize.

Katzenbach v. McClung[edit | edit source]

(1964) [Page 178]

    • o FACTS: The McClung’s (Ollie’s)(D) own Ollie’s Barbecue, a restaurant located in Birmingham, Alabama, near both a state and an interstate highway. Ollie’s (D) caters to a family and white-collar trade, with a takeout service for Negroes. Ollie’s (D) purchases about $150,000 worth of food, 46% of which is meat purchased form a local supplier who procures it from out of state. The trial court below found that a substantial portion of the food Ollie’s (D) sells has moved in interstate commerce. Ollie’s (D) continued its racially discriminatory practices even after the Civil Rights Act of 1964 (Act), which bans such practices in public accommodations, went into effect on July 2, 1964.
    • o RULE: Congress’ commerce authority extends to any public commercial establishment selling goods that have moved in interstate commerce and/or serving interstate travelers.
    • o ANALYSIS: The Court's decision was not based on the interstate impact of this particular restaurant. Rather, the Court found that Congress rationally had concluded that discrimination by restaurants cumulatively had an impact on interstate commerce. The Court is saying that the existence of racially exclusive restaurants in a particular locale end up “rubbing off” on all of the other, non-discriminatory, restaurants in the area. Thus, a black traveler, knowing of one or two discriminatory restaurants in a town, will fail to stop in that town. This results in a loss of potential business for all restaurants in the town. When the aggregation principle is applied to such a situation, the sum total of the loss of interstate business in all such towns and cities most definitely reaches the “substantial effect on interstate commerce” standard.
    • o The Court upheld the Civil Rights Act and its application to Ollie's Barbecue because “[t]he power of Congress [under the commerce clause] is broad and sweeping.”

Regulatory Laws [edit | edit source]

  • Hodel v. Indiana [Page 180]
  • v Upheld a federal law that regulated strip mining and required reclamation of strip-mined land.
  • v “A court may invalidate legislation enacted under the Commerce Clause only if it is clear that there is no rational basis for a congressional finding that the regulated activity affects interstate commerce, or that there is no reasonable connection between the regulatory means selected and the asserted ends.”
  • v Under this test, it is difficult to imagine anything that Congress could not regulate under the commerce clause so long as it was not violating another constitutional provision.

Criminal Laws

  • Perez v. US [Page 181]
    • o TOPIC:
    • o FACTS: Title II of the Consumer Credit Protection Act prohibited loan sharking activities such as charges of excess interest, violence, and threats of violence to collect debts. A study found that organized crime takes over $350 million a year from America’s poor through loan-sharking alone. Perez was convicted of violating the law but argued to SCOTUS that the law could not be constitutionally applied to him because his business wholly operated in New York and there was no proof that he had engaged in organized crime.
    • o RULE: Title II of the Consumer Credit Protection Act is a permissible exercise by Congress of its powers under the Commerce Clause.
    • o ANALYSIS: “The Commerce Clause reaches, in the main, three categories of problems. First, the use of channels of interstate or foreign commerce which Congress deems are being misused . . . Second, protection of the instrumentalities of interstate commerce . . . Third, those activities affecting commerce. It is with this last category that we are here concerned.”
      • “We relate the history of the Act in detail to answer the impassioned plea of petitioner that all that is involved in loan sharking is a traditionally local activity. It appears, instead, that loan sharking in its national setting is one way organized interstate crime holds its guns to the heads of the poor and the rich alike and siphons funds from numerous localities to finance its national operations.”

Commerce Power and the Tenth Amendment[edit | edit source]

The Tenth Amendment between 1937 and the 1990s

  • v In Darby, the Court declared that the Tenth Amendment is “but a truism,” simply a reminder that for Congress to act it must have authority under the Constitution. This approach to the Tenth Amendment was followed without exception until 1976, when the Court invalidated a federal law for violating the Tenth Amendment.
  • v National League of Cities v. Usery
    • o The Court, by a five-to-four margin, declared unconstitutional the application of the Fair Labor Standards Act, which required payment of the minimum wage to state and local employees.
      • “there are limits upon the power of Congress to override state sovereignty, even when exercising its otherwise plenary powers to tax or to regulate commerce.”
    • o The Court found that requiring states to pay their employees the minimum wage violated the Tenth Amendment because the law “operate[s] to directly displace the States’ freedom to structure integral operations in areas of traditional governmental functions.” The Court explained that forcing state and local governments to pay their employees the minimum wage would require that they either raise taxes or cut other services to pay these costs. This would displace decisions traditionally left to the states and “may substantially restructure traditional ways in which the local governments have arranged their affairs.”
    • o Usery held that Congress violates the Tenth Amendment when it interferes with traditional state and local government functions. The Court did not attempt to define what is such a traditional function; the Court only held that forcing payment of the minimum wage was unconstitutional.
  • v Hodel v. Virginia Surface Mining & Reclamation Association
    • o The Court made it clear that Usery only applied when Congress was regulating state governments, not when Congress was regulating private conduct.
    • o Upheld a federal law that regulated strip mining and required reclamation of strip-mined land.
    • o The test for the Tenth Amendment considering Usery.
      • For a federal law to violate the Tenth Amendment, it needed to regulate “the States as States”; it must “address matters that are indisputably attribute[s] of state sovereignty”; it must directly impair the States’ ability to “structure integral operations in areas of traditional governmental functions”; and it must not be such that “the nature of the federal interest . . . justifies state submission.”
  • Garcia v. San Antonio Metropolitan Transit Authority (1985) [Page 184]
    • o FACTS: The federal government applied the Fair Labor Standards Act (Act), which regulates hours, wage and overtime conditions for covered employers, to San Antonio’s Metropolitan Transit Authority (MTA)(D). The MTA (D) objects to such an application on the ground that the Supreme Court’s ruling inNational League of Citiesprohibits the federal government from regulating the States as States, in their traditional and core government functions, a category in which the MTA (D) claims it is included.
    • o RULE: Congress has full authority under the Commerce Clause to regulate the traditional, or core, functions of state and local governments despite (notwithstanding) the Tenth Amendment.
    • o ANALYSIS: InUsery we ruled that the Act could not be applied to state and local governments “in areas of traditional government functions,” but we did not provide an explanation of how a “traditional” function is to be distinguished from a “nontraditional” function. Courts have struggled with this distinction ever since. The intervening years have convinced us that the attempt to draw such boundaries is not only unworkable, but is also inconsistent with the established principles of federalism. Usery Two reasons:
      • 1). The Usery approach had proved unworkable. “We therefore now reject, as unsound in principle and unworkable in practice, a rule of state immunity from federal regulation that turns on a judicial appraisal of whether a particular government function is ‘traditional’ or ‘integral’ . . . Any rule of state immunity that looks to the ‘traditional,’ ‘integral,’ or ‘necessary’ nature of governmental functions inevitably invites an un-elected federal judiciary to make decisions about which state policies it favors and which ones it dislikes.” '
      • 2). The Usery approach had proved unworkable. “We therefore now reject, as unsound in principle and unworkable in practice, a rule of state immunity from federal regulation that turns on a judicial appraisal of whether a particular government function is ‘traditional’ or ‘integral’ . . . Any rule of state immunity that looks to the ‘traditional,’ ‘integral,’ or ‘necessary’ nature of governmental functions inevitably invites an un-elected federal judiciary to make decisions about which state policies it favors and which ones it dislikes.” '
    • o Nine years beforeGarcia, the Court ruled inUsery that the States are free from federal intrusion into their core, traditional functions, which signaled a revival of the Tenth Amendment as an affirmative check on the government’s commerce power. Here, once again, the Tenth Amendment is dead in this regard. The Court asserts that an unelected federal judiciary should not be deciding which state functions are traditional.
    • o The result is that where, before, the Judiciary performed its constitutionally appointed task of acting as a check on congressional power, now Congress has a free hand to meddle in the purely local affairs of state and local governments.

4). Narrowing of the Commerce Power and Revival of the Tenth Amendment as a Constraint on Congress “Commerce Among the States”

  • US v. Lopez (1995) [Page 188]
    • o FACTS: Congress passed the Gun-Free School Zones Act of 1990 (Act or § 922), making it a federal offense “for any individual knowingly to possess a firearm at a place that the individual knows, or has reasonable cause to believe, is a school zone.” A “school zone” is on the grounds of or within 1,000 feet of a school. The Act neither regulates a commercial activity nor contains a requirement that the possession be connected in any way to interstate commerce. Lopez (D), a 12th-grade student, was convicted of violating the Act by carrying a concealed .38-caliber handgun and five bullets into his high school.
    • o RULE: Congressional authority to regulate pursuant to the Commerce Clause extends to only those activities that rationally implicate (1) the channels of interstate commerce; (2) the instrumentalities of interstate commerce; or (3) activities having a substantial effect upon interstate commerce.
    • o ANALYSIS: The Constitution mandates a division of authority, which was adopted by the framers to ensure protection of our fundamental liberties. A healthy balance of power between the States and the Federal Government reduces the risk of tyranny and abuse from either front. [The Court reviewed a series of Commerce Clause decisions fromGibbonsto the 1940s.] These decisions greatly expanded the authority of Congress.“The Act neither regulates a commercial activity nor contains a requirement that the possession be connected in any way to interstate commerce. We hold that the Act exceeds the authority of Congress “[t]o regulate Commerce . . . among the several States.”
    • o “We conclude, consistent with the great weight of our case law, that the proper test requires an analysis of whether the regulated activity “substantially affects” interstate commerce.”
      • “it is not necessary that each individual instance of the activity substantially affect commerce; it is enough that, taken in the aggregate, the class of activities in question has such an effect.” Solid Waste Agency of Northern Cook County v. United States Army Corps of Engineers.
  • US v. Morrison (2000) [Page 200]
    • o FACTS: In 1994, Congress passed the Violence Against Women Act, which contained a provision for a federal civil remedy for victims of gender-based violence, even when victims did not file criminal charges. Brzonkala was allegedly assaulted and raped by Morrison. Brzonkala and the U.S. sued Morrison and Virginia Tech under the Act in federal district court. Morrison challenged the Act as an unconstitutional exercise of Congress’s Commerce Clause powers. The district court held that Congress lacked authority to enact the Act, but the Fourth Circuit reversed. The Fourth Circuit then reheard the case and upheld the district court’s decision. Brzonkala and the U.S. appealed to SCOTUS.
    • o RULE: Congress may not, pursuant to the Commerce Clause, regulate a local activity solely on the basis that it has substantial effects on interstate commerce when viewed in its nationwide aggregate.
    • o ANALYSIS: Congress may not, pursuant to the Commerce Clause, regulate a local activity solely on the basis that it has substantial effects on interstate commerce when viewed in its nationwide aggregate.
      • “As we stated in Lopez, “[S]imply because Congress may conclude that a particular activity substantially affects interstate commerce does not necessarily make it so.””
      • “If accepted, petitioners’ reasoning would allow Congress to regulate any crime as long as the nationwide, aggregated impact of that crime has substantial effects on employment, production, transit, or consumption.”

Solid Waste Agency of Northern Cook County v. United States Army Corps of Engineers (205)

  • v “where an otherwise acceptable construction of a statute would raise serious constitutional problems, the Court will construe the statute to avoid such problems unless such construction is plainly contrary to the intent of Congress.”

Pierce County, Washington v. Guillen

  • v A federal statute provides that if a local government does a traffic study as part of applying for federal funds to improve state roads, that study is not discoverable. Congress’s concern was that local governments would refrain from conducting such investigations if they could be used as evidence against them in suits arising from automobile accidents.
  • v Justice Thomas: “[i]t is well established that the Commerce Clause gives Congress authority to regulate the use of the channels of interstate commerce. . . . [The statutes] can be viewed as legislation aimed at improving safety in the channels of interstate commerce and increasing protection for the instrumentalities of interstate commerce. As such, they fall within Congress’ Commerce Clause power.”
  • Gonzales v. Raich (2005) [Page 208]
    • o TOPIC:
    • o FACTS: Congress enacted the Controlled Substances Act (CSA) which categorized illegal drugs into different “schedules” and prevented their sale, purchase, and possession. California enacted the Compassionate Use Act that allowed the use of medical marijuana within the state by persons needing it for legitimate medical purposes. Raich was a California resident who legally used marijuana to treat legitimate medical issues. Raich brought this suit against Attorney General Gonzales, seeking injunctive and declaratory relief prohibiting the enforcement of the federal CSA.
    • o RULE: Congress may regulate intrastate activity if there is a rational basis for concluding that the activity may have a substantial effect on interstate commerce.
    • o ANALYSIS: Case law has firmly established the power of Congress to regulate purely local activities that are part of an economic class of activities that have a substantial effect on interstate commerce. If Congress decides that the total incidence of a practice poses a threat to a national market, it may regulate all of the activity. When a general regulatory statute bears a substantial relation to commerce, the de minimis character of individual instances is of no consequence.
      • Similar to Wickard, distinguishable from Lopez and Morrison.
      • Wickard thus establishes that Congress can regulate purely intrastate activity that is not itself “commercial,” in that it is not produced for sale, if it concludes that failure to regulate that class of activity would undercut the regulation of the interstate market in that commodity.”
    • o In addition, the activities regulated by the Controlled Substances Act are essentially economic. Prohibiting the intrastate possession or manufacture of an article of commerce is a rational, and common, means of regulating commerce in that product.
    • o Scalia Concurrence: “Congress’s regulatory authority over intrastate activities that are not themselves part of interstate commerce (including activities that have a substantial effect on interstate commerce) derives from the Necessary and Proper Clause.”
    • o O’Connor Dissent: “a single courageous State may, if its citizens choose, serve as a laboratory; and try novel social and economic experiments without risk to the rest of the country.” New State Ice Co. v. Liebmann
    • o Thomas Dissent: “By holding that Congress may regulate activity that is neither interstate nor commerce under the Interstate Commerce Clause, the Court abandons any attempt to enforce the Constitution’s limits on federal power.”

Tenth Amendment Limitations[edit | edit source]

  • NY v. US (1992) [Page 217]
    • o FACTS: Congress passed the Low-Level Radioactive Waste Policy Amendments Act of 1985 (Act). The Act has three provisions regarding radioactive waste disposal: (1) monetary incentives for states to open their own waste sites; (2) access incentives allowing states without waste sites to be denied access to sites in other states; (3) a “take title” provision requiring states to take ownership of waste generated within its borders after January 1, 1996, if the state has been unable to arrange for proper disposal before that date. The take title provision would make states liable for all damages incurred by the waste generator or owner as a result of the State’s failure to procure a disposal site. New York (P) and two of its counties are seeking to have the Act declared invalid as an impermissible invasion of state sovereignty, i.e. a violation of the Tenth Amendment.
    • o RULE: Congress does not have the authority to commandeer state governments by forcing them to implement particular regulations.
    • o ANALYSIS: This case illustrates the different values the Justices place on federalism and the balance of power it strikes between the States and the federal government. The majority view represents a strict and formalistic adherence to federalism, unwilling to compromise the separation of power between state and federal governments. Much of the rationale behind this position is related to the accountability of elected officials, both national and local. The point the majority makes is that if the Federal Government is allowed to order states to enact regulations, the federal officials responsible may be able to evade accountability if local citizens disapprove of the regulation. Instead, local officials may be forced to take the heat for something they are not responsible for.
      • “While Congress has substantial power under the Constitution to encourage the States to provide for the disposal of the radioactive waste generated within their borders, the Constitution does not confer upon Congress the ability simply to compel the States to do so.”
      • Policy: “Where Congress encourages state regulation rather than compelling it, state governments remain responsive to the local electorate’s preferences; state officials remain accountable to the people. By contrast, where the Federal Government compels States to regulate, the accountability of both state and federal officials is diminished . . . it may be state officials who will bear the brunt of public disapproval, while the federal officials who devised the regulatory program may remain insulated from the electoral ramifications of their decision.”
      • A choice between two unconstitutionally coercive regulatory techniques is no choice at all. Either way, “the Act commandeers the legislative processes of the States by directly compelling them to enact and enforce a federal regulatory program,” Hodel v. Virginia Surface Mining & Reclamation Assn., Inc.”
      • “The Constitution does not protect the sovereignty of States for the benefit of the States or state governments . . . or even for the benefit of the public officials governing the States . . . the Constitution divides authority between federal and state governments for the protection of individuals.”
  • Printz v. US (1997) [Page 227]
    • o FACTS: In 1993, Congress amended the Gun Control Act of 1968 by passing the Brady Handgun Violence Prevention Act (Brady). Brady requires the Attorney General to establish a national instant background check system by November 30, 1998, and creates interim provisions until the system goes online. Under these interim provisions, state and local law enforcement personnel must do background checks before issuing permits to purchase firearms. Under Brady, firearms dealers are required to forward Brady Forms to the local chief law enforcement officers (CLEOs), who are then obligated to make “reasonable efforts” within five days to determine whether the sales listed on the forms are lawful. Jay Printz and Richard Mack (CLEOs) (P) are chief local law enforcement officials who challenge the constitutionality of Brady’s interim provisions.
    • o RULE: Congress does not have authority to compel states to enact, enforce, or administer federal regulatory programs, and cannot circumvent this prohibition by conscripting state officials directly.
    • o ANALYSIS: Constitutional: “The Constitution does not leave to speculation who is to administer the laws enacted by Congress; the President, it says, “shall take Care that the Laws be faithfully executed,” Art. II, §3 . . . The Brady Act effectively transfers this responsibility to thousands of CLEOs in the 50 States, who are left to implement the program without meaningful Presidential control . . . the power of the President would be subject to reduction, if Congress could act as effectively without the President as with him, by simply requiring state officers to execute its laws.”
    • o Jurisprudence: “as we concluded categorically in New York: “The Federal Government may not compel the States to enact or administer a federal regulatory program.”
    • o Dissent: “When Congress exercises the powers delegated to it by the Constitution, it may impose affirmative obligations on executive and judicial officers of state and local governments as well as ordinary citizens. This conclusion is firmly supported by the text of the Constitution, the early history of the Nation, decisions of this Court, and a correct understanding of the basic structure of the Federal Government.”
  • Reno v. Condon (2000) [Page 235]
    • o FACTS: In 1994, Congress passed the Driver’s Privacy Protection Act to regulate the disclosure of personal information retained by state DMVs. Congress passed this legislation because states routinely obtained significant personal information from individuals in connection with those individuals obtaining driver’s licenses and then sold that information to private entities at a profit for the state. The private entities could also further resell this information. The Act was designed to prohibit the selling or reselling of individuals’ personal information by DMVs or private entities without obtaining the individual’s consent. South Carolina and its Attorney General, Condon, sued the U.S. and its Attorney General, Reno, that the DPPA violated the Tenth and Eleventh Amendments of the Constitution. The district court granted summary judgment for Condon, and the court of appeals affirmed. Reno appealed to SCOTUS.
    • o RULE: States are required to comply with constitutionally valid legislation regulating state activities, even when compliance means incurring additional costs to be borne by the States.
    • o ANALYSIS: The Court ruled that the Act was constitutional as an exercise of Congress's commerce clause power because “States … sell this personal information to individuals and businesses [and] [t]hese sales generate significant revenues for the States.”
    • o “[We] reject the State’s argument that the DPPA violates the principles laid down in either New York or Printz . . . this case is governed by . . . South Carolina v. Baker. In Baker, we upheld a statute that prohibited States from issuing unregistered bonds because the law “regulate[d] state activities,” rather than “seek[ing] to control or influence the manner in which States regulate private parties . . . Such “commandeering” is, however, an inevitable consequence of regulating a state activity.”

The Taxing and Spending Power[edit | edit source]

  • US v. Butler (1936) [Page 238]
    • o FACTS: The Agricultural Adjustment Act declared that b/c of a crisis in agricultural production, the Secretary of Agriculture, among other powers, could set limits on production of certain crops and impose taxes on production in excess of these limits. The Act also authorized grants to farmers to control production and thus regulate prices.
    • o RULE: Congress may not use the taxing or spending powers to force compliance in an area where the Constitution does not give Congress independent power to regulate.
    • o ANALYSIS: The Court began by noting that the debate over the scope of the taxing and spending power goes back to a dispute between James Madison and Alexander Hamilton. Madison took the view that Congress was limited to taxing and spending to carry out the other powers specifically granted in Article I of the Constitution. In contrast, Hamilton took the position that Congress could tax and spend for any purpose that it believed served the general welfare, so long as Congress did not violate another constitutional provision. The Court expressly endorsed Hamilton's position as “the correct one.”
      • Despite this broad construction of Congress’s taxing power, the Court ruled the Act was unconstitutional because it violated the Tenth Amendment; it regulated production and this, according to the Court, was left to the states.
        • “The act invades the reserved rights of the states. It is a statutory plan to regulate and control agricultural production, a matter beyond the powers delegated to the federal government.”
      • Sabri v. US (2004) [Page 240]
        • o FACTS: Sabri (D) challenged a federal anti-bribery statute as facially unconstitutional. Sabri (D) argued that the statute was facially unconstitutional because it failed to require a connection between federal funds and the alleged bribe. After the federal district court granted Sabri’s (D) motion, the Eighth Circuit Court of Appeals reversed, holding that the statute was constitutional under the Necessary and Proper Clause in furtherance of Congress’s spending power.
        • o RULE: Under the Spending Clause, Congress is authorized to appropriate federal funds for the general welfare and use all rational means necessary and proper to further its spending power.
        • o ANALYSIS: Sabrisubstantially strengthens the federal government’s regulation of state and local matters. By bestowing powers in connection with contributions of federal funds to state and local organizations, the Necessary and Proper Clause effectively grants the federal government carte blanche to regulate any matter directly or indirectly related to the contributions. Yet, despite these important federalist concerns,Sabriwas a unanimous decision of the Court.
        • o “We simply do not presume the unconstitutionality of federal criminal statutes lacking explicit provision of a jurisdictional hook, and there is no occasion even to consider the need for such a requirement where there is no reason to suspect that enforcement of a criminal statute would extend beyond a legitimate interest cognizable under Article I, §8.”
        • o “Congress has authority under the Spending Clause to appropriate federal monies to promote the general welfare, Art. I, §8, cl. 1, and it has corresponding authority under the Necessary and Proper Clause, Art. I, §8, cl. 18, to see to it that taxpayer dollars appropriated under that power are in fact spent for the general welfare, and not frittered away in graft or on projects undermined when funds are siphoned off or corrupt public officers are derelict about demanding value for dollars”
        • o Sabri’s claim was that Congress only could prohibit bribery as to those state, local, and tribal activities that actually got federal money. The Supreme Court rejected this argument and explained: “Money is fungible, bribed officials are untrustworthy stewards of federal funds, and corrupt contractors do not deliver dollar-for-dollar value.”
  • South Dakota v. Dole (1987) [Page 242]
    • o FACTS: A South Dakota law permitted persons age 19 or older to buy beer containing up to 3.2% alcohol. Congress passed legislation (§ 158) directing Dole (D), the Secretary of the Treasury, to withhold 5% of federal highway funds from any state that fails to set its legal age for alcohol consumption at 21 years. South Dakota (P) asserts that § 158 violates the constitutional limitations on congressional exercise of the spending power, as well as the Twenty-first Amendment to the Constitution.
    • o RULE: Valid use of the Spending power is subject to three requirements: (1) It must be used for the general welfare; (2) Any conditions on receipt of funds must be unambiguous; and (3) Any conditions must be related to the federal interest in the particular national projects or programs being funded.
    • o ANALYSIS: Incident to the spending power, Congress may achieve its broad policy objectives by conditioning the receipt of federal funds upon compliance by the recipient with federal statutory and administrative directives. Such use of the spending power is, of course, not unlimited, but is subject to several general restrictions.
      • The first limitation is that the exercise of the spending power must be in pursuit of the “general welfare.” In considering whether this requirement is met, courts should defer substantially to the judgment of Congress. Second, any conditions must be set forth unambiguously, allowing the States to exercise their choice knowingly, cognizant of the consequences of their participation. Third, conditions on federal grants must be related to the federal interest in particular nationwide projects or programs.
    • o Justice O’Connor disagrees with the majority in this case. Justice O’Connor would require there to be a direct nexus between the funds’ purpose and the conditions of their use, while the majority would require only a tangential relationship. Furthermore, the majority seems to be willing to let Congress be the “judge” of whether there exists the required reasonable relationship between the funds’ purpose and the conditions. On another note, both Justices O’Connor and Brennan interpret the Twenty-first Amendment as reserving the regulation of alcohol exclusively to the States. Under this interpretation, Congress is absolutely foreclosed from exercising influence, in any way, on state regulation of the alcohol age limit.
    • o “Our decisions have recognized that in some circumstances the financial inducement offered by Congress might be so coercive as to pass the point at which “pressure turns into compulsion.” Here, however, Congress has directed only that a State desiring to establish a minimum drinking age lower than 21 lose a relatively small percentage of certain federal highway funds.”
    • o Pennhurst State School and Hospital v. Halderman: Congress may place strings on grants to state and local governments as long as the conditions are expressly stated.

Congress’s Powers Under the Post-Civil War Amendments[edit | edit source]

  • v After the Civil War, three extremely important amendments were added to the Constitution.
    • o The 13th Amendment
      • Prohibits slavery and involuntary servitude, except as a punishment for a crime, and also provides in §2, “Congress shall have power to enforce this article by appropriate legislation.”
    • o The Fourteenth Amendment
      • Provides that all persons born or naturalized in the United States are citizens and that no state can abridge the privileges or immunities of such citizens; nor may states deprive any person of life, liberty, or property without due process of law or deny any person of equal protection of the laws. §5 of the Fourteenth Amendment states, “[T]he Congress shall have power to enforce, by appropriate legislation, the provisions of this article.”
    • o The Fifteenth Amendment
      • “[t]he right of citizens of the United States to vote shall not be denied or abridged by the United States or by any State on account of race, color, or previous condition of servitude.” §2 again provides that Congress has the power to enforce it by appropriate legislation.
  1. Whom May Congress Regulate Under the Post-Civil War Amendments?
  • v The Civil Rights Act of 1875 provided that all persons were “entitled to the full and equal enjoyment of the accommodations, advantages, facilities and privileges of inns, public conveyances, on land or water, theatres, and other places of public amusement; subject only to the conditions and limitations established by law, and applicable to citizens of every race and color, regardless of any previous condition of servitude.”
  • v By an eight-to-one decision, the Court held that the act was unconstitutional and adopted a restrictive view as to the power of Congress to use these provisions to regulate private behavior.
    • o In the Civil Rights Cases (1883), the Court held that Congress, pursuant to §2 of the Thirteenth Amendment and §5 of the Fourteenth Amendment, may regulate only state and local government actions, not private conduct.
      • Thirteenth Amendment: Congress’s power was limited to ensuring an end to slavery; Congress could not use this power to eliminate discrimination. Congress could abolish “all badges and incidents of slavery,” but it could not use its power under the Thirteenth Amendment to “adjust what may be called the social rights of men and races in the community.”
      • Fourteenth Amendment: Congress’s authority was only over state and local governments and their officials, not over private conduct: “It does not authorize Congress to create a code of municipal law for the regulation of private rights; but to provide modes of redress against the operation of State laws, and the actions of State officers.”
    • v Jones v. Alfred H. Mayer Co. (247)
      • o Seminal case which held that Congress could prohibit private discrimination in selling and leasing property.
      • o The case involved a private real estate developer who refused to sell housing or land to African Americans. An African-American couple sued under 42 U.S.C. §1982, which provides that all citizens have “the same right, in every State and Territory, as is enjoyed by white citizens thereof to inherit, purchase, lease, sell, hold and convey real and personal property.”
      • o The Court held that §1982 applies to prohibit private discrimination and that Congress has broad legislative power under the Thirteenth Amendment to do so: “Congress has the power under the Thirteenth Amendment rationally to determine what are the badges and incidents of slavery, and the authority to translate that determination into effective legislation.”
    • v Runyon v. McCrary
      • o 42 U.S.C. §1981 applies to prohibit discrimination in private contracting and that this is within the scope of Congress's power under §2 of the Thirteenth Amendment.
      • o Runyon raised the question of whether §1981 prohibits private schools from excluding qualified African-American children solely because of their race. The Supreme Court saw no basis for distinguishing Jones v. Alfred H. Mayer Co. and concluded “that §1981, like §1982, reaches private conduct.”
    • v United States v. Guest
      • o Overruled later, §5 of the Fourteenth Amendment cannot be used to regulate private activity.
  • US v. Morrison (2000) [Page 248]
    • o FACTS: Morrison involved a constitutional challenge to the civil damages provision of the Violence Against Women Act, which authorized victims of gender-motivated violence to sue under federal law. The United States government intervened to defend the law, and it and the plaintiff argued that the civil damages provision was constitutional as an exercise of both Congress's commerce clause power and its authority under §5 of the Fourteenth Amendment.
    • o RULE: Congress’s authority to regulate under the Fourteenth Amendment extends only to state activity, not activities of private individuals.
    • o ANALYSIS: The Court held that the law is not constitutional as an exercise of Congress's §5 power. The Court, said that Congress under this authority may regulate only state and local governments, not private conduct. “the time-honored principle that the Fourteenth Amendment, by its very terms, prohibits only state action.”
    • o Chief Justice Rehnquist said that the opinions in United States v. Guest, indicating congressional power to regulate private conduct, were only dicta. Thus, the civil damages provision of the Act was deemed to exceed the scope of Congress's §5 powers because it “is not aimed at proscribing discrimination by officials which the Fourteenth Amendment might not itself proscribe; it is directed not at any State or state actor, but at individuals who have committed criminal acts motivated by gender bias.”
    • o Morrison is thus an important limit on Congress's powers under §5 of the Fourteenth Amendment. The Court's holding can be defended as being consistent with the long-standing principle that the amendment applies only to government conduct. But the decision can be criticized as an unduly narrow interpretation of a constitutional provision that was intended to have broad scope. The Act was adopted because of perceived inadequacies in state courts and state laws. Critics of Morrison argue that §5 should be read as authorizing congressional action under such circumstances.
  1. What Is the Scope of Congress’s Power?
  • v A second major issue concerning Congress's power under the Reconstruction Amendments concerns the scope of authority under these provisions. Is Congress limited to providing remedies for violations of constitutional rights recognized by the Supreme Court; or may Congress use its power under these amendments to adopt an independent interpretation of the Constitution, even overruling Supreme Court decisions?
  • Katzenbach v. Morgan (1966) [Page 251]
    • o TOPIC: Congress, under §5 of the Fourteenth Amendment, may independently interpret the Constitution and even overturn the Supreme Court.
    • o FACTS: Katzenbach concerned the constitutionality of §4(e) of the Voting Rights Act of 1965, which provides that no person who has completed sixth grade in a Puerto Rican school, where instruction was in Spanish, shall be denied the right to vote because of failing an English literacy requirement. Earlier, in Lassiter v. Northampton County Board of Elections, the SC had upheld the constitutionality of an English language literacy requirement for voting. Congress, in the Voting Rights Act, sought to partially overturn Lassiter by providing that failing a literacy test could not bar a person from voting if the person was educated through the sixth grade in Puerto Rico.
    • o RULE: Congress may pass legislation to enforce the Equal Protection Clause of the Fourteenth Amendment even when the legislation conflicts with state law.
    • o ANALYSIS: The SC upheld this provision as “a proper exercise of the powers granted to Congress by §5 of the Fourteenth Amendment” for two reasons.
      • 1. Congress could have concluded that granting Puerto Ricans the right to vote would empower them and help them to eliminate discrimination against them. The law was constitutional because it was a remedy for discrimination.
      • 2. Congress could find that the literacy test denied equal protection, even though this was contrary to the Court's earlier holding in Lassiter. This aspect of the ruling is much more significant because it accords Congress the authority to define the meaning of the Fourteenth Amendment.
    • o The Court explained that “[b]y including §5 the draftsmen sought to grant to Congress, by a specific provision applicable to the Fourteenth Amendment, the same broad powers expressed in the Necessary and Proper Clause.”
    • o A concern raised by the dissent was that if Congress can use its power under §5 to interpret the Constitution, it conceivably could use this authority to dilute or even negate constitutional rights. In a footnote, Justice Brennan, the author of the majority opinion, responded to this concern: “Contrary to the suggestion of the dissent, §5 does not grant Congress power to exercise discretion in the other direction and to enact ‘statutes so as in effect to dilute equal protection and due process decisions of this Court.’ We emphasize that Congress's power under §5 is limited to adopting measures to enforce the guarantees of the Amendment; §5 grants Congress no power to restrict, abrogate, or dilute these guarantees.”
  • City of Boerne v. Flores (1997) [Page 255]
    • o TOPIC: Congress may not use its §5 powers to expand the scope of rights or to create new rights. Compare to Katzenbach (broad interpretation of Congress’s powers in comparison to a restrictive view here).
    • o FACTS: A church in Texas was prevented from constructing a new facility because its building was classified a historic landmark. The church sued under the Religious Freedom Restoration Act, and the city challenged the constitutionality of the law.
    • o RULE: Section 5 of the Fourteenth Amendment gives Congress the power to enact laws as remedial measures and to prevent constitutional violations, but does not allow Congress to define the substantive scope of constitutional guarantees.
    • o ANALYSIS: The Court held that Congress under §5 of the Fourteenth Amendment may not create new rights or expand the scope of rights; rather Congress is limited to laws that prevent or remedy violations of rights recognized by the Supreme Court, and these must be narrowly tailored—“proportionate” and “congruent”—to the constitutional violation.
      • “Legislation which alters the meaning of the Free Exercise Clause cannot be said to be enforcing the Clause. Congress does not enforce a constitutional right by changing what the right is. It has been given the power ‘to enforce,’ not the power to determine what constitutes a constitutional violation. Were it not so, what Congress would be enforcing would no longer be, in any meaningful sense, the ‘provisions of [the Fourteenth Amendment].’ ”
      • Justice Kennedy defended this conclusion by invoking the need to preserve the Court as the authoritative interpreter of the Constitution. Justice Kennedy quoted Marbury v. Madison and wrote: “If Congress could define its own powers by altering the Fourteenth Amendment's meaning, no longer would the Constitution be ‘superior paramount law unchangeable by ordinary means.’ It would be ‘on a level with ordinary legislative acts, and like other acts, … alterable when the legislature shall please to alter it.’ ”
  • Shelby Country, Alabama v. Holder (2013) [Page 261]
    • o TOPIC: The Court considered the constitutionality of key provisions of the Voting Rights Act, but unlike in the prior decisions, this time the Court declared aspects of the law unconstitutional.
    • o FACTS: 2 of the Voting Rights Act prohibits state and local governments from having election practices or systems that have a discriminatory effect against minority voters. Lawsuits can be brought to enforce it. But Congress believed that this was not sufficient to stop discrimination in voting. Congress knew that litigation is expensive and time consuming. Congress also knew that Southern states especially had the practice of continually changing their voting systems to disenfranchise minority voters. §5 of the Voting Rights Act provides that jurisdictions with a history of race discrimination in voting may change their election systems only if they get “preclearance” from the attorney general or a three-judge federal district court. §4(B) of the Act defines those jurisdictions that must get preclearance, nine states and many local governments with a history of race discrimination in voting. Each time the law was about to expire, Congress extended it.
    • o RULE: Any departure from the basic principle of equal sovereignty of states must be justified by a showing that the departure is justified by current needs and sufficiently related to the problem that it targets.
  • o ANALYSIS: The Court, 5 to 4, held §4(B) unconstitutional and thereby also effectively nullified §5 because it applies only to jurisdictions covered under §4(B). It is the first time since the nineteenth century that the Court declared unconstitutional a federal civil rights statute.
    • The Constitution and laws of the United States are “the supreme Law of the Land,” but outside the strictures of the Supremacy Clause, states retain broad autonomy in structuring their governments. This allocation of powers is not just an end in itself, but secures to citizens the liberties that derive from the diffusion of sovereign power. More specifically, the Framers of the Constitution intended that the States would retain the power to regulate elections.
    • In addition to the sovereignty of states, there is also a fundamental principle of equal sovereignty among the states, which is “essential to the harmonious operation of the scheme upon which the Republic was organized.” The Voting Rights Act departs from these basic principles. It suspends all changes to state election law—however innocuous—until they have been precleared by federal authorities. States must beseech the Federal Government for permission to implement laws that they would otherwise have the right to enact and execute on their own.

Congress’s Power to Authorize Suits Against State Governments[edit | edit source]

The 11th Amendment

  • v The Eleventh Amendment was adopted to overrule Chisholm v. Georgia (1793). Chisholm involved an attempt by a South Carolina citizen to recover money owed by Georgia. He sued in federal court pursuant to the language of Article III that expressly allows federal courts to hear suits against state governments by citizens of other states. Georgia claimed that it had sovereign immunity and could not be sued without its consent. The Supreme Court, in a four-to-one decision, ruled in Chisholm’s favor. They concluded that the clear language of Article III authorized suits against a state by citizens of another state.
  • v Hans v. Louisiana (1890), the Court held that the Eleventh Amendment also bars suits against a state by its own citizens.
  • v There are three ways around the Eleventh Amendment to hold state governments accountable in federal court.
    • o Ex parte Young (1908), state officers may be sued for injunctive relief or for damages to be paid by them, but state officers cannot be sued where it is the state treasury that will be paying damages to compensate for past wrongs.
    • o States may waive their Eleventh Amendment immunity and may consent to be sued in federal court. But the “test for determining whether a state has waived its [Eleventh Amendment] immunity from federal-court jurisdiction is a stringent one.” The Court has held that waivers must be explicit; there is no doctrine of implied or constructive waiver of the Eleventh Amendment.
    • o The Supreme Court has held that Congress, acting pursuant to §5 of the Fourteenth Amendment, may authorize suits against state governments. This authority is established in the cases in the next section. Following the cases creating this authority, three Supreme Court decisions construing the scope of Congress’s §5 power are presented. Finally, the section concludes by presenting the Supreme Court’s recent decision in Alden v. Maine, holding that Congress cannot authorize suits against state governments in state courts. Alden held that state governments have sovereign immunity and cannot be sued in state courts without their consent even for violations of federal laws.
  • Fitzpatrick v. Bitzer (1976) [Page 274]
    • o TOPIC: Congress could authorize suits against state governments if it acts pursuant to §5 of the Fourteenth Amendment.
    • o FACTS: Congress amended Title VII of the Civil Rights Act to allow a federal cause of action against state governments for employment discrimination. The provision is being challenged under the Eleventh Amendment. State governments may be sued for violating Title VII of the 1964 Civil Rights Act, which prevents employment discrimination based on race, gender, and religion.
    • o RULE: No constitutional provision prohibits Congress from providing for a private cause of action in the federal courts against a state government as a means of enforcing the guarantees of the Fourteenth Amendment.
    • o ANALYSIS: The Fourteenth Amendment followed the Eleventh Amendment and thus can modify it. More importantly, the Court said that the Fourteenth Amendment was intended as a limit on state power. Justice Rehnquist explained: “When Congress acts pursuant to §5, not only is it exercising legislative authority that is plenary within the terms of the constitutional grant, it is exercising that authority under one section of a constitutional Amendment whose other sections by their own terms embody limitations on state authority. We think that Congress may, in determining what is ‘appropriate legislation’ for the purpose of enforcing the provisions of the Fourteenth Amendment, provide for private suits against States or state officials which are constitutionally impermissible in other contexts.”

Pennsylvania v. Union Gas Co. (1989)

  • v Congress may override the Eleventh Amendment and authorize suits against state governments pursuant to any of its constitutional powers, as long as the law in its text expressly authorizes such suits.
  • Seminole Tribe of Florida v. Florida (1996) [Page 276]
    • o TOPIC: The Supreme Court expressly overturned Pennsylvania v. Union Gas.
    • o FACTS: Congress passed a law allowing states to be sued for failing to negotiate in good faith with Indian tribes regarding the formation of gaming compacts between those parties. The law is challenged as a violation of the Eleventh Amendment’s sovereign immunity.
    • o RULE: Congress may not, outside enforcement of Fourteenth Amendment Guarantees, authorize federal lawsuits against states in abrogation of the Eleventh Amendment’s guarantee of state sovereign immunity.
    • o ANALYSIS: The Court stressed that Pennsylvania v. Union Gas is an unprecedented expansion in Congress's power to authorize suits against state governments. “Even when the Constitution vests in Congress complete law-making authority over a particular area, the Eleventh Amendment prevents congressional authorization of suits by private parties against unconsenting States. The Eleventh Amendment restricts the judicial power under Article III, and Article I cannot be used to circumvent the constitutional limitations placed upon federal jurisdiction.” The Court held that Congress only can authorize suits against state governments, and override the Eleventh Amendment, when it acts pursuant to §5 of the Fourteenth Amendment.
    • o The Court holds inSeminole Tribe of Floridathat Congress may not vest federal courts with jurisdiction over federal question suits against state governments, regardless of whether the plaintiff is from the would-be defendant state or some other state or nation. This rule is, of course, qualified by Congress’s authority to enforce the substantive guarantees of the Fourteenth Amendment, as set forth inFitzpatrick v. Bitzer.How does this ruling jibe with the text of the Eleventh Amendment? The Eleventh Amendment reads: “The Judicial power of the United States shall not be construed to extend to any suit in law or equity, commenced or prosecuted against one of the United States by Citizens of another State, or by Citizens or Subjects of any Foreign State.” In plain English, this means that Congress may not vest jurisdiction in the federal courts over suits against States brought by citizens of a different state, or of another country. The Amendment thus does not address suits brought against a state by one of its own citizens.
  • Florida Prepaid Postsecondary Education Expense Board v. College Savings Bank and US (1999) [Page 281]
    • o TOPIC: Congress must show a pattern of state violations of the fourteenth amendment to abrogate state sovereignty.
    • o FACTS: Congress enacted legislation providing a right to sue states in federal court over patent infringement. The legislation is challenged as beyond Congress’s Fourteenth Amendment enforcement authority.
    • o RULE: For Congress’s abrogation of State sovereignty pursuant to the Fourteenth Amendment to be constitutional, it must (1) show a history or pattern of unconstitutional activity by States giving rise to a need for remedial or preventive federal regulation, and (2) limit the scope of the remedy, making it proportionate to the constitutional violations giving rise to the need for enforcement.
    • o ANALYSIS: SCOTUS held that the law was not a valid exercise of power under §5 of the Fourteenth Amendment and thus could not be used to sue the state government.
      • Although patents are property and the Fourteenth protects property from being deprived by state governments without due process, the authorization of suits was impermissible because it was not “proportionate” or “congruent” to remedy constitutional violations. “In enacting the Patent Remedy Act, however, Congress identified no pattern of patent infringement by the States, let alone a pattern of constitutional violations. Unlike the undisputed record of racial discrimination confronting Congress in the voting rights cases, Congress came up with little evidence of infringing conduct on the part of the States.”
      • The Court held that the law was not valid under §5 because “[t]he legislative record thus suggests that the Patent Remedy Act does not respond to a history of ‘widespread and persisting deprivation of constitutional rights' of the sort Congress has faced in enacting proper prophylactic §5 legislation.”
      • BUT: barring patent infringement suits against state governments in federal court means that a state government can infringe patents without ever facing a lawsuit.
  • Kimel v. Florida Board of Regents (2000) [Page 285]
    • o TOPIC: Congress may not use its fourteenth amendment authority to remedy age discrimination in employment by states.
    • o FACTS: The named case involved a suit by current and former faculty and librarians at Florida State University, including Kimel. They alleged that the university's failure to provide promised pay adjustments discriminated against older workers and thus violated the ADEA. A companion case was brought by Wellington Dickson, an employee of the Florida Department of Corrections, who claimed that he was denied promotions because of his age. Another of the consolidated cases involved faculty members at a state university in Alabama who claimed age discrimination.
    • o RULE: In order for Congressional legislation abrogating State sovereign immunity pursuant to § 5 of the Fourteenth Amendment to be valid, its substantive requirements must be congruent with and proportionate to the unconstitutional actions of the States.
    • o ANALYSIS: The Court concluded that the burdens the ADEA imposes on state and local governments are disproportionate to any unconstitutional behavior that might exist.
    • o Under prior decisions, only rational basis review (being rationally related to a legitimate government purpose) is used for age discrimination. The Court explained that there is not a “history of purposeful discrimination” based on age and that “age also does not define a discrete and insular minority because all persons, if they live out their normal life spans,” will experience it. The Court said that states “may discriminate based on age without offending the Fourteenth Amendment if the age classification is rationally related to a legitimate state interest.” The Court said that age often is a relevant criteria for employers. Therefore, the broad prohibition of age discrimination in the ADEA was deemed to exceed the scope of Congress's power. The ADEA prohibits a great deal of conduct that is otherwise constitutional.

RATIONAL-BASIS TEST: A principle whereby a court will uphold a law as valid under the Equal Protection Clause or Due Process Clause if it bears a reasonable relationship to the attainment of some legitimate governmental objective.INTERMEDIATE SCRUTINY: A standard lying between the extremes of rational-basis review and strict scrutiny. Under the standard, if a statute contains a quasi-suspect classification (such as gender or legitimacy), the classification must be substantially related to the achievement of an important governmental objective.STRICT SCRUTINY: To pass strict scrutiny, the legislature must have passed the law to further a "compelling governmental interest," and must have narrowly tailored the law to achieve that interest.

  • Board of Trustees, University of Alabama v. Garrett (2001) [Page 289]
    • o FACTS: The Court considered whether state governments may be sued for violating Title I of the Americans with Disabilities Act, which prohibits employment discrimination against the disabled and requires reasonable accommodation for disabilities by employers. Garrett was the director of nursing at the University of Alabama, Birmingham hospital. She was diagnosed with breast cancer and took time off work to have surgery, chemotherapy, and radiation. When she returned to work, she was informed that her position as director of nursing was no longer available. She sued under Title I of the ADA. The plaintiffs' key argument was that the elaborate legislative history documenting government discrimination against the disabled made the ADA different from other laws the Court had considered in the last few years.
    • o RULE: The Americans with Disabilities Act does not hold state governments liable for employment discrimination on the basis of an employee’s disability.
    • o ANALYSIS: In a 5-to-4 decision, the Court rejected this argument and held that state governments may not be sued for violating Title I of the ADA.
      • The ADA was a substantial expansion of rights compared to the Constitution.
      • Under equal protection, discrimination based on disability only need meet a rational basis test, being rationally related to a legitimate government purpose. The ADA prohibits much more than would fail a rational basis test, and its requirement for reasonable accommodation of disabilities is significantly greater than the Constitution requires. The Court then concluded that Title I of the ADA is not “proportionate” or “congruent” to preventing and remedying constitutional violations.
      • Chief Justice Rehnquist added a footnote to make clear that the Court was not declaring the ADA unconstitutional as applied to state governments, but rather only holding that state governments could not be sued by individuals for violations. He explained that the federal government still could sue the states to enforce the law and that suits against individual government officers for injunctive relief were also permissible. But damages actions against state governments are barred.
  • Nevada Department of HR v. Hibbs (2003) [Page 301]
    • o FACTS: Hibbs (P) sued his government employer for wrongful discharge and violations of the Family and Medical Leave Act when he was fired while caring for his injured wife.
    • o RULE: State governments are liable for money damages for interfering or restraining state employees’ exercise of their rights under the Family and Medical Leave Act of 1993.
    • o ANALYSIS: When Congress abrogates the states’ Eleventh Amendment sovereign immunity, it must do so unmistakably in the language of the statute and act pursuant to a valid exercise of its power under § 5 of the Fourteenth Amendment. Since the language of the Act itself clearly states that a “public agency” is covered by its terms, Congress’s intent to apply the Act to state governments is unmistakably clear. The Act intends to eliminate gender-based classifications in private and public employment and therefore requires evidence of a pattern of constitutional violations in public employment to fall within Congress’s § 5 powers.
    • o “FMLA aims to protect the right to be free from gender-based discrimination in the workplace.”
      • The Court said that Congress, recognizing social realities, found that the absence of family leave policies disadvantaged women in the workplace. Although the FMLA is gender neutral in that it requires leaves be granted to both men and women, and Hibbs was male, the Court said that Congress clearly intended the law to prevent gender discrimination in employment.
    • o Distinguished Kimel and Garrett claiming they involved types of discrimination that receive only rational basis review, whereas gender discrimination triggers intermediate scrutiny under equal protection. “Here, however, Congress directed its attention to state gender discrimination, which triggers a heightened level of scrutiny. Because the standard for demonstrating the constitutionality of a gender-based classification is more difficult to meet than our rational basis test, … it was easier for Congress to show a pattern of constitutional violations.”
  • Tennessee v. Lane (2004) [Page 308]
    • o TOPIC: The Court followed and extended Hibbs.
    • o FACTS: A criminal defendant literally climbed on his hands and knees to get to a second-floor courtroom because it was not accessible to those with disabilities. He sued the state government pursuant to Title II of the Americans with Disabilities Act, which prohibits state and local governments from discriminating against people with disabilities in government programs, services, and activities.
    • o RULE: To abrogate the states’ Eleventh Amendment immunity, Congress must have unequivocally expressed its intent to abrogate that immunity and do so under a valid grant of constitutional authority.
    • o ANALYSIS: The Court, in a 5-to-4 decision, held that Lane's suit against the state was not barred by sovereign immunity. The Court emphasized that there is a well-established fundamental right' of access to the courts. Congress has greater latitude to legislate under §5 when dealing with a claim that receives heightened judicial scrutiny, whether because it is a fundamental right or a type of discrimination that receives heightened scrutiny. Does not address whether states can be sued under Title II when there is not a fundamental right that is implicated.'
  • Alden v. Maine (1999) [Page 308]
    • o TOPIC: States sovereign immunity extends to suits in state courts.
    • o FACTS: Two probation officers sued their state employer in state court for violating federal employment laws. The State raises the state sovereign immunity defense.
    • o RULE: Congress does not possess the authority, under its Article I powers, to abrogate the States’ sovereign immunity from suits in its own state courts.
    • o ANALYSIS: State governments cannot be sued in state court without their consent because sovereign immunity is broader than the protections of the Eleventh Amendment. The Court said that the “preeminent purpose” of sovereign immunity is to protect the “dignity” of state governments and that such dignity would be impermissibly offended by allowing states to be named as defendants in agency proceedings without their consent. Federal Maritime Commission v. South Carolina State Ports Authority.
    • o Justice Kennedy acknowledged that the Constitution and its framers were silent about the ability to sue state governments in state courts; but it was unthinkable that the states would have ratified the Constitution had they thought that it made them subject to suit without their consent.
    • o “We hold that the powers delegated to Congress under Article I of the United States Constitution do not include the power to subject nonconsenting States to private suits for damages in state courts.”

Preemption; intro to dormant commerce clauseDormant Commerce Clause Analysis

  • Is the law facially discriminatory? Presumed Invalid.
    • Burden shifts to state under strict scrutiny
    • Do alternative nondiscriminatory means exist to achieve legitimate governmental objective?
    • Is there proof of a legitimate objective?
  • Is the law neutral on its face but discriminatory in purpose or effect? Presumed Invalid.
    • Burden shifts to state under strict scrutiny
    • Do alternative nondiscriminatory means exist to achieve legitimate governmental objective?
    • Is there proof of a legitimate objective?
  • Is law is nondiscriminatory? Presumed Valid.
    • Does it otherwise impose a burden on interstate commerce?
    • Do benefits to government outweigh burdens on interstate commerce?.

What is Discrimination under DCC?

  • Does the challenged statute involve disparate treatment of in-state and out-of-state economic interests?
  • Does it have the same effect on in-state and out-of-state interests?
  • Was it intended to benefit in-state interests at the expense of out of state interests?
  • Is it protectionist toward in-state interests?
  • Does it impose greater economic burdens on those outside the state, to the economic advantage of those within?
  • Does the statute treat in-state and out-of-state interests evenhandedly?
  • Is there a disproportionate adverse effect felt by out-of-staters or a disproportionate advantage enjoyed by in-staters?

Why a DCC?

  • Anti-protectionism
  • Anti-trade war
  • Functioning of interstate commerce
  • While maintaining state autonomy where possible
  • Note: Does not restrict states from creating their own standards assuming they don’t discriminate, have a valid purpose, and do not excessively burden interstate commerce.
    • These state standards can effectively set policy for rest of nation as manufacturers or producers seek common denominator to maximize markets.
    • California Automobile Emission Standards

Limits on state power that derive from the existence of a national government and of other states

  • v There are two possibilities when considering whether a state or local law is invalidated because of these restrictions.
    • o Where Congress has acted.
      • If Congress has passed a law and it is a lawful exercise of congressional power, the question is whether the federal law preempts state or local law. Article VI provides that the “Constitution and the Laws of the United States which shall be made in Pursuance thereof; and all Treaties made, or which shall be made under the Authority of the United States, shall be the Supreme Law of the Land.” Because of the supremacy clause, if there is a conflict between federal law and state or local law, the latter is deemed preempted.
    • o Where Congress has not acted—or at least the judiciary decides that federal law does not preempt state or local law.
      • Even though there is not preemption, state and local laws can be challenged under two principles:
        • Dormant Commerce Clause: the principle that state and local laws are unconstitutional if they place an undue burden on interstate commerce. SCOTUS has inferred this limit on state regulatory power from the grant of power to Congress to regulate commerce among the states. Even if Congress has not acted, even if its commerce power lies dormant, state and local governments cannot place an undue burden on interstate commerce.
        • Privileges and Immunities Clause of Article IV, §2: “The Citizens of each State shall be entitled to all Privileges and Immunities of Citizens in the several States.” SCOTUS has interpreted the clause as limiting the ability of states to discriminate against out-of-staters with regard to constitutional rights or important economic activities. This clause primarily has been applied by the Supreme Court when there have been challenges to state and local laws that discriminate against out-of-staters with regard to their ability to earn a livelihood.

Gade v. National Solid Wastes Management Association, the Court summarized the tests for preemption:“Preemption may be either express or implied, and is compelled whether Congress' command is explicitly stated in the statute's language or implicitly contained in its structure and purpose. Absent explicit preemptive language, we have recognized at least two types of implied preemption: field preemption, where the scheme of federal regulation is so pervasive as to make reasonable the inference that Congress left no room for the States to supplement it, and conflict preemption, where compliance with both federal and state regulations is a physical impossibility, or where state law stands as an obstacle to the accomplishment and execution of the full purposes and objectives of Congress.”

  1. ' Express Preemption'— Whenever Congress has the authority to legislate, Congress can make federal law exclusive in a field. The clearest way for Congress to do this is to expressly preclude state or local regulation in an area. Thus, some federal laws contain clauses that expressly preempt state and local laws.

(454) Lorillard Tobacco Co. v. Reilly

  • v Instant Facts: Massachusetts state regulations governing the advertising and promotion of tobacco products were challenged as preempted by federal regulations governing the advertising and promotion of cigarettes.
  • v Black Letter Rule: When Congress expressly preempts state or local action on a specified matter, any conflicting state regulation must yield to federal law.
  • v Invalidated a Massachusetts law that prohibited outdoor advertising for cigarettes, such as billboards, within 1,000 feet of a playground or school.
  • v SCOTUS relied on a federal law adopted in 1969 that proscribes any “requirement or prohibition based on smoking and health … imposed under State law with respect to the advertising or promotion” of cigarettes.
    • o “In the 1969 amendments, Congress not only enhanced its scheme to warn the public about the hazards of cigarette smoking, but also sought to protect the public, including youth, from being inundated with images of cigarette smoking in advertising. In pursuit of the latter goal, Congress banned electronic media advertising of cigarettes. And to the extent that Congress contemplated additional targeted regulation of cigarette advertising, it vested that authority in the FTC.”
  • v Justice Stevens, dissenting, argued that the state law was not preempted because it regulated the location and not the content of cigarette advertisements.
    • o The majority rejected this distinction: “But the content/location distinction cannot be squared with the language of the pre-emption provision, which reaches all ‘requirements' and ‘prohibitions' ‘imposed under State law.’ A distinction between the content of advertising and the location of advertising in the FCLAA also cannot be reconciled with Congress' own location-based restriction, which bans advertising in electronic media, but not elsewhere.” The majority said that general zoning regulations limiting outdoor advertisements could be applied to cigarettes, but not laws targeting just tobacco products.
  • v Riegel v. Medtronic
    • o The Medical Devices Amendments of 1976, 21 U.S.C. §360k, preempt states from imposing “requirements” different from federal law after the FDA approves a medical device.
    • o The Court held that this preempts state tort liability against manufacturers for devices approved by the FDA. The Court reasoned that tort liability, like regulation, changes behavior and essentially creates requirements. Justice Ginsburg was alone in dissent and stressed that there should be a presumption against preemption. She said that if Congress wanted to preempt tort liability, it could do so, but that this law only preempted states from imposing “requirements.”
  • v Chamber of Commerce v. Whiting
    • o The Court considered a provision of federal immigration law that expressly preempts “any State or local law imposing civil or criminal sanctions (other than through licensing and similar laws) upon those who employ . . . unauthorized aliens.”
    • o An Arizona statute—the Legal Arizona Workers Act—provides that the licenses of state employers that knowingly or intentionally employ unauthorized aliens may be, and in certain circumstances must be, suspended or revoked.
    • o The Court concluded that the Arizona law was not preempted.
      • “Because we conclude that the State’s licensing provisions fall squarely within the federal statute’s savings clause and that the Arizona regulation does not otherwise conflict with federal law, we hold that the Arizona law is not preempted. When a federal law contains an express preemption clause, we ‘focus on the plain wording of the clause, which necessarily contains the best evidence of Congress’ preemptive intent.’ [Federal law] expressly preempts States from imposing ‘civil or criminal sanctions’ on those who employ unauthorized aliens, ‘other than through licensing and similar laws.’ The Arizona law, on its face, purports to impose sanctions through licensing laws. The state law authorizes state courts to suspend or revoke an employer’s business licenses if that employer knowingly or intentionally employs an unauthorized alien.”
  1. Implied Preemption— 1. Conflicts Preemption
  • Florida Lime v. Paul (1963) [Page 461]
    • o FACTS: A Florida avocado producer challenged the constitutionality of a California statute that was more restrictive than federal regulations.
    • o RULE: When both a state and federal regulation can be satisfied, federal law does not preempt the state regulation.
    • o ANALYSIS: The Court concluded—that the federal regulation was the floor, but not the ceiling; it was the minimum standard, but states were allowed to set stricter regulations if they wished.
    • o Numerous other cases reflect the same problem: When states set stricter standards in an area than does federal law, it is necessary to decide whether the federal government meant its law to be exclusive or only intended to set a minimum standard that states may exceed. Unfortunately, almost never does the statute or regulation answer this question and rarely is there any legislative history on point. Thus, mutual exclusivity preemption, like the other areas of preemption, involves the Court making a judgment about what Congress or a federal agency intended when they said nothing about their intent.
  1. Implied Preemption— 2. Preemption Because State Law Impedes the Achievement of a Federal Objective
  • Pacific Gas & Electric Co. v. State Energy Resources Conservation (1983) [Page 462]
    • o FACTS: State law, which effectively placed a moratorium on construction of new nuclear power plants within the state, was not preempted by federal law that governed the regulation of safety aspects concerning nuclear power plants.
    • o RULE: State law is preempted if it stands as an obstacle to the accomplishment of the full purposes and objectives of Congress; however, the Court will not interfere where there is a permissible basis for the state law.
    • o ANALYSIS: SCOTUS rejected both preemption arguments and upheld the state law by concluding that Congress's intent was to ensure safety, while the state's goal was economic. Held federal law did not preempt state tort actions against state utilities.
      • As to the field preemption argument, the Court said Congress intended that the federal government have exclusive authority to regulate safety, “but that the States retain their traditional responsibility in the field of regulating electrical utilities for determining questions of need, reliability, cost, and other related state concerns.”
    • The Court concluded that the California law was not preempted because its main purpose was economics and not safety.
      • “California has maintained, and the Court of Appeals agreed, that [the law] was aimed at economic problems, not radiation hazards.… Without a permanent means of disposal, the nuclear waste problem could become critical, leading to unpredictably high costs to contain the problem or, worse, shutdowns in reactors.” The Court said that because it “accept[s] California's avowed economic purpose … the statute lies outside the occupied field of nuclear safety regulation.”
    • The utility also argued that the state law was preempted because it impeded the federal goal of encouraging the development of nuclear reactors as a source of electrical power.
      • The Court acknowledged that “[t]here is little doubt that a primary purpose of the Atomic Energy Act was, and continues to be, the promotion of nuclear power.” Yet the Court rejected the preemption argument by characterizing Congress's purpose as encouraging nuclear power only to the extent that it was economically feasible. “Congress has left sufficient authority in the States to allow the development of nuclear power to be slowed or even stopped for economic reasons.”
    • Thus, in determining whether the California law interfered with achieving the federal objective, the Court had to make two major choices: One was in characterizing the federal objective; the other was in characterizing the state law and its purpose.
      • If the Court saw a broad purpose for the Atomic Energy Act in encouraging the development of nuclear power, then the state law, which obviously limited it, would be preempted. The Court avoided preemption by more narrowly characterizing the federal goal as promoting nuclear reactors only when they were economically feasible.
      • Additionally, if the Court characterized California's purpose as ensuring safety before construction of nuclear power, then the law would have been preempted. The Court avoided preemption by accepting California's claim that its goal was economics, even though the law was written in terms of preventing construction of nuclear plants unless the safety of disposal was ensured.

Implied preemption in the context of whether drug companies can be sued for the failure to warn patients of potential side effects, even though the FDA approved the warning labels.

  • Wyeth v. Levine
    • Held that a drug company could be sued on a failure to warn theory even though its warning label had been approved by the FDA. There is no conflict between allowing such tort liability and federal law; drug companies always can engage in more speech to warn consumers of side effects. The Court said that allowing liability would further, not undermine, the federal regulatory goal of drug safety and well-informed patients and physicians.
  • PLIVA v. Mensing
    • Held that makers of generic drugs may not be sued on a failure to warn theory. Concluded that the Hatch-Waxman Amendments to the Food and Drug Act allow generic drugs to be sold if they are the equivalent of nongeneric drugs and as long as they have the same warning label as the FDA has approved for nongeneric drugs. The Court said that this precludes makers of generic drugs from changing their labels and thus preempts state tort suits for failure to warn.
  • Mutual Pharmaceutical Co. v. Bartlett
    • A woman took the generic form of the pain reliever sulindac. She suffered a rare, though known side effect: two-thirds of the skin on her body burned, blistered, and decayed. She sued under state tort law claiming a design defect.
    • In a 5-4 decision, the claim was preempted. The Court said that under the Hatch-Waxman Act a generic drug can be sold if it is chemically the same as a brand name drug and if it has the warning label that was approved for the brand name drug. The Court explained that the generic drug company cannot change the drug’s chemical composition and cannot change the warning label (as held in PLIVA v. Mensing) and therefore liability is preempted.
  1. Preemption Because Federal Law Occupies the Field

Even though federal law does not expressly preempt state law, preemption will be found if there is a clear congressional intent to have federal law occupy a particular area of law. The most important example of this is immigration law.

  • v Hines v. Davidowitz
    • o A Pennsylvania law required aliens to register with the state, carry a state-issued registration card, and pay a small registration fee. The Supreme Court deemed this law preempted by emphasizing that alien registration “is in a field which affects international relations, the one aspect of our government that from the first has been most generally conceded imperatively to demand broad national authority.” The Court stressed the extensive federal regulation in the area, including a “broad and comprehensive plan describing the terms and conditions upon which aliens may enter this country, how they may acquire citizenship, and the manner in which they may be deported.” Indeed, a federal law specifically required alien registration with the federal government.
    • o Two aspects of Hines are particularly noteworthy.
      • First, the Court found preemption of a state law that complemented the federal law; the state law in no way interfered with the federal law or its implementation. The Court said that states cannot “contradict or complement” federal immigration law.
      • Second, the Court found field preemption in Hines even in the absence of express preemptive language in the federal statute. Congress certainly could have explicitly preempted state alien registration in the federal law that required aliens to register. But Congress did not do this.
  • Arizona v. US (2012) [Page 462]
    • o FACTS: Hundreds of thousands of deportable aliens are apprehended in Arizona (D) every year, and accounts in the record suggested an “epidemic” of crime, safety risks, property damage, and environmental problems associated with illegal migration across private land near the Mexican border. In response to these issues, Arizona (D) enacted SB 1070, which criminalized violations of some provisions of federal immigration laws. Section 3 of SB 1070 made it a misdemeanor for an alien to fail to comply with federal alien registration laws. Section 5 of the law also made it a misdemeanor for an unauthorized alien to seek or engage in work in Arizona (D), and it gave arrest and investigative authority to law enforcement officers. Section 6 authorized officers to make an arrest without a warrant if the officer had probable cause to believe the person being arrested had committed a public offense that made the person removable from the United States, and section 2(B) provides that officers who conduct stops, detentions, or arrests must make efforts to verify the person’s immigration status. The United States (P) brought suit to enjoin enforcement of SB 1070, claiming that federal immigration law preempted state laws on the subject. The district court granted a preliminary injunction, and the Ninth Circuit Court of Appeals affirmed.
    • o RULE: When Congress has occupied an entire field, complementary state regulation is impermissible, even if that state regulation parallels federal standards.
    • o ANALYSIS: In a 5-3 ruling, affirmed the district court's preliminary injunction as to three of these provisions.
      • Justice Kennedy began by accepting the argument of the U.S. that immigration is solely in the control of the federal government. Anything done with regard to immigration has foreign policy implications and states cannot have their own foreign policy.
        • The Court quoted Hines that states cannot “contradict or complement” federal immigration efforts and “[o]ne of the most important and delicate of all international relationships … has to do with the protection of the just rights of a country's own nationals when those nationals are in another country.”
      • Found that §3 was preempted based on Hines, which had held that alien registration was exclusively in the province of the federal government.
      • The Court found the criminal prohibition against those unlawfully in the country from seeking or receiving employment in Arizona (§5C) was preempted based on Congress's explicit choice to impose sanctions on employers and not employees.
      • The Court found that §6 is preempted because it was solely in the discretion of the federal government to decide whether to detain a person for possible deportation.
        • “By authorizing state officers to decide whether an alien should be detained for being removable, §6 violates the principle that the removal process is entrusted to the discretion of the Federal Government . . . Decisions of this nature touch on foreign relations and must be made with one voice.”
      • Finally, the Court reversed the preliminary injunction as to the provision that allows police to question individuals about their immigration status if they are stopped for other reasons and if there is reasonable suspicion that they are not lawfully in the United States.
        • The Court stressed that it was too uncertain as to how this would be implemented to allow it to decide constitutionality at this stage. “At this stage, without the benefit of a definitive interpretation from the state courts, it would be inappropriate to assume §2(B) will be construed in a way that creates a conflict with federal law.”
        • But this provision was substantially narrowed as the Court said that police cannot extend the duration of a stop to check immigration status and also that state and local police cannot arrest individuals whom they determine to be illegally in the country. Moreover, the Court left open the possibility of an “as applied” challenge to this provision of S.B. 1070 if it could be shown that it was being applied in a racially discriminatory fashion.

Dormant commerce clause; privileges and immunities under Article IVThe Dormant Commerce Clause isn't really a constitutional clause at all; rather the Dormant Commerce Clause refers to the implied restriction created by the Commerce Clause that limits states' ability to burden or discriminate against interstate commerce. The purpose of this restriction is to promote free and unrestricted interstate commerce and national unity. The notion that the grant of power in the Commerce Clause contains an inherent, corresponding limit on state power is not uncontroversial, and the constitutionality of the concept is still debated today. Well, courts today now use a combination of two tests: rational basis review plus a balancing test. State laws will satisfy rational basis review if they are rationally related to a legitimate state goal. In addition, courts will weigh the state's interest against the burden imposed on interstate commerce. Courts applying this test will first consider the legitimacy of the state's purpose.Typically, regulations aimed at promoting the general welfare will be considered to serve a legitimate end; laws designed to protect or promote the economic prospects of in-state residents at the expense of out-of-state residents are much less likely to be found to have a legitimate goal. And since it's unlikely that a state would come out and say that the purpose of a given law is to discriminate against interstate commerce in favor of state residents, courts will look beyond what the law says on its face and try to ascertain its true intent.

  • H.P Hood and Sons v. Du Mond (1949) [Page 477]
    • o FACTS: The Court declared unconstitutional a New York law that prevented a company from constructing an additional depot for receiving milk. The effect of the New York law was to keep more milk for in-staters at the expense of those in Massachusetts.
    • o RULE: States may not enact laws that burden the exportation of local products in order to protect and advance local economic interests.
    • o ANALYSIS: This case presents the traditional arguments for having a dormant Commerce Clause.
      • First, a historical argument, the framers intended to prevent state laws that interfered with interstate commerce.
      • Second, an economic justification, the economy is better off if state and local laws impeding interstate commerce are invalidated.
      • Third, there is a political justification for the dormant Commerce Clause: States and their citizens should not be harmed by laws in other states where they lack political representation. See McCulloch v. Maryland.
  • Cooley v. Board of Wardens (1851) [Page 483]
    • o National/Local Subject Matter Test
    • o FACTS: A Pennsylvania law required all ships entering or leaving the Port of Philadelphia to use a local pilot or to pay a fine, which went to support retired pilots.
    • o RULE: The Congressional power to regulate commerce is not exclusive of all state powers to regulate commerce.
    • o ANALYSIS: Important case in which the Court drew a distinction between subject matter that is national, in which state laws are invalidated under the dormant commerce clause, and subject matter that is local, in which state laws are allowed.
    • o The Court found that regulating pilots was a local matter both because of differences among ports and also because a federal law adopted in 1789 expressly allowed states to regulate piloting.
    • o There are, however, several problems with the Cooley
      • First, it allows state regulations, no matter how protectionist or how much they interfere with interstate commerce, so long as the subject matter is deemed local.
        • In Cooley, the state's obvious goal was to help local pilots at the expense of out-of-state pilots. If the goal was to protect safety, this could have been accomplished by requiring a license, based on experience or a test, in order to pilot a ship into the Port of Philadelphia. Moreover, the concern for safety is belied by the law's allowing nonlocal pilots to be used if a fee was paid to a fund that benefited local pilots.
      • Second, there is not a clear distinction between what is national, demanding local regulation, and what is local, requiring diverse regulation.
      • Cooley articulates no criteria for making this determination, and it seems inherently arbitrary; in almost any area there likely are some benefits from national uniformity and some gains from local diversity.

Many cases applied the Cooley test throughout the nineteenth century and into the twentieth century.

  • v Welton v. Missouri
    • o The Court used the Cooley approach to invalidate a law that required peddlers of out-of-state merchandise to pay a tax and obtain a license, whereas no similar requirements existed for in-state merchants. The Court said that “transportation and exchange of commodities is of national importance, and admits and requires uniformity of regulation.”
  • v Wabash, St. Louis & Pacific Ry. Co. v. Illinois
    • o The Court used the Cooley approach to invalidate a state law that regulated railway rates for goods brought to or from other states. The Court emphasized that there would be enormous burdens on interstate commerce if all states adopted such laws and thus concluded that it was an area that required national uniformity and not local regulation.
  • v Smith v. Alabama
    • o The Court upheld a state law requiring that all locomotive engineers operating in the state be licensed by a state board of examiners
  • v Erb v. Morasch
    • o The Court upheld a city’s ordinance that restricted train speed within the city.
  • v Atchison Topeka & Santa Fe Ry. Co. v. Railroad Commn.
    • o The Court upheld a state law that required electric headlights of prescribed brightness on all trains operating within the state.
  1. Shifting to a modern balancing approach in the Dormant Commerce Clause
  • v The modern approach is based not on rigid categories but, rather, on courts balancing the benefits of a law against the burdens that it imposes on interstate commerce. But the Court never has expressly overruled any of the earlier tests and sometimes invokes them in explaining a particular result.
  • v The Court's shift to a balancing approach in dormant commerce clause analysis is evident from comparing two cases, Barnwell Bros. and Southern Pacific Co.
  • South Carolina State Highway Department v. Barnwell Bros, Inc. (1938) [Page 484]
    • o RULE: A state law placing width and weight limitations on trucks operating on state highways does not impose an unconstitutional burden on interstate commerce so as to violate the Commerce Clause.
    • o ANALYSIS: Upheld a state law that imposed length and width requirements for trucks operating in the state. The Court emphasized the state's important interest in protecting highway safety and in preserving its roadways.
  • Southern Pacific Co. v. Arizona ex rel. Sullivan, Attorney General (1983) [Page 486]
    • o FACTS: Declared unconstitutional a state law that limited the length of railroad trains, to 14 passenger or 70 freight cars, operating in the state. “viewed as a safety measure, [it] affords at most slight and dubious advantage, if any, over unregulated train lengths.”
    • o RULE: In deciding whether a state law—created for its safety measures—violates the Commerce Clause, the court will balance the benefits of the law against the burdens it imposes on interstate commerce.
    • o ANALYSIS: A balancing test: “Hence the matters for ultimate determination here are the nature and extent of the burden which the state regulation of interstate trains, adopted as a safety measure, imposes on interstate commerce, and whether the relative weights of the state and national interests involved are such [as to make the law permissible].”
    • o The difference is that in Barnwell, the Court believed that the burdens on interstate commerce were outweighed by the benefits in terms of road safety; whereas in Southern Pacific, the Court decided that the burdens on interstate transportation were greater than the safety benefit to the state from its law. In other words, the central issue in dormant commerce clause cases is whether the benefits of the state law outweigh its burdens on interstate commerce.
    • o Such a balancing test gives courts enormous discretion because it is attempting to weigh and compare two completely different things: burdens on interstate commerce and the benefits to a state or local government.

Balancing Test Used Depends on Whether There Is Discrimination The way in which the Court balances is not the same in all dormant commerce clause cases, but instead varies depending on whether the state or local law discriminates against out-of-staters or treats in-staters and out-of-staters alike.

  • v State law is discriminatory to out-of-staters -> strong presumption against the law and upheld only if it is necessary to achieve an important purpose
    • o There are two exceptions where laws that otherwise would violate the dormant commerce clause will be allowed: congressional approval and the market participant exception.
  • v State law is nondiscriminatory to out-of-staters -> presumption to uphold and it will be invalidated only if its burdens on interstate commerce outweigh its benefits

Criticism of the Balancing Approach Some Justices—most notably Rehnquist, Scalia, and Thomas—have objected to this balancing test and have argued in favor of upholding all state laws that are deemed nondiscriminatory. Scalia contended: “Weighing the governmental interests of a State against the needs of interstate commerce is, by contrast, a task squarely within the responsibility of Congress, and ill-suited to the judicial function.”

  1. Determining Whether a Law Is Discriminatory

Facially Discriminatory Laws

  • City of Philadelphia v. NJ (1978) [Page 489]
    • o FACTS: A New Jersey law effectively kept scarce landfill space in the state exclusively for New Jersey's use by preventing the importation of any wastes from out of state.
    • o RULE: State laws that regulate commercial activity may not, on their face or in effect, favor in-state interests over out-of-state interests.
    • o ANALYSIS: “where simple economic protectionism is effected by state legislation, a virtually per se rule of invalidity has been erected.” Expansively declared that “all objects of interstate trade merit Commerce Clause protection.” If the state or local law affects interstate commerce, then the dormant commerce clause may be applied. The key initial question is whether the state law discriminates against out-of-staters or whether it treats in-staters and out-of-staters alike.
  • Hunt, Governor of the State of NC v. Washington State Apple Advertising Commission (1977) [Page 492]
    • o FACTS: A North Carolina law required that all closed containers of apples sold or shipped into the state bear “no grade other than the applicable U.S. grade or standard.” The law was facially neutral in that all apples sold in the state—whether produced in state or out of state—had to comply with this rule.
    • o RULE: A facially neutral state law is unconstitutional and violates the Commerce Clause if it has a discriminatory effect on interstate commerce.
    • o ANALYSIS: The Court found that the law should be treated as discriminatory because of its effect on the sale of Washington apples. Washington had a system for grading apples that was different from and more stringent than the federal standard.
      • “The challenged statute has the practical effect of not only burdening interstate sales of Washington apples, but also discriminating against them . . . The first, and most obvious, is the state's consequence of raising the costs of doing business in the North Carolina market for Washington apple growers and dealers, while leaving those of North Carolina counterparts unaffected.… Second, the statute has the effect of stripping away from the Washington apple industry the competitive and economic advantages it has earned for itself through its expensive inspection and grading system.… Third, by prohibiting Washington growers and dealers from marketing apples under their State's grades, the statute has a leveling effect which insidiously operates to the advantage of local apple producers.”
  • Exxon Corp v. Governor of MD (1978) [Page 495]
    • o FACTS: A Maryland law prohibited a producer or refiner of petroleum products from operating a retail service station within the state. Because virtually all petroleum products sold in Maryland were produced and refined out of state, the law meant that these out-of-state oil companies could not own service stations in Maryland. The obvious beneficiary was local businesses.
    • o RULE: A state law that causes some business to shift from one interstate supplier to another does not impermissibly burden interstate commerce.
    • o ANALYSIS: The majority declared: “[T]he Act creates no barriers whatsoever against interstate independent dealers; it does not prohibit the flow of interstate goods, place added costs upon them, or distinguish between in-state and out-of-state companies in the retail market. The absence of any of these factors fully distinguishes this case from those in which a State has been found to have discriminated against interstate commerce.”
  • West Lynn Creamery, Inc. v. Healy (1994) [Page 499]
    • o An example in which a court deemed discriminatory a facially neutral tax law.
    • o FACTS: Massachusetts imposed a tax on all milk dealers, but the funds from the tax went into a fund to pay subsidies to in-state dairy farmers.
    • o RULE: A state pricing order, which imposes a tax on the sale of local products, the proceeds of which are distributed as a subsidy to in-state producers of the product, discriminates against interstate commerce and thus is unconstitutional under the Commerce Clause.
    • o ANALYSIS: The state was taxing both in-staters and out-of-staters, but in effect refunding the taxes paid by in-staters through the subsidy system. The net effect was that the tax was borne disproportionately by out-of-staters and thus was unconstitutional.
  • State of Minnesota v. Clover Leaf Creamery Co. (1951) [Page 501]
    • o FACTS: A Minnesota law prohibited the sale of milk in plastic disposable containers, but allowed its sale in paper disposable containers. The law had a substantial discriminatory effect in favor of in-state businesses and against out-of-state businesses because Minnesota had a substantial paper industry but no plastics industry. The law thus created a demand for paper products traditionally produced in state and prevented the out-of-state plastics industry from having access to this Minnesota market. Moreover, the state trial court found that the “actual basis” of the statute “was to promote the economic interests of certain segments of the local dairy and pulpwood industries at the expense of the economic interests of other segments of the dairy industry and the plastics industry.”
    • o RULE: A facially neutral state law will violate interstate commerce if the incidental burden imposed on interstate commerce is clearly excessive in relation to the putative local benefits.
    • o ANALYSIS: “Minnesota's statute does not effect ‘simple protectionism,’ but ‘regulates evenhandedly’ by prohibiting all milk retailers from selling their products in plastic, nonreturnable milk containers, without regard to whether the milk, the containers, or the sellers are from outside the State. This statute is therefore unlike statutes discriminating against interstate commerce, which we have consistently struck down.”
    • o In cases like Exxon, and Clover Leaf Creamery Co., proof of discriminatory impact, even with evidence of a protectionist purpose, was insufficient for the Court to deem the law discriminatory. The cases turned not on differences about the rule, but on the Court's appraisal of the particular facts and its assessment of whether there was discrimination.
    • o In other words, proof of either a protectionist purpose for the law or a substantial discriminatory impact is sufficient to establish that a law is discriminatory. A court will assess each situation and decide whether there is sufficient evidence of discriminatory purpose and/or effect. Although there are no clear criteria for this inquiry, several factors seem particularly important.
      • First, a law is likely to be found discriminatory if its effect is to exclude virtually all out-of-staters from a particular state market, but not if it only excludes one group of out-of-staters.
        • In Exxon, only out-of-state petroleum producers and refiners were kept from operating in the state; other out-of-staters could own service stations in the state. In Clover Leaf Creamery Co., out-of-state plastics industry was disadvantaged, but out-of-state paper companies could sell milk containers in the state.
      • Second, a law is likely to be found discriminatory if it imposes costs on out-of-staters that in-staters would not have to bear.
        • In Hunt, the Court emphasized the costs imposed on Washington apple producers compared to the North Carolina apple industry.
      • Third, the Court is more likely to find discrimination if it believes that a law is motivated by a protectionist purpose, helping in-staters at the expense of out-of-staters. The Court often has said that the central purpose of the dormant commerce clause is to prevent such protectionist legislation.
        • In sum, a law will be found discriminatory either if it facially discriminates against out-of-staters or if it is facially neutral and is deemed to have a discriminatory purpose and/or impact.
  1. Analysis If a Law Is Deemed Discriminatory
  • Dean’s Milk Co. v. Madison (1983) [Page 503]
    • o FACTS: A city's ordinance required that all milk sold in the city had to be pasteurized within five miles of the city. The law prevented milk that was pasteurized in other states from being sold in the city, but it also precluded milk that was pasteurized in other parts of that state from being sold in the city.
    • o RULE: Where reasonable and adequate alternatives are available, a local health ordinance that places a discriminatory burden on interstate commerce violates the Commerce Clause.
    • o ANALYSIS: The law discriminated against milk from other states, notably Illinois, and explained that Madison could achieve its goal of ensuring safe milk by less discriminatory alternatives such as by sending its inspectors to importing producers or by relying on inspections by federal authorities.
      • In a footnote, the Court said that it was irrelevant that the law also discriminated against in-staters: “It is immaterial that Wisconsin milk from outside the Madison area is subjected to the same proscription as that moving in interstate commerce.”
  • Maine v. Taylor and US (1986) [Page 504]
    • o FACTS:
    • o RULE: State statute that affirmatively discriminates against interstate commerce passes vigorous strict scrutiny test where it attempts to prohibit significant damage to State’s environmental well-being.
    • o ANALYSIS:
  1. Analysis If a Law Is Deemed Nondiscriminatory
  • Loren J. Pike v. Bruce Church, Inc. (1970) [Page 507]
    • o FACTS: The Court invalidated an Arizona regulation that required cantaloupes grown there to be packed in the state rather than in another state. The Court said that it “has viewed with particular suspicion state statutes requiring business operations to be performed in the home State that could more efficiently be performed elsewhere.”
    • o RULE: Where a state statute regulates even-handedly to effectuate a legitimate local public interest, and its effects on interstate commerce are only incidental, it will be upheld unless the burden imposed on such commerce is clearly excessive in relation to the putative local benefits.
    • o ANALYSIS: The Pike Test; here, the law imposed a burden on interstate commerce that exceeded the putative local benefits.
      • Where a nondiscriminatory law effectuates a legitimate local interest and its effects on interstate commerce are only incidental, it will be upheld unless the burden imposed on interstate commerce is clearly excessive in relation to the putative local benefits. Where there is a legitimate local interest it must be measured against the burden it composes.
    • Bibb v. Navajo Freight Lines (1959) [Page 509]
      • o FACTS: The Court declared unconstitutional a state law that required that all trucks in the state use curved mudguards to prevent spatter and enhance road safety. It was “one of those cases—few in number—where local safety measures that are nondiscriminatory place an unconstitutional burden on interstate commerce.”
      • o RULE: In determining whether a state’s nondiscriminatory highway safety law violates the dormant Commerce Clause, the court will uphold the law unless the total effect of the law as a safety measure is so slight or problematical as not to outweigh the national interest in keeping interstate commerce free from interferences that seriously impede it.
      • o ANALYSIS: The Court found that the law put a substantial burden on interstate commerce because straight mudguards were legal in 45 other states and curved mudguards were illegal in one other state. Trucks would have to either avoid Illinois or stop at the border to change their mudguards. Moreover, the trial court found that curved mudflaps have “no” safety benefits over straight ones and actually create “hazards previously unknown” by increasing the heat around the truck's tires.
  • Consolidated Freightways Corp. of Delaware v. Raymond Kassel (1982) [Page 511]
    • o FACTS: The Court declared unconstitutional an Iowa law banning 65-foot double trailers. The Court again weighed the “asserted safety purpose against the degree of interference with interstate commerce.” The Court said that the “State failed to present any persuasive evidence that 65-foot doubles are less safe than 55-foot singles.… Statistical studies supported the view that 65-foot doubles are at least as safe overall as 55-foot singles and 60-foot doubles.”
    • o RULE: Although state regulations concerning highway safety carry a strong presumption of validity, if the furtherance of safety is marginal or the burden on commerce is substantial, the regulations will be declared invalid under the Commerce Clause.
    • o ANALYSIS: Moreover, the Court found that the Iowa law “substantially burdens interstate commerce” by forcing these trucks to avoid Iowa or to detach the trailers and ship them separately. These cases indicate that the Court will evaluate state laws restricting truck size on a case-by-case basis considering the specific evidence as to the safety benefits of the laws compared to their burden on interstate commerce. The same is true when the Court evaluates state laws limiting train size, although there is probably less of a presumption of deference when a state is regulating railroads as compared to highways.